You are on page 1of 123

Passage 1-Passage10 解析

passage 1
文章结构分析:文章首句作为背景,引出主题“基于发展情景中鱼的社会认知研究(相对于个体研究)
的不足”。第二句重点引入 C,W, K 的研究:关注鱼群密度与认知学习(后续见食成功)的关系。他们做
对比实验:小群中长大的 G 鱼比大群中的 G 鱼更快发现食物。后文进一步解释:大群中成长的 G 鱼
更少学习。总之,该研究验证了社会环境与认知的关系。
1.答案:B
题型:目的主旨题
解析:该题问文章主旨目的,根据文章结构分,可知 B 最准确。
A 选项错在 flaw;,
C 选项没有提到文章前两句关于实验的内容,只说到了后面的结论,所以不如 B 选项完整。(此题
C 选项最具诱惑性)
D 选项错,文章就介绍了一个研究发现。
E 选项只概括了研究中的对比。
2.答案:D
题型:推理题
解析:该题只能选项定位。D 选项可以定位到文章首句,是其同义改写: Although social learning (the
acquisition of specific behaviors by observing other individuals exhibiting those behaviors) is well
documented among fish, few studies have investigated social learning within a developmental context in
these taxa.
3.答案:D
题型:推理题
解析:根据人名定位到第二句之后。D 选项正确。
文章说,他们的实验是针对 laboratory-housed guppies 做的,文章最后一句总结是针对所有 G 鱼类
说的(包括野生 G 鱼)可知作者默认了 D 选项正确,否则不能得出最后一句结论。

passage 2
文章结构分析:文章首段主要介绍一个矛盾现象: Hurston 写了两个版本,1935 年版本和 1929 年版本。
矛盾的是: 35 年发表的版本不同于最初的 29 年版本。第二段给出原因解释,进而作出评论。
1.答案: “While we cannot know for certain why Hurston' s original manuscript went unpublished during her
lifetime, it may have been because publishers wanted something more than a transcription of tales.”
题型:句子功能选择题
解析: 由对矛盾现象的解释可以定位到第二段的第一句,即答案所在句。
2.答案:E
题型:推理题
解析:original version 指的是 1929 年未发表的版本。
A 选项中与编辑交流在文中早不到依据。而且作者推测 1935 年版本才是与编辑交流的结果;
B 选项中最小化 Hurston 自已的陈述以最大化材料,在文中找不到对应的依据;
C 选项说将手稿放置一边,以期在将来出版时赢得最多潜在的读者,在文中也无依据。而且 1929
年版本在当时没有被出版的,因此 Hurston 自己也不可能预计以后的出版情况;
D 选项说 Hurston 不情愿的删除了一些编辑不满意的因素,文中也没有提到。只是作者推测 1935
年版木的某些内容是应编辑的要求添加的;
选项 E 说 Hurston 决定不添加评论内容, 还原故事本来面貌, 对应文中“This version differs from Mules
and Men in that it simply records stories, with no descriptive or interpretive information."”其中 this version
指的是 1929 年版,即 original version。

passage 3
文章结构分析:文章中心结论第二句后半句: so psychologists have speculated that sharply increasing
Internet use can cause depression 增加上网时间会引起抑郁。
文章理由以该句结论为界,分为前后两个部分:
前半部分理由看起来是一个链状推理:减少面对面社交会引起抑郁。用来上网的时间不能用于面对面
社交。
后半部分理由是一个对比研究:最近增加两倍上网时间的常规网络用户比没有增加的常规网络用户有
更高的抑郁发生率。
答案:C
题型:逻辑单题假设题
解析:题目问文章论证基于的假设。
前后两半部分理由哪一部分有漏洞呢?问题出在前半部分!它貌似一个链状推理,但它与严密支持结
论的链状推理之间是有出入的。
这样一个链状推理才是严密的,无懈可击的:
理由 1:减少面对面社交会引起抑郁
理由 2:增加上网的时间会减少面对面社交时间
结论:增加上网时间会引起抑郁
问题:即使用来上网的时间不能用于面对面社交(原文表达),但是否能增加每天其他时间的面对面社
交机会呢? 比如,上网用来 dating,上网之后意犹未尽,相约面对面热聊。如果出现这种情况,原文
推理就不是我们需要的严密推理。所以,文章推理依据的假设就是要排除这种情况。就是 C 选项。
A 选项针对文章后半段的对比研究,也是争议最大的选项。
A 选项取非:研究中增加两倍上网时间的网络用户之前经历了面对面交流机会的严重减少,暗示说这
些人实验之前就是抑郁的,似乎试图颠倒对比研究中增加上网与得抑郁之间的因果关系。但 A 选项
“earlier”并未明确刚好在实验研究之前,“opportunities for face-to-face social contact”也不等于实际
的面对面交流。因此 A 选项取非并不能颠倒对比实验的因果关系。

passage 4
文章结构分析:文章两段是典型的对比结构。
第一段是美国黑人戏剧最近之前的主导流派:自然主义。自然主义的典型特征是反映现实生活,人物
命运决定于现实环境。
第二段是以 AK 为代表的最近变化:超现实主义,远离自然主义。塑造人物不依赖于与外部环境的联
系。
1. 答案:D
题型:主旨题
解析:
该题问文章主旨,D 最准确。
A 选项只概括了首段内容;
B 选项概括主题偏离,没有体现对比变化;
C 选项无视二段内容;
E 选项 best 概念原文无,且没有体现一段。
2. 答案:A
题型:细节题
解析:该题问 K 的艺术主张。
A 选项定位到第 2 段第二句: The overall goal of her work has been to depict the world of the soul and the
spirit, not to mirror concrete reality。由此可知选 A。
3.答案:C
题型:加强题
解析:该题问最能加强作者论断( K 的作品远离首段提及的自然主义传统)的选项。C 最合适。K 只使
用古代神话传说中人物形象。意思是 K 的人物角色不依赖于周围环境。这与首段自然主义风格最相反:
Naturalism see seach individual as inextricably bound to the environment and depicts each person as
someone controlled by, instead of controlling, concrete reality.

passage 5
文章结构分析:文章首段首先介绍一个理论难题:关于 invasibility 原因没有统一意见。第二句 However
后,作者指出,一种基于资源波动的理论能解决该理论难题。第二段详细介绍该理论。该理论主要用
resource-release mechanisms 来解释 invasibility,否定物种丰富性的解释。
1.答案:C
题型:目的主旨题
解析:
这道题是典型的主旨题。
选项 A 说是评价一个理论的实验观察上的成功。但原文中并没有讲到实验观察的地方,而是理论上
的推测;
选项 B 说解释了为什么某一现象没有一致的理论原因,但文章主要讲了新理论如何解决旧理论的不
一致性,而不是“为什么”的问题;
选项 C 说支持了一种有潜力可以解决原有理论僵局的新理论,符合题意;
选项 D 说由一个理论推测出了可以被检测的推论,但文中并没有提到推论,也没有提到检测问题;
选项 E 说描述了解释某些特定实验结果的困难。原文中只由第一段说某些原有假说与实验结果不一
致,并非全文都在描述困难。
2.答案:B
题型:推理题
解析:根据 field studies 定位到文章首段,however 转折后,previous studies 仍指 field studies。选项 B
是对转折后的概括。
3.答案:E
题型:细节题
解析:根据高亮关键词定位首段。尽管关键词出现在首段,因二段是对首段理论的详细展开,答案很
可能出现在二段。E 选项定位该句 since 原因从句:In particular, the theory predicts that there will be no
necessary relationship between the species diversity of a plant community and its susceptibility to invasion,
since near-complete exploitation can each occur in both species-rich and species-poor communities.该句用
来否定物种多样性的解释,因为不同的(物种丰富)情况会有一样的结果:(外来物种)的几乎彻底
开发利用。

passage 6
文章结构分析:文章首句指出最近研究与之前怀疑者观点的矛盾:怀疑者认为北极地区降温,最近研
究发现升温。后文依次列举方式解释了几个原因:1)雪和冰的反射, 2)空气稀薄 3)少蒸发损耗。
1.答案:ABC
题型:细节题
解析:该题问可能导致地球极地温度大幅变化(swing)因素,定位到文章中第 2 句:several factors lead
to greater temperature swings at Earths polar regions。可知 ABC 都正确。
2.答案:C
题型:细节题
解析:该题问文章中 skeptics (猜疑者)指出格棱兰地区温度变化的目的,定位到首句,可知 C 选项:他
们认为温度是向相反方向变化的( reversal of the temperature trend) ,他们认为以格陵兰为代表的整个北
极地区温度是下降的,这与主流观点 recent scientific report 的结论是相反的。B 选项不准确,等于把
格陵兰当作北极地区的例外特例,而不是原文表达的典型代表。

passage 7
文章结构分析:全文首句介绍很多历史学家对 CM 的传记的观点:关注虔诚性,牺牲历史准确性。
Yet 转折后,是作者对 CM 传记的正面评价,反驳了对 CM 的指责。
1.答案:D
题型:主旨题
解析:
该题 A 选项说反驳了一个历史界通行的理论,但文中并没有提到任何理论;
B 选项说是一个不同寻常的记录历史时代的方法,但文中显然也没有提到任何方法;
C 选项说总结了关于一个历史人物的研究,但显然文章并没有全部在说某一个历史人物的研究工
作。文中提到了 Governor John Winthrop,目的是为了说明 Mather 的 critical observations;
D 选项说反驳了关于某个传记作家某部作品的一些看法,纵观全文,作者的确是一直反驳一些历史
学家对于 Mather 的传记的看法,正确;
E 选项说指出了矛盾的历史报告的微妙不同,显然与主题已无关。
2.答案:E
题型:推理题
解析: 问题问的是支持 Mather 传记的准确性的最有利证据, Yet 转折后作者评价说, modern studies have
profited both from the breadth of information that Mather provides...and from his critical observations of ...
后文是对该评价的展开。可知选 E 最有力,quantity=breath, nature=critical observation。
3.答案:E
题型:推理题
解析:选项 A 中的 annals 在原文中找不到对应,排除;
选项 B 中说对 Governor Winthrop 的描述暴露了其所有缺点,这个在文中没有提到;
选项 C 说 Mather 写传记主要基于自己的亲身经历,原文中找不到对应,且于 Mather amassed all
sorts of published and unpublished documents as sources 相违背;
选项 D 说 biography is an inherently unreliable genre of historical writing 在原文中找不到对应。而且
历史学家只是批评 Mather,并未涉及历史写作类型。
选项 E 可定位至“The charge also obscures Mather’s concern with the settlers material, not just
spiritual, prosperity.”正确。

passage 8
文章结构分析:文章针对 Mediterranean monk seals 的大量死亡提出两个可能原因: 1)是不明病毒,2)
是有毒细菌。然后根据海水样本发现的有毒细菌,认为死亡是由有毒细菌引起的。加强结论的选项就
要支持有毒细菌论,削弱乃至否定病毒论。
答案:D
题型:逻辑单题加强题
解析:题目要求加强文章结论。D 选项支持力度最大,用的是逆否命题的思路:如果是病毒引起的,
应该有大量小海豹死亡。现在只有成年海豹死亡,没有小海豹死亡,所以不是病毒原因。从而大大支
持了有毒细菌论。

passage 9
文章结构分析:文章开始讲主流观点:抵制消费的政治上的利用起源于美国。However 之后,介绍
Mary ODowd 的相反观点:爱尔兰的妇女抵制进口品的历史可以追溯到 17 世纪,它是有自己的历史
的。
1.答案:D
题型:主旨题
解析:这是一道文章主旨题。因此答案选 D。A 选项不准确,作者的立场并不明确。

2.答案:A
题型:句子功能题
解析:高亮的句子为“Breen did acknowledge that a few isolated boycotts may have taken place in other
countries."这一句的上一句是说 Breen 认为抵制消费活动起源于美国,显然高亮句是 B 做的让步。
A 选项最准确(qualify 翻译成限制,限定)。下一句是 Mary 的完全相反的观点:抵制行为起源于爱
尔兰,D 选项不正确,高亮句不是为后句的观点提供合理性。

passage 10
文章结构分析:文章结论是:最近的要求运输小石了的卡车加盖帆布的法律不能减少飞溅石子对后续
车辆造成的伤害,理由是,石子从车厢飞溅出来的可能性远远小于从轮胎缝隙飞溅出来的可能性。
答案:A
题型:加强题
解析:该题要求加强论证。
A 选项加强作用最强。加盖帆布,后续车辆更可能靠近运输石子的卡车,从而提高受伤害的概率(虽
然盖帆布降低了从车厢飞溅石子的机率,但没有降低从轮胎缝隙飞溅的机率)。
B 选项说很多司机已经携带帆布,但携带不等于加盖。
C 选项考虑的是所有伤害的问题,过于宽泛。
D 选项讨论的是空车状态,完全无关。
Passage1 1-Passage20 解析

passage 11
文章结构分析:文章首先说 1970 年代女权主义文学批评出现前对 FF 评价:当作维护传统价值情感主
义代表。女权主义兴起后,强调 FF 非情感因素,尖锐社会批评。新问题出现:如何协调社会批评和
传统价值的矛盾。H 尝试解决:用感情修辞掩饰激进变革目标。T 提出针对感情主义自身的替代解释:
可以通过左右读者情感方式服务激进变革目标。
1.答案:AC
题型:推理题
解析:
该题问这对高亮句中矛盾的正确表述。A 选项可以通过首句得知。C 选项可从第 3 句 Most feminist
scholars find 得知。
2.答案:D
题型:推理题
解析:该题问 T 最会支持的针对文章中评论者的表述。T 的观点立场体现在最后一句:T 提出针对感情
主义自身的替代解释:可以通过左右读者情感方式服务激进变革目标。DE 选项之中,D 选项更好。
之前评论者有 insipid 的明显负评。原文末句 T 的观点,等于承认之前评论者至少认识到感情主义可
以服务于保守主义目标,不能等同于 E 选项,完全意识不到情感主义的作用。

passage 12
文章结构分析:文章理由是:新型植物汽车燃料比传统汽油更高效,环境污染更少,价格也更便宜。
结论是:司机转用新型植物汽油的话,能降低每年燃油花费,还能减少环境破坏。
答案:D
题型:逻辑单题假设题
解析: 题目要求找论证基 J 的假设。注意文章理由到结论的概念跳跃:理由讨论的是单位(例如每升)
问题,结论是消耗/排放总量问题。作者忽略了另外的重要变量:驾驶里程的总量问题。D 选项指出该
被忽略变量不应有不利变化,是文章推理基于的假设。

passage 13
文章结构分析:文章首句介绍老观点:鸟产卵是受制于能量和营养消耗的。第 2 句之后, L 提出不同
观点:产蛋规模受制于父母成功饲养的能力。后续研究发现,自己觅食的种类( P )产蛋规模受制于其
他因素。潜在意思是说,父母饲养的种类( A) , 成功饲养的能力有可能是重要的,但老观点也是有其
道理的。
1.答案:A
题型:推理题
解析:该题问的是首句生物学家的观点,定位可知 A 选项。
2.答案:AC
题型:推理题
解析:该题问 L 关于产卵规模的观点,选项定位可知 A 和 C。
A 选项定位到:He suggested that clutch size had instead evolved in relation to the number of young that
the parents could successfully rear. Subsequently, most studies focused on limitations operating during
chick rearing, particularly among altricial species (species in which the parents feed their young in the nest).
C 选项定位到: Lack, however, suggested that clutch size the number of eggs a bird lays perbreeding cycle
is far below the potential limit of egg production. (文章末句发现并未否定该观点)。

passage 14
文章结构分析:文章首句是作者对 Peterson 的作品价值的总体评论;第二句将其研究与另一位学者
Fosters 相比较。第三句之后主要强调 Peterson 和 Fosters 的区别。
1.答案: AB
题型:细节题
解析:
本题问的是 Peterson 认为女性面临的障碍有哪些,定位到“Peterson sees these same women as having
been fundamentally estranged from the nation by a dominant culture unsympathetic to Black women, and by
a Black intelligentsia whose male view of race concerns left little room for Black female intellect.”这句,可
知选 A 和 B 选项。
2.答案: A
题型:推理题
解析:此题问的是 Peterson 和 Fosters 研究的相似之处,定位到原文的“Its scope resembles that of Frances
Smith Fosters 1993 study, but its approach is quite different." 一句,可知选 A。

passage 15
文章结构分析:文章的结构比较简单明了,先说 Belasco 的现实主义戏剧引起了公众轰动;转折后说
进步戏剧评论家对其负评价,戏剧评论家拥护 New Stagecraft theories,反对 theatrical literalism.
1.答案: D
题型:推理题
解析:该题定位至原文“While Belasco’s meticulously detailed reproduction of an immeldiately recognizable
setting impressed the public, it was derided by progressive theater critics”一句可知答案选 D。
2.答案: B
题型:推理题
解析: 问题中问的是戏剧评论家的看法,定位至原文“The New Stagecraft rejected theatrical literalism; it
drew inspiration from the subjectivity and minimalism of modern painters, advocating simplified sets
designed to express a dramatic texts central ideas. ..... a true artist eliminated the inessential to create more
meaningful, expressive stage images.”可知选 B 选项。

passage 16
文章结构分析:文章理由是:S 的电视新闻工作者是其他行业工人 B 政党支持率的两倍,S 的新闻专
业学生对 B 政党的支持率比 S 电视新闻工作者更高。
文章结论是:假设新闻专业不改变政党支持,随着新闻专业学生参加工作,当前的 S 的电视新闻工作
者是其他行业工人 B 政党支持率之间的显著差异会进步扩大。
答案: D
题型:逻辑单题假设题
解析:
考虑否定结论的可能情况:
1)S 新闻专业学生毕业不在本地工作,
2)S 新闻专业学生大部分不从事电视新闻工作,
3)参加电视新闻工作的 S 新闻专业部分不支持 B 政党,而参加其他行业的 S 新闻专业学生大部分支
持 B 政党。等等。
D 选项正确。
取非 D 选项:希望参加电视新闻工作的 S 新闻专业学生比 S 新闻专业学生作为整体有显著更低的 B
政党支持率。如果这样的话,则破坏结论:差异不会进一步扩大,甚至缩小。

passage 17
文章结构分析:文章首先介绍早期自然主义者的观点:北美有两种 beaver,dam beavers,bank beavers,
bank beavers 不会筑坝。In fact 之后,作者指出,bank beaver 不需要筑坝,它们会筑坝,必要的时候
才筑坝。
1.答案: A
题型:推理题
解析: 该题定位至原文“In fact, dams are primarily a strategy for dealing with annual variations in water
levels. If water levels fall in summer, as they do in most of North America, then beavers lodge entrances may
be exposed.With stabilized water levels, their homes are much safer."可知答案选 A。
2.答案: B
题型:推理题
解析: 从文章结构分析可知,作者否定了用是否会筑坝作为 beaver 分类的依据,两种 beaver 都会筑坝,
可以当作同一个物种。

passage 18
文章结构分析:首段用对比(大小不般配的天体相撞/大小类似的天体相撞)方式,引入大碰撞对月亮起
源的解释。
第二段详细介绍大碰撞说对月亮起源的解释:该理论能解释之前解释的诸多不足。
第三段作者把大碰撞说引申到其他行星,文章末尾表达了作者谨慎乐观的态度。
1.答案: D
题型:细节题
解析:
该题所问高亮句子是“Massive projectiles striking much larger bodies create various kinds of craters,
including multi-ring basins- the largest geologic features observed on planets and moons”,下一句是“In
such collisions, the impactor is completely destroyed and its material is incorporated into the larger body”,
由此可知选 D。
2.答案: E
题型:细节题
解析: 该题问作者对 titannic collision 模型的观点,定位至原文“Such conjectures are tempting, but, since
no early planet was immune to titanic impacts, they could be used indiscriminately to explain away in a
cavalier fashion every unusual planetary characteristic”一句可知选 E。
3.答案: E
题型:推理题
解析: 根据问题中的关键词 compounds and elements 可定位至原文“The difficulty in recondensing this
vapor in Earth's orbit, and its subsequent loss to the vacuum of outer space, might account for the observed
absence in lunar rocks of certain readily vaporized compounds and elements."可知月球上现在已经没有某
些 compounds and elements 了,而这是与地球相比较的结果而言,所以地球上是还有这些物质的,选
E。C 选项不准确,根据原文,这些元素从来没有在月亮存在过:The difficulty in recondensing this vapor
in Earth's orbit, and its subsequent loss to the vacuum of outer space,含有这些元素的气体难于在月亮的
旋转轨道凝固,直接流失到太空了。
4.答案: A
题型:目的题
解析: 问题针对第二段的目的主旨。从文章结构分析可知,第二段是用 Titanic collision impacts 理论解
释月亮起源,该理论能解释之前理论无法解释的一些特殊现象。 所以选 A。

passage 19
文章结构分析:通读全文可知,文章以分类方式介绍探究印刷术产生之前罗马文献流传历史的方法。
首段首句引入主题,指出探究罗马文本流传情况的两种证据:内部证据和外部证据。接着介绍内部
证据。作者对该类证据持偏负面态度,认为其 may paint a distorted picture.
第二段介绍一种外部证据:中世纪的一个文本。作者认为该类证据也不能准确表明当时的真实情况。
第三段介绍另一种外部证据:引用。作者对其也是偏负面态度:引用的未必是一手资料。
第四段总结,警告过度概括的倾向。
1.答案: C
题型:目的主旨题
解析: 该题问文章主旨。
A 选项侧重于 changes,然而文章中并没有相应内容;
D 选项说提倡某种证据,但文中没有体现作者的偏好;
BC 选项中,B 选项说“对比”了研究古罗马文献的证据,文章有分类,但对几种分类证据都是负
面评价,“对比” 不准确;
C 选项说列举了研究古罗马文献流传过程中的困难,正确;所以选 C。
2.答案: D
题型:类比题
解析: 根据关键词 stemma 定位至原文“Internal evidence is used to reconstruct the relationship of the
surviving manuscripts of a Roman text to one another, as represented in a modern stemma codicum: a
diagram depicting the genealogical relationship of surviving manuscripts and those the stemmas editor
believes existed at onetime."可知 stemma 是用来构建“代际关系”的,所以选 D。
3.答案: AB
题型:推理题
解析:主要依靠原文对应,按照文章内容进行选择。定位到原文“It is equally possible that the work
survived far into the Middle Ages in numerous copies in monastic libraries but were unnoticed due to lack of
interest.” 可知选 A、B.定位到“Quotations from a Roman text by a medieval author are another category
of external evidence: but does the appearance of a rare word or grammatical construction- or even a short
passage- really indicate a medieval author's firsthand knowledge of this or that ancient work, or does such
usage instead derive from some intermediate source, such as a grammar book or a popular style manual?”可
知不选 C。
4. 答案:首段末句:“Conversely, the stemma can also bestow a semblance of separation on manuscripts
written within a few months of one another or even in the same room.”
题型:句子功能选择
解析: 由"underestimate the connection between certain manuscripts”,可知要找的句子体现学者低估关
联性:实际关系近的文本,被认为是关系远的。首段末句:manuscripts written within a few months
of one another or even in the same room,文本之间关系紧密。bestow a semblance of separation,
被分开,当作关系远。

passage 20
文章结构分析:文章首段首先介绍一个具体发现:1995 年发现木星大气缺乏之前预测的气态水,进而
指出该发现的意义:复制地球生命的条件可能性很低。
第二段、第三段详细展开地球生命所需的 4 种偶然条件:1)地球公转轨道不大不小。2)木星的
保护作用。3)木星保护的偶尔失灵带来地球生命所需要的重要物质。4)卫星月球的稳定作用。
1. 答案: A
题型:主旨题
解析:通读全文可知本文主要介绍了地球生物圈形成的四个条件,由原文 “Recently planetary
scientists have suggested that the external preconditions for the development of Earth's biosphere
probably included four paramount contingencies.” 可知选 A。
2.答案: E
题型:推理题
解析: 这是道综合理解题,首先是要找准原文,选择有据。
定位到“Recently planetary scientists have suggested that the external preconditions for the development
of Earth's biosphere probably included four paramount contingencies.",概括 4 个关键条件,可知选 E。
3.答案: E
题型:目的题
解析: 定位到原文 “Finally, Earth's unique and massive satellite, the Moon, plays a crucial role in stabilizing
the obliquity of Earth's rotational axis, this obliquity creates the terrestrial seasonality so important to the
evolution and diversity of life. Mars, in contrast, has a wildly oscillating tilt and chaotic seasonality, while
Venus, rotating slowly backward, has virtually no seasonality at all.”通过对比有卫星和没有卫星的不同情
况,可知选 E。
4.答案: B
题型:细节排除题。
解析:文中只有“Thus the evolution of life has depended on a frequency of cometary impacts sufficient to
convey water, as well as carbon and nitrogen”一句,并未提到水、碳、氮的先后顺序。
Passage21-Passage30 解析

passage 21
文章结构分析:文章首段主要介绍女权主义的历史观:建立独立于传统男权主义的女权主义历史。文
章第二段是作者的评论:女权主义历史观有缺陷,主要是会贬低历史上女性作家的贡献。
1.答案: D
题型:目的题
解析:该题根据关键词 Rosalind Miles 和 third millennium B.C 定位到原文“Rosalind Miles's description
restates the popular view: Women's history by contrast has only just begun to invent itself. Males gained
entry to the business of recording, defining and interpreting events in the third millennium B.C.; for women,
this process did not even begin until the nineteenth century." 可知选 D。
2.答案: E
题型:推理题
解析:由关键词 herstory 定位到首次出现位置:“ Feminist accounts of the 1970s and 1980s viewed
historiography /the writing of history) as overwhelmingly his, coining the term herstory /and presenting it
as a compensatory feminist practice"可知迭 E。
3.答案: C
题型:细节题
解析:根据关键词 Mary Astell 定位到原文二段:“ Astell, like those who echoed her sentiments two and a
half centuries later, must be credited for admirable zeal in setting out to right scholarly wrongs, but her
supposition that historians were only male is inaccurate.”可知选 C。
4.答案: A
题型:推理题
解析:根据关键词 Astell’s supposition 定位到原文“Astell’s judgment is at the same time understandable,
given that historical writing by women of the late seventeenth century was not published until the nineteenth
century.”可知选 A。

passage 22
文章结构分析:通读全文可知文章主要介绍了历史学家们对忠诚于联邦的南方人的研究的演变过程。
首段,1930 年之前,很少有人注意到这一研究议题,30 年代开创此研究领域,但有诸多不足;重
复南部政府观点,忽略个人特定身份和经验等。
第二段,介绍 60-70 年代该研究领域进展,主要是从政治、经济角度理解南部反叛者第三段,最近
的研究进展,开始关注地方特点,个人经验。
1. 答案: B
题型:目的主旨题
解析:该题问文章主要目的,从文章结构分析可知答案选 B。
2.答案: C
题型:推理题
解析:根据关键词定位到文章第三段。通过第三段主旨和之前段落的区别可以确定 C 选项,只有该选
项最能体现第三段最新研究的特点:地方性、个人性。
3.答案: C
题型:推理题
解析:锁定关键词 before 1930 可知定位到原文“Until around 1930 few United States Civil War historians
paid much attention to Southerners who opposed the1861-1865 secession from the United States by a
confederacy of Southern states..”选 C。
4.答案: D
题型:句子功能题
解析:高亮句“emphasis has been placed on the ways loyalist Southerners diverged from the political and
economic mainstream of Confederate nationalism.”是第二段最后一句。 它的功能既要考虑和前文,尤
其是第二段的关系,也要考虑和后文第三段的关系。第三段首句“Only recently have some Civil War
historians begun to make Unionists and their experiences, rather than the Confederate state, the center of
inquiry"说明后来的研究者已经将研究的重心转移到了别的方面,因此选 D。

passage 23
文章结构分析:文章是典型新书内容介绍加评论。
第一段先介绍了 Louis Gerteis 新书中的观点,北部改革者从 1803 年代就攻击南部奴隶制,用的是
功利白由主义价值观:个人应该有实现个人最大利益的自由。
第二段把 Louis Gerteis 的观点置入大的理论框架:Growth of a Dissenting Minority interpretation, 然
后讲述了历史学家对该理论柜架的反驳理由。
最后一段则主要是作者的评论,认为是 Gerteis 为 Growth interpretation 赋予新的活力,主要理由是
强调 Gerteis 和之前该观点阵营的区别。
1.答案: C
题型:目的主旨题
解析:通过文章结构分析,可知 C 选项最准确。
2.答案: D
题型:细节题
解析:第二段中历史学家批评 Growth interpretation 会“homogenize the political diversity of Northern
reformers",而第三段中的 Gerteis“does not reduce the Northern reformers to a homogeneous group or try to
argue that there formers shared views undermined their differing party loyalties" 因此 Gerteis formulation
of the Growth interpretation 可能会比传统的解释更为可信。所以选 D。
3.答案: C
题型:推理题
解析:由第二段中的“this interpretive framework, which once dominated antislavery historiography"可知
这一理论原本非常有影响力:第三段中“Gerteis revives the Growth interpretation"可知它被赋予新的解
释,所以选 C。
4.答案: E
题型:排除题
解析:本题是选择最不能支持 Gerteis 理论的情况,重点找和原文陈述矛盾的选项。定位到原文第三
段: Nevertheless, Gerteis argues, these disparate party affiliations did not diminish the actuality of reformer
unity, most prominent in the 1830s. Gerteis differs from traditional adherents of the Growth framework
by...asserting that the basic elements in the antislavery coalition were firmly in place and accepted by all
elements in the Northern reform community as early as the late 1830s. 可知 E 选项和原文矛盾。

passage 24
文章结构分析:文章首段从 E.H Carr 的论文引入历史学界的一个关键问题:历史事件是否存在原因的
主次顺序。马克思主义者认为存在,坚持经济因素是所有事件的首要原因。经典的韦伯社会学派认为
特定事件有特定原因,但仍认为存在主因。作者引用韦伯本人反驳了主因说。
第二段介绍 Runciman 的与作者立场殊途同归的研究思路:很难区分事件主因和背景条件,从而很难
确定事件主因。
1. 答案: D
题型:目的主旨题
解析:该题问文章主要目的,从文章结构分析,可知选项 D 最准确,文章主要是对传统观念提出了挑
战。
2.答案: B
题型:假设题
解析:抓住关键点“most historians share the assumption”,定位到原文开头“Historian E.H Carr's thesis
that all debates concerning the explanation of historical phenomena revolve around the question of the
priority of causes is so familiar to historians as to constitute orthodoxy within their profession...”可知选 B。
3.答案: E
题型:目的题
解析:由关键词"indispensable cause”定位到首段:If any historical event is the product of a number of
factors, then each of these factor is indispensable to the occurrence of the event. But how can one cause be
more indispensable than another? And if it cannot, how can there be a hierarchy of indispensable causes?由
首段结构关系可知 E 选项最准确。
4.答案: A
题型:组织结构题
解析:本问考查文章结构。由文章结构分析可知 A 选项最准确。

passage 25
文章结构分析:本文是典型的人物成就介绍加评论。
首段上来说,现代女权主义给予 MW 她本人值得的评价。尽管一度被人低估,MW 关于性别道德
的立场现在仍有价值。
第二段详细介绍 MW 的性别道德立场:男女应该真正平等,反对当时流行的感伤主义文学的性别
立场。
第三段从一个特定角度展开 MW 的思想:对 Edmund Burke 的评论。作者认为,Edmund Burke 把传统
上对女性的态度应用到贵族,MW 正确地反对了该做法。
1.答案: E
题型:高亮句子作用题
解析:本题问高亮句作用,高亮句子为第三段“A defender of Burke called Wollstonecraft's book an
incoherent mass of treacherous candour, interested generosity, and, if not false, at least unnecessary
accusation." 主要是 Burke 的支持者对 MW 的批评,所以选 E。
2.答案: A
题型:目的题
解析:此问考察了 Burke 描述法国王后的句子的作用,定位到第 3 段上下文“Burke's rhapsody on the
queen of France (glittering like the morning star, full of life, and splendor,and joy) was, for Wollstonecraft,
an example of the argument that beauty and instinct must often prevail over reason, the argument on which
Burke took his stand as a defender of the old order." 结合第 3 段结构,该内容出现的位置是 MW 利
Edmund Burke 的支持者的分歧争论。可知 A 选项最准确,为 MW 争论的一个立场提供例证。
3.答案: B
题型:推理题
解析:问题针对作者对于 Virginia Woolf 对 A Vindication of the Rights of Men and A Vindication of the
Rights of Woman 的评论的看法。对应原文“Virginia Woolf wrote in 1925 of Wollstonecraft’s A
Vindication of the Rights of Men and A Vindication of the Rights of Woman that they felt like books so
true that they seem now to contain nothing new in them, it was more a wishful than an accurate statement of
the case.”可知 Virginia Woolf 非常赞赏这本书的观点,但是作者评论说 it was more a wishful than an
accurate,即作者并不认同 self-evident 的说法,所以选 B。
4.答案: A
题型:推理题
解析:问题考察现代女权主义对于 MW 作品的态度,定位原文首段首句“Modern feminism has brought
the reputation of the English writer Mary Wollstonecraft (1759-1797) to something approaching the luster it
deserves. 首段后文说,.... beginning in the nineteenth century her fame as a writer was hidden by dis
proportionate attention to her unconventional and, at the time, shocking personal life."即现代女权主义者高
度认可 MW, 区分开之前对 MW 私人生活的负面评价。所以 A 最准确。

passage 26
文章结构分析:文章开头介绍传统实践(首句高亮句),需要闭合愈合的伤口通常需要缝针。However
转折后说,降低医疗费用的压力不断增大,Consequently 句是作者的核心结论,也就是针对新实践的
预测:一种帮助伤口愈合的黏合剂会成为新的常规实践。也就是说,虽然这个题划线句子是第一句,
但是 prediction 却是 consequently 后面那句话。 文章后文(包括第 2 句高亮句)是对于核心结论(预测)
的论述:由于黏合剂价格不比缝针贵,也不需要拆线,所以对于粘合剂也可以闭合的伤口,还是使用
黏合剂更合算。
答案: E
题型:高亮句句子功能题
解析:由文章结构思路分析,可知 E 选项最准确。D 选项后半部分错误地判断了文章的主要结论。

passage 27
文章结构分析:首段切入主题:新研究质疑旧观点。旧观点是:冰川间期是一贯地温暖的。
第二段展开新研究情况:GRIP, GISP2, 两个通过研究格陵兰深层冰川了解当时气候的项目。
第三段详细介绍 GRIP 研究的机制。气候温暖,积雪中 0-18 多于 0-16。气候寒冷,风更强,更多
dust and calcium ions.
第四段,尽管两个项目有矛盾之处,GRIP 被认为更可靠。
1. 答案: C
题型:目的主旨题
解析:从文章结构分析可确定选 C。
2.答案: E
题型:类比题
解析:通过第三段对 GRIP 项目的机制描述,可知其主要特征为:通过对现在能发现的古代冰层中的
物质构成分析推知当时的气候状况。E 选项最相似:通过现在能发现的证据推知早期情况,所以选 E。
3.答案: D
题型:细节题
解析:注意关键点“prior to the GRIP findings” 可以定位到原文的第一段“Based one evidence from tree
rings, pollen samples and other records,scientists have for a long time assumed that interglacials- -warm
interludes between ice ages- were as mild and uniform as the Holocene, the present interglacial, has been for
all of its 8,000 to 10,000 years."和第三段“Findings from the upper sections of the cores have confirmed
what scientists already knew climate during the last ice age fluctuated rapidly."从而选 D。
4.答案: C
题型:推理题
解析:根据关键词 oxygen-16 and oxygen-18 定位到原文“When the air was warm, vapor containing the
heavier isotope, oxygen-18,condensed and formed precipitation, in the form of snow, more readily than did
vapor containing oxygen-16. Thus, snow that fell during warmer periods contains proportionally more
oxygen-18 than snow deposited during cold spells.”可知 oxygen-16 andoxygen-18 含量会随冷暖波动,热,
0-18 多,冷,0-16 多。又知:Data from GRIP seem to indicate that the Eemian climate swung at least as
wildly as the climate of ice age periods.气候冷热有波动。选 C。

passage 28
文章结构分析:文章首先指出早期寿险公司面临的问题,之后介绍解决该问题的方法:选取了解当地
情况的德高望重人士担任经纪人。末句总结纠正误解:经纪制度的首要功能不是拉客,而是复刷“熟
人社会心态”。
1.答案: A
题型:目的主旨题
解析:通过全文结构分析可知 A 最准确。
2.答案: D
题型:推理题
解析:根据关键词 city neighborhoods 定位到原文末句... but, rather, to recreate the glass-bowl mentality
associated with small towns or city neighborhoods.可知选 D。

passage 29
文章结构分析:文章先介绍了传统的关于 Younger Dryas 成因的理论解释:the release of water from
glacial Lake Agassiz 导致全球气温变低。However 转折后,作者阐述了新发现对于这一结论的挑战,
1. 答案: D
题型:推理题
解析:该题根据关键词“the release of water from glacial Lake Agassiz"定位到原文“For years, the leading
theory for what caused the Younger Dryas was a release of water from Glacial Lake Agassiz ...However.
evidence has emerged that the Younger Dryas began long before fresh water flooded the North Atlantic."可
知选 D。
2. 答案: A
题型:主旨题
解析:问题考察文章大意,根据文章结构可知应该选 A。

passage 30
文章结构分析:文章首段首先介绍研究者认为火星从来没有湿热期的观点及理由:火星大部分表面是
由未风化材料组成的。黑体句是研究者用来支持自己观点的假设推理。文章第二段是作者的质疑:科
学家看得不仔细,新证据表明火星表面是有风化材料的。
1.答案: B
题型:细节题
解析:该题问科学家们通过什么发现了火星表面的 weathered material。回到原文“New high-resolution
mapping data and close-up surface studies have revealed clays and other hydrated minerals in many regions"
可知选 B。 A 选项不如 B 准确, 文章第二段提到 craters, 但数据研究针对 many regions, 不仅仅 volcanic
craters."。
2.答案:B
题型:推理题
解析:高亮的句子是“If water flowed for an extended period, researchers reasoned, it should have altered
and weathered the volcanic minerals, creating clays or other oxidized, hydrated phases (minerals that
incorporate water molecules in their crystal structure)”科学家推测如果水流经过,会形成泥土或其他水
相关物质。A 说揭示了为什么火星上找水很困难,显然不符合;B 说成功的指出了水流经火星表面的
结果,正确;C 说这是基于水和火山矿物质相互反应的错误的假设,文中没有提到。所以选 B。
Passage31-Passage40 解析
passage 31
文章结构分析:文章是典型驳论结构。首句是科学家共识:人类影响气候变化是最近的工业革命后才
有的。However 转折后,作者论证说:8000 年前人类农业活动就已经开始影响气候。
1.答案: D
题型:高亮句子功能题
解析:本句提问高亮句子是“However, evidence suggests that human agricultural activities may have
had such an effect much earlier” 是相对于前一句“ Scientific consensus is that humans first began to
have a warming effect on Earth's climate within the past century, after coal-burning factories, power plants,
and motor vehicles began releasing carbon dioxide (CO2) and other greenhouse gases in significant
quantities into the air” 而言的。虽然科学共识认为人类气候变暖的影响从上世纪才开始,但实际上证
据表明农业活动在很久以前就有相同的效果。所选 D。
2.答案: E
题型:目的题
解析:定位到原文“concentrations of CO2 started rising about 8000 years ago, even though natural trends
indicate they should have been dropping” 可知虽然根据 natural trends CO2 的浓度应该下降,但实际上
它的浓度在 8000 年前开始上升。这一上升的效果是人类农业活动造成的,所以选 E。

passage 32
文章结构分析:文章理由是,潜水者通过喷射毒素的方式捕捉 princess fish。 潜水者小心控制捕捞数
昂,文章结论是:潜水者的活动对于 princess fish 族群没有危害。
答案: D
题型:逻辑单题削弱题。
解析:D 选项说毒素会危害到 princess fish 赖以生存的珊瑚礁,而珊瑚礁是 princess fish 生存的生态环
境,那么潜水者的捕捞活动对于 princess fish 就有可能产生有害影响,从而破坏结论。

passage 33
文章结构分析:文章的首段通过和现代艺术理论的对比,突出文艺复兴时期建筑特色:艺术风格不拘
泥于建材木身的性质, 并用 Etruscans 和 Florentine Renaissance builders 对 Tuscanstone 的运用做了对比:
用来表达宏大主题的材料,文艺复兴建筑师用来表达精致优雅风格。
第二段,前半段延续首段,通过另一案例,Baroque architects 和 Renaissance architecture 对 travertine
marble 的运用的对比,说明同样材料用于不同风格。后半段介绍不同艺术种类之间风格的一致性,探
究文艺复兴建筑风格的解释。
第三段,介绍 Renaissance architects 的“创造性”不仅局限于材料的使用方面。
1.答案: E
题型:主旨题
解析:该题问文章主旨,E 选项相对最准确本文的主旨是介绍意大利建筑师对于建筑材料的态度,对
比时提到巴洛克建筑。A 选项 quality 概念偏离主题,B 选项 architecture 的概念太大,C 选项 different
cities 偏离主题,D 选项仅仅针对末段内容。
2.答案: D
题型:推理题
解析:根据 painting and architecture of the Baroque era 定位到原文二段末: Similarly, in the next century,
the emphasis on shading and corporeal density in Baroque painting mirrored the use of Roman travertine
marble in Baroque architecture to create broad shadow and powerful masses.该句直接表达的就是绘画和
建筑风格的一致性,通过 similar 结合前句文艺复兴时期绘画与建筑风格的一致性,可以确定 D 选项。
3.答案: C
题型:推理题
解析:通过关键词定位到原文 While Roman travertine marble may have lent itself to fine carving, the
Florentine passion for fine detail is no less marked in Florentine Renaissance painting than in Florentine
Renaissance architecture.可确定 C 选项。B 选项有干扰性,但原文 Italian Renaissance architecture was
shaped less by the "nature”of the materials at hand than by the artistic milieu of Renaissance Italy, which
included painting and sculpture as well as architecture 表达的“被艺术氛围影响比被建筑材料影响更多”
不能理解为“完全不受建筑材料影响”。
4.答案: C
题型:推理题
解析:根据"scholars"定位到原文“Nor were the sepractices confined, as some scholars insist, to the later and
supposedly decadent phases of the art Material, then, was utterly subservient to style”,取非,可得 C 选项。

passage 34
文章结构分析:文章首先提出观察到的现象:C 文化 B 时期的陶瓷制品有烧制后修改的痕迹。C 和 W
两人提出质疑,质疑修改是针对设计错误。理由: 1)很多设计错误未改。2)烧制前改正更方便。
1.答案: C
题型:目的题
解析:
该题问作者提到 C 和 W 两人的目的。C 最准确,one potential explanation 就是潜在的“修改是针对
设计错误”的解释,C 和 W 质疑该解释。D 选项不准确,文章没有提出正面的原因解释。
2.答案: A
题型:细节题
解析:A 选项根据原文: usually, intricate geometric patterns painted in black on white slipped surfaces .....
less often, the original design was simply covered with white slip. 可知,更少设计被白色覆盖。

passage 35
文章结构分析:首段,首先介绍 Architectural morphology 概念,接着引入主题: 建筑形态学应用到新
墨西哥教会建筑,体现本土印第安文化对传统欢洲建筑的影响。
第二段,首先介绍很多研究的重点:强调气候、建筑技术等客观因素。However 转折后,作者强调文
化因素的重要性。
第三段, 用 kiva 的例子,支持第二段转折后自己的观点:文化因素更重要。
第四段,纠正很多历史学家的错误认识,把 kiva 放在教会建筑群中间,不是历史学家理解的
superposition, 而是 conciliatory attitude.
1.答案: A
题型:目的主旨题
解析:问题考察文章大意。体会首段和第四段,可以确定选 A。
2.答案: E
题型:推理题
解析:回到原文最后一段,“Freestanding kivas discovered in the ruins of European-style missionary
communities have been explained by some historians as examples of "superposition”。Under this theory,
Christian domination over indigenous faiths is dramatized by surrounding the kiva with Christian buildings”
可知选 E。
3.答案: B
题型:加强题
解析:问题要求的是能够强化传教士对 kiva 的态度的论述。由文中可知 most Spanish missionaries,
arriving in the Southwest with little or no military support, wisely adopted a somewhat conciliatory attitude
toward the use of the kiva at least initially. This fact, and the careful, solitary placement of the kiva in the
center of the mission- complex courtyards, suggests an intention to highlight the importance of the kiva
rather than to diminish it.让 kiva 位于教会建筑中间,体现传教士采取了和解的态度,主要是因为传教
区没有军事力量保护。求异法对比:有军事保护,没有 kiva 放在教会建筑中间。B 选项体现求异法对
比思路。
4.答案: A
题型:细节题
解析:定位到原文“Certainly, the limitations imposed by manual labor and the locally available materials of
mud-brick and timber necessitated a divergence from the original European church model”说明建筑技术是
导致这些教堂不同于欧式教堂的一个原因。所以选 A。

passage 36
文章结构分析:文章首先引入 hot Jupiters 巨行星的特殊天文现象,标准的基于 solar-system 的理论认
为 hot Jupiters 不可能直接产生于靠近恒星的位置,试图将其解释为:产生于距离恒星距离远的地方,
迁移到靠近恒星的位置。
一种解释为:该巨行星的引力场作用于其出身的前行星盘,前行星盘和该行星的相互引力使其失去
动力,从而旋转向恒星。
另一解释认为:该巨行星的引力场如此强大,以至于该行星在其出身的前行星盘上刻出凹槽 groove,
并把前行星盘分割成内外分区。该巨行星和内外分区的交互作用使得其失去动力,旋转向恒星。
1. 答案: B
题型:目的题
解析:
问题:是何力量阻止该巨行星继续旋转撞向恒星?
该题问细节目的,由关键词 Earth 定位到“They are called hot Jupiters because each is similar in mass to
Jupiter, the largest solar-system planet, but orbits its parent star at a fraction of the distance at which Earth,
let alone Jupiter, orbits the Sun”可知 hot Jupiters 得名于它们和木星的质量相似,同时它们与母恒星的
距离较近。提到地球主要是强调与母星的距离很近,选 B。
2.答案: D
题型:细节题
解析:问题要求选择一项两个解释中只有一种解释讨论过的要素。由两种解释区别可知选 D。C 选项
another hypothesis 提到了:it creates a groove in the disk, 该 disk 重复前文 one hypothesis 中 disk, 也是
由 dust and gas 组成的。
3.答案: E
题型:推理题
解析:根据“attempts” 定位到“In the standard, solar-system-based theory of planetary formation, such as
a massive planet could not form so close to a star. So most attempts to explain a hot Jupiter's existence
envision it forming farther away, then migrating inward”,根据文章结构和后文内容可知之后解释都符
合 standard theory of planetary formation, 所以选 E。

passage 37
文章结构分析:文章是典型的转折结构。第二句 but 转折前,几位二十世纪中期杰出诗人被冠以
confessional 标签。转折后,作者质疑该标签,认为这些诗人是艺术的,不是 confessional 的。
1. 答案: AB
题型:推理题
解析:该题需要选项定位。定位到原文“The motive for confession is penitential or therapeutic...But these
poets always approached their writing as artists, and their motive was aesthetic"可知 A 对; B 说 superficially
appropriate 是对的, 因为作者转折后部分认可转折前称谓: Writing from experiences like madness,
despair, and lust, their aim ...... C 说是由于 absence of any other convenient term 导致的错误,文中没有体
现。
2.答案: C
题型:推理题
解析:定位到原文“But these poets always approached their writing as artists, and their motive was aesthetic.
Writing from experiences like madness, despair, and lust, their aim was to make effective art”可知诗人的动
机对于诗歌的价值是有影响的,A 错; “Writing from experiences like madness, despair, and lust, their aim
was to make effective art, not to cure themselves”,该句是作者认为的诗人的创作意图,不能理解为 B
选项;诗人在实际经验上没有任何完成后感受到的解脱,B 错; “.....to ignore their unanimous disdain
for the idea of confessional poetry” 说明这些诗人并不希望自己的是做被标上 confessional poetry 的标签,
C 对。

passage 38
文章结构分析:文章是典型的多层转折结构。文章开头介绍老观点:彗星曾经被认为是纯粹的形成外
部行星的残留物。 然后说 1970 年开始的新认识:紫外线放射足以使其表面不纯。Nevertheless 转折
后,说,太阳的热度足以剥除彗星表面的不纯物质,意思是彗星仍然可以是纯粹的。文章最后说,彗
星本身的放射性同位素足以使其内部演化,意思是彗星更可能是不纯的。
1.答案: ABC
题型:细节题
解析:该题需要选项定位。定位到原文“laboratory simulations revealed there was sufficient ultraviolet
radiation reaching comets to darken their surfaces and there were Sufficient cosmic rays to alter chemical
bonds or even molecular structure near the surface”可知选 AC 选项,根据原文 comets might contain
decaying radioactive isotopes that could have warmed cometary interiors to temperatures that caused the
interiors to evolve 可知 B 选项。
2.答案: C
题型:细节题
解析:该题需要结合题干选项定位。A 选项说,overturned by analysis of what happens when....要求原文
说的是实地研究证据,原文是 laboratory simulation 实验室模拟,A 错。B 选项,原文..... must be pristine
relics of the material that formed the outer planets 中,关于外部行星的物质组成,并不明确是否是观察揭
示的,B 错。C 选项,原文“Astronomers
Who study planet formation once believed that comets-because they remain mostly in the distant Oort cloud,
where temperatures are close to absolute zero -must be pristine relics of the material that formed the outer
planets” 是对星系外围物质的主观表述,是一种 consideration,可知 C 选项正确。

passage 39
文章结构分析:传统方法需要麻醉猴子采集血样判定 DNA, 确定个体猴子亲属关系,新方法可以通过
毛发判定 DNA,文章结论是,就能确定个体猴子亲属关系了(新方法好使)。
l.答案: C
题型:逻辑单题假设题
解析:C 选项取非,不能确定毛发的个体猴子归属,破坏文章结论。所以选 C。

passage 40
文章结构分析:文章是典型的研究评论:作者认为 B 的 village sketches 把社区描画成单质固定的,认
为 B 忽略了女性作家的 Village sketches.女性作家把社区描画成动态活跃的。
1.答案: BC
题型:细节题
解析:根据关键词 village sketches written by women 定位到原文“These women's geographical mobility
was restricted (although women writers of the time were not uniformly circumscribed in this way), and their
subject matter reflected this fact” 和“These writers represent community as dynamic, as something that
must be negotiated and renegotiated because of its members' divergent histories, positions, expectations, and
beliefs” 可知选 B、C 选项。
2.答案: 文章第一句
题型:句子功能选择题
解析:问题要求选出男性作家和女性作家描述乡村社区不同之处的句子,对应到原文首句:“ Buell'
study of village sketches(a type of fiction popular in the United States in the1830s and 1840s)provides a
valuable summary of sketches that portray the community as homogeneous and fixed, but it ignores those by
women writers, which typically depicted the diversity that increasingly characterized actual village
communities at that time."
3.答案: E
题型:细节题
解析: 根据人名定位到到原文“Yet their texts were enriched by what Gilligan, writing in a different context,
has called the ability to attend to voices other than one's own” ,根据“writing in a different context”,
可知选 E。
Passage41-Passage50 解析
passage 41
文章结构分析:文章首段首先引入主题:1820-1870 的女性小说。作者借 Nina Baym 之口,给予正评
价,指出其流行性。但与社会大众相反,评论界对其是 mixed 评价。Theoretical opposition 认为其腐化
堕落。Clerical opponents 认为其是道德说教上的竞争者,攻击更加猛烈。男性作家同样也贬低女性作
家。
第二段首先转折段。出版商对该时期女性小说热情欢迎。少数评论者对其重视,关注到个人作家独特
价值。但 On the whole, however 之后,作者认为,评论界总体上对该时期女性小说还是偏负评价。Baym
承认女性小说主题单一,但为其辩护:认为女性作家其实并不把自己当作艺术家,而是职业写手。对
女性作家而言,娱乐性和教育性并不矛盾。
1.答案: B
题型:目的主旨题
解析:从文章结构分析可知选 B。
2.答案: D
题型:细节题
解析:回到原文“According to Baym, the genre began with Catharine Sedgwick's New-England Tale (1822),
manifested itself as the best-selling reading matter of the American public in the unprecedented sales of
Susan Warner's Wide, Wide World(1850), and remained a dominant fictional type until after 1870”作者
提到 Wide, Wide World 主要想以此为例证明女性小说非常受欢迎,所以选 D。
3.答案: E
题型:推理题
解析:拿 clergy 定位到第一段的倒数第二句话。并且和最后一句话的开头 similarly 形成类比关系。男
性小说家和 clergy 反对女性作者的理由都是一致的,都认为自己的功能受到女性取代的威胁,由此可
知答案。
4.答案: B
题型:推理题
解析:回到原文“Some few reviewers wrote about these works with attention and respect, distinguishing
between the works of the different authors and identifying individual strengths and weaknesses"说明只有很
少数的 reviewers 才能区分不同的作家,大名数 reviewers 是不能很好地区分这些女作家的,选 B。

passage 42
文章结构分析:文章关于对 Cuban-American 作家的评论。Carolina Hospital 认为用英语写作的古巴裔
作家不应该被忽略。另一评论者注意到:英语载体和西班牙语载体同样重要,但强调审点不同。
1.答案: AC
题型:细节题
解析:该题需要选项定位。回到原文“they represent a new phase of Cubanness, one that is expressed in the
language comfortable to its writers---English” 可知 A 正确; “Cuban-American writers who write primarily
in English are dismissed from anthologies and analyses of Cuban exile literature on that account”可知 B 错
误;“She argues that since these writers were born in Cuba and consider themselves Cuban writers, they
should not be ignored, and maintains that they represent a new phase of Cubanness, one that is expressed in
the language comfortable to its writers---English”可知 C 正确。
2.答案: C
题型:词汇题
解析:词汇题。回到原文“Cuban-American writers who write primarily in English...they represent a new
phase of Cubanness, one that is expressed in the language comfortable to its writers--- English"可知上下文
要求词义为:表达流畅的,运用熟练的,选 C。

passage 43
文章结构分析:文章首先介绍地球任何地点阳光照射的变化性导致热能储存的重要性。然后,文章把
热能储存分类为两种: latent heat, sensible heat.后文给予详细介绍(主要针对 latent heat 缺点)。
1.答案: B
题型:内容主旨题
解析:根据全文结构分析可知选 B。
2.答案: A
题型:细节题
解析:选项定位。原文首句:“Constant variations in the amount of sunlight available on Earth at any given
location make energy storage a necessary design feature of terrestrial solar- energy systems”在地球上的任
意一个地方,光照量总在变化,使得储存能量成为 solar-energy systems 的一个特征。可知选 A。
3.答案: AC
题型:推理题
解析:A 定位原文“but in any substance this storage is available only at the unique fixed temperature at
which the particular phase transition occurs in that substance.”C 定位原文“Moreover, materials that have
transitions at the temperatures that terrestrial solar-energy systems are likely to encounter are usually
destructively corrosive at those temperatures.”

passage 44
文章结构分析:章针对 exotic insect pests 对森林生态系统的影响:短线影响和长线影响。之后,文章
解释为何外来物种造成长线影响的原因。
1.答案: ABC
题型:细节题
解析:文选项定位。问到原文“Short-term effects include the disturbances directly associated with the action
of the pest, which may cause the defoliation, loss of vigor, or death of trees. Long-term effects are primarily
mediated by changes in tree species composition and the consequent alterations of forest structure,
productivity, and nutrient uptake” 可知选 A、B、C。
2.答案: C
题型:目的题
解析:定位到原文“Shifts in forest species composition ramify through the ecosystem in many ways because
tree species have different, often unique properties”可知选 C。

passage 45
文章结构分析:本文解释 teak 代替 oak 成为造船木料的原因:唯一膨胀率相同的木料,抵制腐蚀和虫
害,适合大规模运输。
1.答案: AB
题型:推理题
解析:题目问的是 teak 相比 mahogany 的优点,回到原义“Only teak matches oak's expansion coefficient
and stays watertight. It is unmatched in its resistance to rot and pests.....可知选 A、B。
2.答案: B
题型:词汇题
解析:回到原文“as British oak supplies grew scarce, shipbuilders there turned to teak and found in it an
ideal substitute"可知上下文要求的义项是:求助于,寻求。选 B。

passage 46
文章结构分析:原文结论“it cannot be concluded that virtually the only biologists interested in research that
such experiments could address are those biologists already employed by the space program”是双重否定
句,正面理解是:政府太空实验项目已经雇用的生物学家之外应该还有其他生物学家对太空实验项目
感兴趣。
答案: B
题型:逻辑单题填空题
解析:空格要求支持该结论。根据文章理由,区分理解三个概念范畴:1)生物学家,2)大学生物学
家,3)政府太空实验项目已经雇用的生物学家。画维恩图可知,选项 B 最支持结论。

passage 47
文章结构分析:文章首先介绍传统历史学家对特定对象(日本 1880 一 1930 的纺织工业)的研究方法:
分开研究。However 转折后,Hunter 倡导新的研究方法:关注共性,当作整体研究。
1.答案: E
题型:细节题
解析:该题定位题干中的关键词 historians' traditional analysis,回到原文“Because the branches were
highly diverse—in scale, skill requirements, and technology—historians traditionally have analyzed them
separately”可知选 E。
2.答案: A
题型:细节题
解析:定位到题干中的关键词 historians' traditional analysis, However 之后, 是 Hunter 对其反驳:the
workforces of all three were drawn primarily from the same population: young, mostly rural women aged 10
to 25. Noting this commonality, Hunter argues that a consideration of the three branches of production
together is long overdue: examining elements common to the different branches of textile production may,
she asserts, permit the identification of gender-based factors that may have influenced the operation of the
Japanese female labor market as a whole" 可知选 A。
B 选项是 historians' traditional analysis 分开研究的原因,但不是 Hunter 的反驳点。

passage 48
文章结构分析:文章是典型的现象解释结构。
首先引入特殊现象:生物学家发现居住在海底、不依赖太阳光源的虾的繁殖有季节性规律。可能性解
释:和它们的后代繁殖有关,因为后代迁移的时候需要食物,而作为食物的海藻的供给量是有季节性
的。
1.答案: E
题型:高亮句子功能题
解析:本题问高亮句作用。从文章结构分析可知,高亮句提供研究证据:研究者发现为后代提供食物
的物种并没有季节性繁殖的现象,并且居住在有非季节性开花的海藻的区域的物种也没有季节性繁殖
的特征。也就是说如果海藻有季节性,繁殖也有季节性,海藻没季节性,繁殖也没季节性,所以很显
然,作者研究者用的是求异法思路,从反面论证了前文结论的正确性。选 E。

passage 49
文章结构分析:首段指出事实趋势:因为气候变化,泥石流的频率、强度发生变化。作者认为这很正
常:降水和地震是泥石流的两个外部因素。过去证据表明泥石流的周期变化与气候波动有关系。
二段指出当前泥石流变化的特点:人为因素影响。森林砍伐和土地使用变化因为对植被有直接影响,
所以会影响泥石流的频率、强度。作者最后指出,山地生态系统对于这些因素变化对反应程度尚是未
知因素,处理该问题是山地地貌保持对关键因素。
1.答案: A
题型:目的题
解析:定位句在文章第一段第二句。提到两个外部机制,插入语部分的为第二个。只有一个有第三句
论据支持解释。由此得到 A。
2.答案: C
题型:推理题
解析:C 选项能定位到原文“Evidence from the past clearly indicates that cycles of elevated landslide
activity have been followed by cycles of low activity, and that these are correlated with climate fluctuations
over a variety of timescales",其中,low activity = decrease。
3.答案: AC
题型:推理题
解析:A 选项定位到原文“What sets current changes in landslide activity apart is the likely influence of
anthropogenic [i.e.. human-caused] factors, either acting alone or in concert with climate, which can further
modify the process of landsliding and the nature of ecosystem responses。C 选项定位到原文二段首句:
What sets current changes in landslide activity apart is the likely influence of anthropogenic[i.e.,
human-caused] factors.

passage 50
文章结构分析:文章开头是一个对比转折:extinctions of indigenous species caused by the introduction of
non-indigenous predators and pathogens, 由非本地捕食者或病原体引起的本地动物灭绝很多,但是
surprisingly few extinctions of indigenous species can be attributed to competition from introduced species,
即物种竞争引起的灭绝其实是很少的。作者然后举出北美的证据。但是,作者认为,从该证据可能得
出两种替代性解释。
1.答案: A
题型:目的主旨题
解析:从文章结构分析可知 A 选项最准确。
B 选项,conflicting data 没有提到。
C 选项,resolving a debate 原文没有。
D 选项,comparing two theories 原文没有。
E 选项,refuting a proposed explanation 不准确。
2.答案: E
题型:目的题
解析:问题考察作者举北美非本地植物例子的作用。根据文章结构,该例子为上位句服务: However,
surprisingly few extinctions of indigenous species can be attributed to competition from introduced species.
可知选 E。
Passage51-Passage60 解析
passage 51
文章结构分析:文章作者围绕对于 the literary personality 的观点,介绍不同学派的学者对其看法,并
表明自己的态度。多种当代批评流派重视。解构主义( Deconstructionists )轻视,认为 intertextuality,
subtext, metatexts 更重要。新历史主义者(New historicists )也轻视,认为作家生活年代的思想潮流更重
要。However 转折后是作者立场,作者认为,literary personality 有其关键价值。
1.答案: A
题型:目的主旨题
解析: 由文章结构分析可知 A 最合适。
2.答案: B
题型:推理题
解析:问题要求选择 new historicists 和 deconstructionists 观点的共同之处,回到原文 Deconstructionists
view the literary personality, like the writer's biographical personality,as irrelevant.....New historicists also
devalue the literary personality 可知二者都否认 literary personality 的价值,所以选 B。
A 选项是新历史主义观点,解构主义没有提到。
C 选项是解构主义观点,新历史主义没有提到。
3.答案: B
题型:词汇题
解析:根据上下文:New historicists also devalue the literary personality, since, in their emphasis on a work's
historical contexts, they credit a writer with only those insights and ideas that were generally available
when the writer lived. 知其对应 devalue, 是 devalue 反义词,选项 B 最准确。

passage 52
文章结构分析:文章通过一个实际问题的解决介绍 WS 的研究。WS 被邀请解决一个大学讲堂的回声
问题。WS 发现了问声问题的原因,成功解决了问题,并且形成数学公式。
1.答案: C
题型:
解析:细节题
根据关键词定位到原文“a university lecture hall in which the echo of a speaker's words rendered them
unintelligible" 和“By hanging panels of sound-absorbing felt on the walls, Sabine reduced the echo enough
to make the hall usable”可知 C 选项最准确。
2.答案: D
题型:推理题
解析:该题目需要概括 WS 的工作,结合文章结构分析可知 D 最准确。

passage 53
文章结构分析:首段介绍一些现代学者的观点:作者应该是 Lucius Annaeus Seneca the Younger。从年
代相关的证据上,Lucius Annaeus Seneca the Younger 符合要求。
第二段针对首段作者确认的疑问:Lucius Annaeus Seneca the Younger 本人为何没有提及这些作品?作
者介绍了两个可能解释之一:Lucius Annaeus Seneca the Younger 的谨慎态度。But 转折后,作者提出
自己的质疑:Lucius Annaeus Seneca the Younger 作为皇帝的辅导师,不应该谨慎呀!Lucius Annaeus
Seneca the Younger 不应该是这些悲剧的作者。
1.答案: E
题型:目的题
解析:文章针对 8 个现存悲剧的作者确认问题。
根据关键词定位到原文“... and the plays must have been written by 96C. C., when Quintilian quotes
Medea, one of the tragedies.”,Medea 被别人引用,自然应该现有 Medea.可知 E 选项,该时间是悲剧
被创作的最晚可能时间。
2. 答案: E
题型:细节题
解析:该题需要选项定位。E 选项可定位到原文第二段首句“it is remarkably, however, that Seneca
himself never mentions the plays, since there are certainly passages in them that could be used to illustrate
points of his philosophy”。
3. 答案: B
题型:推理题
解析:根据文章结构定位到原文第二段末:“but if any one could safely attach his name to dramas, surely
it would be Seneca, the emperor’s tutor. And although Herrmann offers Seneca’s modesty as an explanation,
Seneca is not averse referring to his other writings. The evidence for equating Seneca with the author of the
tragedies seems circumstantial”, 由作者因为 Seneca 没有其他作品中提到这些 plays 而对 Seneca 是不是
真正的作者感到怀疑,可知 B 选项,作者认为,如果 Seneca 真是作者,他应该提到这些悲剧作品。
4. 答案: C
题型:推理题
解析: 题目问作者就 Seneca 作为皇帝导师的身份暗示了什么。 定位到第二段第二句: In the early Roman
Empire, playwrights were sometimes exiled or executed for line constructed as directed against the emperor;
thus, Seneca's silence maybe simple prudence. But if anyone could safely attach his name to dramas, surely it
would be Seneca, the emperor's tutor.前句说当时剧作家有时因剧本内容受到迫害,以之解释 Seneca 的
不署名。但后句说:如果有什么人是安全的,当然首先是 Seneca, 皇帝的导师。作者否定用“担心受
到迫害”来解释 Seneca 的不署名,暗示说,因为皇帝导师的什么,Seneca 是安全的。
5.答案: D
题型:排除题
解析: 以下断言都是与文章中 Seneca 作为戏剧作者的身份一致的,除了:
D 选项原文没有依据。原文第二段首句:there are certain passages in them that could be used to illustrate
points of his philosophy。能够被用来阐明作者的哲学观点,不同于 D 选项:Seneca 戏剧中确定有提到
作者哲学作品的内容。
其他选项,A 选项,原文首段第二句: Since nobody of that name is known。
B 选项, 原文第二段第三句:In the early Roman Empire, playwrights were sometimes exiled or executed
for line constructed as directed against the emperor。
C 选项,原文首段:internal references to earlier poets, most notably Ovid。
E 选项, 原文首段: the plays must have been written by 96 C.E., when Quintilian quotes Medea, one of the
tragedies。

passage 54
文章结构分析:文章围绕地球深处新发现(独立于太阳光,只依赖液态水的生命)展开,说明对于太空
生命探索的意义。作者用 Europa 举例,说其地下可能存在液态水,从而有可能存在独立于阳光的生
命。
1.答案: BC
题型:细节题
解析:题目针对 Europa 存在生命的条件,定位原文“As Europa orbit sits planet, however, it flexes due to
the gravitational tug-of-war between it, its sister moons, and Jupiter. Through friction, this flexing produces
heat in the moon's interior capable of melting ice”可知选项 B、C。
2.答案: B
题型:高亮句子功能题
解析:由文章结构分析可知 B 选项最准确。Europa 例子主要用来推测。A 选项不准确,后文没有说对
Europa 的猜测能够被核实。

passage 55
文章结构分析:文章首先介绍一些历史学家对于以 McCormick 为主倡导的“party period paradigm”的
质疑。Voss-Hubbard 主要引用第三党的存在作为证据。Formisano 主要强调该时期 nonpartisan and
anti-party governance 的证据。Yet 转折后,作者为 McCormick 进行了辩护:作者认为“party period
paradigm”仍然是合理的观点。
1.答案: D
题型:目的主旨题
解析: 根据文章结构分析,可知 D 选项最准确。
A 选项“common”概念不准确。
C 选项,challenge the validity of evidence 不准确。
2.答案:Yet McCormick would deny that the existence of anti-party sentiment during the period undermined
the paradigm, since he has always acknowledged the residual strength of such sentiment during the
nineteenth century.
题型:句子功能选择题
解析: 问题要求选择体现一位历史学家对手质疑的反应,由文章结构分析,可知原文该句最体现此功
能。
3.答案: A
题型:词汇题
解析: 定位原文“that political parties—especially the two major parties—in the United States between the
years 1835 and 1 900 evoked extraordinary loyalty from voter sand dominated political life.”,根据上下文句
义要求,可知 A 最准确:“引起, 诱发”某种情感态度。

passage 56
文章结构分析:文章首先介绍地面植物对于土壤结构(soil composition) 的重要影响,以其作为理由说
明外来植物物种的入侵对于土壤结构的影响。
1.答案: BC
题型:细节题
解析: 题目问植物为土壤提供资源的方式,根据关键词定位到原文第二句:“Plants provide organic
matter for soil communities through the decomposition of leaf litter, by oozing nutrients from roots, or
through other methods of deposition of organic compounds into the soil environments”。可知选择 BC 选项。
A 选项原文没有提及。
2.答案: C
题型:推理题
解析: 该题需要选项定位。答案 AB 在原文中找不到依据。根据原文"As a result of these diverse
methods by which plants supply resources, unique soil communities form under different plant species and
under plant communities that differ in composition "可知 C 正确。

passage 57
文章结构分析:文章针对新研究成果之前观点的关系展开。开头介绍的是被新研究成果核实的老观点:
1)物种丰富性和海拔负相关。
2)本地物种和地理因素(海岛分隔)相关。
3)基因区分演化(genetic differentiation)受物种所在生态环境影响。
However 转折后是被新证据推翻的老观点,新证据表明:
1)一些本地物种能象外来物种一样适应利用不规则生态栖息地(disturbed habitat)
2)所有物种(本地/外地)基因变化程度都高
3)(本地/外地)都能在小区域延续独立种系(independent lineages),本地物种分化对物种丰富
性贡献大。
1. 答案: D
题型:目的主旨题
解析: 从全文整体结构分析可知选 D 选项最合适。
A 选项,a new theory 文章无。
B 选项,文章没有 debates。
C 选项,文章没有讨论证据是否具有相关性( relevance )。
E 选项,文章没有说菲律宾果蝇 atypical。
2. 答案: D
题型:高亮句子功能题
解析: 根据文章结构,注意到高亮句 genetic differentiation has been influenced by the ecology of the
species and the current and past geographic and geological conditions 前有一个重要的表示并列的逻辑信
号词“and”, 可知高亮句和前半句一起服务于首句提出的观点。所以,该题答案选 D。
3. 答案: A
题型:推理题
解析: 该题需要根据文章结构比较选项。
B 答案错误,原文中并没有出现 atypical 的描述;
C 不准确,原文中并没有提到研究者之间的争论;
D 错误,因为 far more previous hypotheses have been overturned than supported;
E 错误, vulnerable 概念原文没有。
A 选项最准确,因为文章重点在于转折后;far more previous hypotheses have been overturned than
supported。
passage 58
文章结构分析:文章是对最近研究趋势(关于选举政治中的性别差异)的解释:因为几个研究的发现
表明选举行为的性别差异不再显著。However 转折后说,F & S 认为,南方的性别差异仍然显著。
1. 答案: 第二句
题型:句子功能选择题
解析: 该题要求找针对特定趋势的原因,根据文章结构可以定位到第二句:This shift in focus from
gender inequality in political participation may be due to the finding in several studies of voting behavior in
the United States that since 1980...... “due to”部分就是原因解释。
2. 答案: E
题型:细节题
解析: 该题需要选项定位。E 选项可以定位到原文 Recent studies of the gender gap in the history of
United States policies tend to focus on candidate choice rather than on registration and turnout. 其中,the
frequency with which they vote 对应原文:registration and turnout。此题容易错选 D 选项,注意 voter
preference 指的其实就是 candidate choice,这个题没有说不同地区的 candidate choice 的差异,而是说
的男女投票人的报名和出席的差异。

passage 59
文章结构分析:文章围绕最近的关于古玛雅水管理的研究发现展开。水利用设施的发现被认为有利于
中央集权的管理制度。文章末句,作者指出,该观点不适合于水资源丰富的地区(尽管对于缺乏水资
源的地区有道理)
1.答案: D
题型:推理题
解析: 该题需要理解选项与文章结构的匹配程度。
A 选项,a widely scattered population 文章没有。
B 选项,原文没有强调前提必要性:pretext。
C 选项,原文没有提到 challenges。
E 选项,原文没有提到 continued maintenance。
D 选项,最合适地概括了文章结构。
2.答案: ABC
题型:细节题
解析: 定位到原文 In the central and southern Maya Lowlands, this kind of water control was necessary to
support large populations throughout the year due to the scarcity of perennial surface water and the seasonal
availability of rainfall 可知 AC 正确。pattern of rainfall 对应于 seasonal availability of rainfall,insufficient
resources such as ponds, rivers and lakes 对应于 scarcity of perennial surface water。
B 选项对应首句:Recent studies of ancient Maya water management have found that the urban
architecture of some cities was used to divert rainfall runoff into gravity-fed systems of interconnected
reservoirs.

passage 60
文章结构分析:文章围绕 Ralph Ellison 对于视觉艺术的态度。Yet 转折前介绍他的认可态度,转折后
重点强调他的负面态度:勉强接受两个对于 Invisible Man 插图版的出版,持续拒绝电影版的改编,坚
持认为只有语言才能表达美国身份的复杂性,认为该复杂性来自美国建国文件和当代经验的冲突。
1.答案: E
题型:目的题
解析: 该题针对文章细节, 根据关键词 Elision's apprenticeship 定位到原文 Ralph Ellison was passionately
interested in visual ...... even apprenticing with sculptor Richmond Barthe for a time. Yet he was wary of
projects aiming to provide a visual rendering of his novel Invisible Man。结合文章结构,可知举这个例子
是为后文他拒绝视觉艺术作出让步,他对视觉艺术不是完全负面态度。
2.答案: C
题型:推理题
解析: 根据关键词 tension 定位到原文 This complexity consisted of the tension arising from the collision of
the United States' written ideals, as outlined in the founding documents, and the historical and contemporary
experience molding the national consciousness,结合前句:Ellison insisted that only language could capture
the complexity of American identity ,可知 Ellison 相信文字能比 visual art 更好的描述该矛盾,所以选 C。
Passage61-Passage70 解析
passage 61
文章结构分析:文章对比了陆地农业和海洋渔业两种食物获得方式,并重点强调了海洋渔业的问题:
与陆地农业不同,海洋渔业难于同时实现两个目的:增加食物产量和保护生态系统丰富性。
1.答案: B
题型:目的主旨题
解析: 该题针对文章主旨,通过结构分析可知 B 选项最合适。
2.答案: E
题型:细节题
解析: 定位到原文 produce greater yields from less land,thereby freeing land for conservation purposes,
结合和海洋渔业的对比,可知 E 选项。

passage 62
文章结构分析:通读全文可知文章新发现否定老观点结构。
老观点:increasing-disadvantage model/ceiling metaphor for women.
相关新证据:
1)私人领域,流动机会随职位升高而增加
2)公共领域,无性别差异或低级别更多不利
3)国际研究中,多数相反证据。
1.答案: B
题型:目的主旨题
解析: 该题针对文章主旨,根据文章结构分析,可知 B 最合适。
2.答案: A
题型:细节题
解析: limited evidence 的原句是"women's probability of being located in a higher versus a lower
hierarchical level declined relative to men's a higher levels." ,这个说法就是女性在高地位的概率降低,这
也就是 increasing-disadvantage model 的同义改写,所以 A 选项符合。注意这个题不是在问 limited
evidence 这句话的功能,而是在问 limited evidence 本身的内容。
3.答案: E
题型:推理题
解析: 题目问文章作者与老观点的共同之处。老观点( increasing-disadvantage model )认为女性随职位
升高面对更多因素。作者引用新研究反对老观点,在文章最后又清楚表明自己立场:Women's scarcity
in top organizational ranks may simply represent the cumulative effect of a constant-or even
decreasing-disadvantage at successive hierarchical levels。可见,虽然作者和老观点就不利因素是否随职
位提高而增加有分歧,但都同意:妇女面对不利歧视因素。所以选 E。

passage 63
文章结构分析:文章是研究介绍加评论结构。George Orwell 研究商业文本,认为其抑制普通民众的
本能社会主义倾向,公众的激进本能(社会主义倾向)会被商业文本中 convert patrician conservatism 所
蒙蔽。作者 George Orwell 的这些研究高评价, 但认为有早于 George Orwell 的对流行商业文化的严肃
研究。
答案: B
题型:推理题
解析:该题需要选项定位。 B 选项可以定位到原文 George Orwell examined commercial texts......The public,
he concluded, was often being duped by a convert patrician conservatism, conveyed through commercial
culture。
A 选项无依据。在文章中,ordinary people 是商业文本的作用对象。
C 选项与原文相反。
D 选项与作者评论相反。
E 选项原文无依据。

passage 64
文章结构分析:文章是关于 Charlotte Bronte 的几个传记评论的介绍及评论,主要按照时间顺序介绍评
论。文章作者认为:Elizabeth Gaskell 的评论强调 CB 作品受工业化时代环境影响小,体现乡村文化,
强调 CB 思想观点的正确性。 Matthew Arnold 关注 CB 作品中隐藏的颠覆性、 Winifred Gerin
破坏性思想。
还是受到 Elizabeth Gaskell 的影响。女权主义者引入对 CB 的新解释,但只有 Juliet Barker 考虑到周围
变化的世界对 CB 的影响。
1.答案: C
题型:目的主旨题
解析: 该题考察文章主旨,根据文章时间线索结构可知选 C。
2.答案: B
题型:细节题
解析: 根据人名定位到原文:this was at variance with the subversive spirit Matthew Arnold accurately
discerned, albeit with distance, deep within Bronte's fiction 可知 Matthew Arnold 认为 Bronte 的小说包含
颠覆性的精神,是危险的,所以选 B。
3.答案: E
题型:细节题
解析:根据人名定位原文:Elizabeth Gaskell promoted the long-persisting romantic view of Bronte as
having no connection with the rest of English society at a time when industrialization was causing much
turbulence, but as having sprung naturally...out of the English countryside 表明 Gaskell 认为 Charlotte
Bronte 深受乡村自然环境影响,所以选 E。

passage 65
文章结构分析:文章第一段说 Seligia 石棉在建筑业有着广泛的应用。但由于这种材料会导致某种癌
症,石棉将会被禁止。第一个高亮句是作者预测,也是作者核心结论:the ban will result in reducing the
incidence of that type of cancer in Seligia by as much as 50 percent。
第二个黑体句(第二段开头)是作者让步承认的与核心结论矛盾的信息:less than 30 percent of the death
certificates of Seligians who have died of that cancer have "construction" listed as the deceased's
occupation。作者之后补充辩论说,因为从事建筑业的人老了后会改行,而通常死亡证明上又只会记
录最后的一个职业。
答案: A
题型:高亮句子功能题
解析:根据文章结构可知,第二个高亮句需要后文补充,如果没有补充信息,就会质疑核心结论。A
选项是对两句黑体句在全文中功能的最准确概括。

passage 66
文章结构分析:文章首先指出矛盾现象:学者关注到 19 世纪早期的作家群体,但对该时期的文学评
论研究很不充分。之后是作者对该矛盾(文学评论研究不充分)的解释:行业壁垒,缺乏批评理论训练,
研究既不够集中,又不够具有大局观。
1. 答案: B
题型:细节题
解析: 该题根据关键词 literary scholars, historians 定位到原文 historians study people across the nation,
but much literary scholarship called"American" actually examines works produced in northeastern states ,可
知 B 选项正确。
2. 答案: D
题型:细节题
解析: 定位到原文对矛盾的解释:Few historians have extensive training in critical theory and its
specialized languages, .... And historians usually study the operations of capitalism in its details, while
literary critics produce a generalized picture of literary commodification. 可知文学批评的独特的性质是阻
碍历史学家的研究的原因,所以选 D 选项。

passage 67
文章结构分析:文章针对一个文学现象( Samuel Johnson 拒绝赞助)理解的评论。However 转折后,作
者反驳旧理解观点( a key moment in the history of publishing ) ,认为该事件不重要,仅仅是一个情感
表达。
1.答案: E
题型:目的题
解析:该题要求根据文章上下文理解题干关键词的作用。Johnson's 1762 pension award 其实支持了作者
的观点:该拒绝事件没有那么重要, 他之后仍然接受了类似的赞助。E 选项最准确。
2.答案: C
题型:高亮句子功能题
解析:该题考察高亮句作用。根据文章结构,可知该句用来支持作者核心观点,否定老观点解释。C
选项最准确表述了高亮句和老观点的关系。

passage 68
文章结构分析:文章首先描述矛盾现象:更多来自欧亚大陆的适应寒冷动物迁徙到非洲,更少来自非
洲的适应炎热气候动物迁徙到欧亚大陆。作者从大陆桥和大陆桥两边的适宜栖息地角度解释。气候寒
冷期,大陆桥存在时间更长,适合适应寒冷气候动物迁徙到非洲。相反,气候温暖期,只有短暂的海
水上升滞后期大陆桥才存在,适应炎热气候动物有更少机会迁徙到欧亚大陆。
1.答案: A
题型:细节题
解析:
该题根据关键词 ice age 定位到原文 During periods of climatic cooling, such as the various ice ages, the
land bridge is open for a long time (because sea level remains low) and is usable by cold-adapt species
because cool habitats then extend across it,结合前文可知选 A。
2.答案: D
题型:细节题
解析:根据关键词 short lag 定位原文 intercontinental migrations require both a land bridge connecting the
two continents and the suitable habitat both on and across that land bridge ,可知选 D。
3.答案: E
题型:细节题
解析:题干所指 the apparent bias in antelope-migration direction 即指文章矛盾现象。综合后文解释中的
比较,可知 E 选项:寒冷时期(适应向热带迁徙)比温暖时期(适应向寒冷地区迁徙)路桥更容易存在。

passage 69
文章结构分析:文章围绕 Colin Calloway 和 Bernard Bailyn 两人对同一现象(美国革命前英国政府对于
阿巴拉契亚山脉以西土地的封存)的不同解释以对比的形式展开。Colin Calloway 认为是英国政府预防
贪婪的白人殖民者掠夺印地安人土地。Bernard Bailyn 认为英国政府是出于垄断贸易、避免战争目的
而封存土地。
1.答案: D
题型:目的主旨题
解析:根据文章结构分析,可知 D 选项最准确。
2.答案: E
题型:目的题
解析: 通过高亮关键词定位到原文 White Americans seized Native American land by any available means,
they were continuing a tradition dating back to the earliest years of English settlement in North America。概
括该句可知选项最准确。
3.答案: E
题型:推理题
解析: 问题要求选择两人的共同点。E 选项针对两人同一解释对象。

passage 70
文章结构分析:文章首段指出特殊现象:一夫一妻制灵长类动物中存在父亲对婴儿的照顾现象。George
Edwards 观察到该类现象。
Kleiman 认为一夫一妻制,父亲对婴儿亲属关系的确信是该类现象的原因。Symons 为首认为一夫一
妻制在哺乳动物中相当罕见(意思是父亲对婴儿照顾现象也很少见)。第二段指出最近的发现可能颠覆
老观点。
Hardy 认为:1)灵长类一夫一妻制远比之前认为的更普遍。2)父亲照料远比之前认为的更普遍。之
前认为:一夫一妻制,父亲对婴儿亲属关系的确信导致父亲照料婴儿。现在认为存在替代可能性:父
亲照顾婴儿现象有助于( pre-adapt )一夫一妻制。
作者评论认为:两种情况都可能正确。父亲婴儿照料不是局部特殊现象。父亲在婴儿照料方面发挥比
之前认为更大作用。
1.答案: C
题型:目的题
解析:
该题可以用人名定位到第二段第二句。第二句服务于段首主旨句:Recent findings, however, make it
necessary to reverse this picture.可知 Hardy 的发现质疑了以前的结论,所以选 C。
2.答案: E
题型:细节题
解析: 该题考察作者对 evolutionary relationship between male care and monogamy 的总体评价。定位到
第二段作者评价:Either scenario could be true ,可确定 E 选项。
3.答案: C
题型:细节题
解析:根据关键词定位到原文:Whereas previously, it was assumed that monogamy and male certainty of
paternity facilitated the evolution of male care, it now seems appropriate to consider the alternative
possibility,其 that 从句说: that the extraordinary capacity of male primates to look out for the fates of infants
did in some way pre-adapt members of this order for the sort of close, long-term relationships between males
and females that, under some ecological circumstances, leads to monogamy.该证据质疑了老观点,可知 C
选择正确。该新证据并没有说之前的证据有问题,B 不准确。
4.答案: B
题型:加强题
解析:根据关键词定位到上下文可知 the alternative possibility 指的是 the extraordinary capacity of male
primates to look out for the fates of infants did in some way pre-adapt members of this order for the sort of
close, long-term relationships between males and females that, under some ecological circumstances, leads
to monogamy。即父亲照顾婴儿现象有助于( pre-adapt) 一夫一妻制。B 选项最能支持这一结论:父亲
照顾婴儿现象早于一夫一妻制。A 现象早于 B 现象,能够为称为 B 的原因提供支持。
Passage71-Passage80 解析
passage 71
文章结构分析:文章首段描述第十四条修正案痛苦的出台过程:从 1866 年的民权法案,经过立法、
司法、行政几方的权力角力,第十四修正案 1868 年终于获得通过。
第二段介绍第十四修正案出台后,立法机关的美好理想和司法机关的丑陋实践之间的矛盾。司法机
关通过案例判定施加的影响,背离了第十四修正案种族平等的理想。
第三段介绍第二次世界大战后,第十四条修正案才开始真正发挥作用。司法机关通过一些关键案例
的判定,真正落实乃至扩展了第十四修正案种族平等,反对歧视的思想。 该题针对文章主旨。
所以综上可知最好的答案是 C 选项。
1.答案: C
题型:内容主旨题
解析:
A 选项错,所概括内容与原文矛盾:The broad language of the amendment strongly suggests that its
framers were proposing to write into the Constitution not a laundry list of specific civil rights
B 选项针对第三段内容,范围过小。
D 选项与文章主旨矛盾。首段即明确指出对于起草者意图的共识。
E 选项针对第三段,概括也不准确。
2.答案: A
题型:细节题
解析: 回到关键词 state action limitation 定位到原文第二段:In the Civil Rights Cases of 1883,for
example, the Court invented the "state action”limitation, which asserts that “private” decisions by owners of
public accommodations and other commercial business to segregate their facilities are insulated from the
reach of the Fourteenth Amendment's guarantee of equal protection under the law。概括可知 A 选项,the
state action limitation 违背了第十四条修正案的平等原则, 纵容了私人领域的种族歧视和种族隔离现象。
3.答案: B
题型:削弱题
解析: 根据关键词 framers of the Fourteenth Amendment 定位到原文第 1 段:....all interpreters agree that
the framers' immediate objective was to provide a constitutional warrant for the Civil Rights Act of 1866,
which guaranteed the citizenship of all persons born in the United States and subject to United States
jurisdiction. 可知作者认为起草者的意图是给所有提供平等公民权。哪个选项把起草者提供平等公民
权的范围理解得最小,哪个正确。A 选项说,起草者预计到后文提到的 state action limitation,只保护
公共领域的平等权利,B 选项说 framers had merely sought to prevent discriminatory acts by federal
officials,只针对联邦政府官员的歧视行为。B 选项所指范围明显小于 A 选项。B 破坏力最强。
4.答案: B
题型:目的题
解析: 根据关键词 the original proponents 定位到原文:all interpreters agree that the framers’ immediate
objective was to provide a constitutional warrant for the Civil Rights Act of 1866, which guaranteed the
citizenship of all persons born in the United States and subject to United States jurisdiction, 可知 B 选项。
5.答案: A
题型:推理题
解析: 虚拟取非题。问题问的是在什么情况下第十四条修正案可能不会被颁布,回到首段第十四修正
案出台的关键因素:The act was vetoed by President Andrew Johnson, who argued that the Thirteenth
Amendment, which abolished slavery, did not provide Congress with the authority to extend citizenship and
equal protection to the freed slaves。可知是由于总统质疑 Congress,使得 Congress 推翻了他的反对才使
得这条修正案通过的。将该关键因素取非,可知,如果 Congress 没有受到质疑,这条修正案也不会被
通过了。
6.答案: D
题型:排序题
解析:由 the Civil Rights Act of 1866....... was designed primarily to counter the Supreme Court's ruling in
Dred Scott V. Sandford 可知 II 在 I 前;由 Congress promptly overrode Johnson's veto, supporters of the act
sought to ensure its constitutional foundations with the passage of the Fourteenth Amendment 可知 IV 在 III
前。又由于 Fourteenth Amendment 在 1868 年通过,所以总的顺序是 II I IV III,选 D。
7.答案: E
题型:推理题
解析: 根据关键词 the second of the two doctrines 定位到原文 Second,the Court relaxed the state action
limitation on the Fourteenth Amendment, bringing new forms of private conduct within the amendment's
reach。第二条 doctrine 放松了 state action limitation 对 Fourteenth Amendment 的限制,结合二段可知将
使得 private conduct 也将受到 Fourteenth Amendment 的约束,所以对于 commercial business 来说更难
进行 racial discrimination 了,选 E。

passage 72
文章结构分析:文章首段首先介绍传统老观念:认为大约 250 万年前的地球生物大灭绝是一个渐进的
过程,主要由气候变化引起;Yet 之后一些学者对老观点提出了挑战:PW 等通过考古证据,认为流
星撞击地球才是导致剧烈大灭绝的原因。第二段展开新老观点的证据,相互的辩驳及此消彼长。文章
最后,作者引出月球证据,支持了新观点。
1.答案: B
题型:目的主旨题
解析:该题针对文章主旨目的,通过文章结构可知选 B。
2.答案: C
题型:细节题
解析: 题干观点是第二段老观点,定位到第二段原文对老观点的质疑:The problem with ascribing all
tillites to glaciation, however, is that tillite deposits are widespread in Precambrian sediments,结合后句,
当时气候温暖,不应该产生如此多的 tillites,由此推出有其他来源产生 tillites。可知 C 选项是质疑老
观点、支持新观点的关键。
3.答案: D
题型:推理题
解析:文中说 Rampino and others question one of the traditionalists' basic assumption: the periodic
occurrence of glaciation throughout Earth's history.传统观念学者认为 terrestrial tillites 是由于冰川活动引
起的,而 Rampino 则认为是 meteor impacts 引起的。月球上的 certain Moon rocks 肯定是由于 meteor
impacts 引起的,如果能够证明 terrestrial tillites 和 certain Moon rocks 的相似性,就支持 Rampino 的观
点:terrestrial tillites 也是由 meteor impacts 导致的,也就驳斥了传统学者的观点:terrestrial tillites 是
冰川引起的。D 选项 正确。
4.答案: B
题型:推理题
解析: 回到原文 Supporters of the traditionalist view note that species begin disappearing from the fossil
record well before the end of the Permian period.Yet, while such a pattern could have resulted from a slowly
rising extinction rate, some scientists argue that just because a species disappears from the fossil record
doesn't necessarily mean it became extinct at that point。可以说明双方就某种生物从化石记录中消失所代
表的意义存在争论:老观点认为化石消失代表灭绝,新观点(some scientists )认为消失未必代表灭绝。
B 选项最准确概括双方分歧。

passage 73
文章结构分析:文章首段针对矛盾现象( Jane Austen 生活在浪漫主义时代,但被认为脱离于主流浪漫
主义男性诗人)展开老观点:Jane Austen 脱离于主流浪漫主义男性诗人,即使女权主义批评兴起之后。
第二段介绍相反观点:Jane Austen 和浪漫主义男性诗人是有密切关系的。
第三段介绍特殊案例:the present essay,从特定角度支持二段观点。
1.答案: C
题型:目的主旨题
解析:由文章结构,尤其二段三段关系,可知 C 选项最准确。
2.答案: C
题型:细节题
解析: 题目要求确认首段首句主旨句的主要支持理由。C 选项针对第二句主要理由:Although her dates
(1775-1817) place her squarely within the period, she traditionally has been studied apart from the male
poets whose work defined British Romanticism for most of the twentieth century。
3.答案: A
题型:细节题
解析: 题目针对首段"historicist and feminist criticism"和之前观点的区别,定位到原文 which challenged
many previous characterizations of Austen as detached from the major social, political and aesthetic currents
of her time, she continued to be ....可知 A 选项。
4.答案: B
题型:推理题
解析: 定位到原文 she continued to be distinguished from her male contemporaries....for exam.....Anne
Mellor declares that Austen, along with other "leading women intellectuals and writers of the day" "did not",
participate in the Romantic "spirit of the age" but instead embraced an alternative ideology that Mellor labels
"feminine Romanticism”可知包括末句在内的历史主义和女权主义批评没有化解,而是加重了首段首
句描述的矛盾现象。B 选项最准确。

passage 74
文章结构分析:文章理由说 trucks 对公路的破坏取决于单 axle 承重量的提升,新规定增加了允许卡车
载重的总量。结论说新规定会允许能对公路造成更加严重的破坏下车上路。
答案: B
题型:逻辑单题削弱题
解析:从理由到结论的推理中忽略了相关重要因素,因为:破坏力(单轴承重量) =总重量/轴数。B 选项
最准确描述逻辑漏洞。

passage 75
文章结构分析:文章理由提到,EP 自然条件下生长需要火,火创造两个关键条件:
1)一氧化氮使其发芽,
2)清除灌木,使其能见阳光。
答案: E
题型:逻辑单题推理题
解析:
人工条件提供了自然关键条件 1),E 选项是正确推理,不满足第 2 个条件,该植物无法长期存活。

passage 76
文章结构分析:文章首先指出一个现象:全球海平面相对陆地上升,解释原因是:南北极夏季融化雪
多于冬季形成雪。紧接着说 this hypothesis is not undermined by observation:斯堪的纳维亚的海平面相
对陆地下降(首句黑体) ,之后是针对该矛盾的解释原因。
答案: C
题型:高亮句子功能题
解析:分析段落结构可知 C 选项。

passage 77
文章结构分析:文章首先指出现象:编辑们因为国际新闻页面的减少而被批评。编辑关键是:读者应
该为减少担负全责,因为读者对于国际新闻的兴趣减少导致相关报道销量减少(首句黑体);但是作者
认为:编辑仍然应当承担部分责任,理由是:编辑可以通过提高报道频率的方式来增加读者的兴趣。
答案: A
题型:高亮句子功能题
解析:由结构分析可知,第一个高亮句是支持编辑的理由,编辑的立场是作者反对的立场;第二个高
亮句是作者支持自己立场的理由,也是批评编辑辩解的理由。A 选项最准确。

passage 78
文章结构分析:文章背景信息是:B 在冬天将时间拨慢一个小时。 文章理由是:冬令时调整不会增
加黑暗中的驾驶数量(时间) , 文章的核心观点是:冬令时调整不会影响车祸概率。
答案: B
题型:逻辑单题假设题
解析:整理清楚文章结构后,主动考虑破坏结论的因素。如果因为早上和晚上微妙的差异,导致时间
调整后早上事故率的降低大于晚上事故率的下降,那么作者的结论就会被破坏。文章推理要成立,必
须假设结论被破坏的可能性不存在。B 选项否定了早上和晚上的差异。

passage 79
文章结构分析:文章首句介绍特殊 EB 作品的特殊现象:作品质量高导致大家容易忽略其低产量。第
二句 but 转折后说 EB 本人对此很在意抱歉。之后是作者的评论:作品数量低促成后代对她的特殊高
评价,但也是导致她与当时文学界疏远关系的原因。
1.答案: AB
题型:细节题
解析: A 选项定位到末句:Yet the size of her work is also a sign of Bishop's alienation, that is, her uneasy,
resistant relation to the literary culture that today claims her as the major poet of her generation.
B 选项定位到原文 The small, manageable size of Bishop's body of work has facilitated its extraordinary
critical reception since her death.
2.答案: A
题型:词汇题
解析:定位到原文上下文:But the size and pace of her output were always in her own eyes a failing
(vaguely moral in complexion) for which she apologized throughout her career.短语"vaguely moral in
complexion'是对主句的解释,从主句 in her own eyes 可知 complexion 和"视觉"有关,A 选项 appearance
最合适。短语“vaguely moral in complexion 理解为:看起来有微弱的道德意味。

passage 80
文章结构分析:背景:Shoreland County 买下了一块荒地防止其被开发,为此,Shoreland County 放弃
了这块地上未来的财产税。理由:财产税基于市场价值,如果进行开发会带来巨额的税收。结论:由
于这次收购拒绝开发,Shoreland County 的税收总收入会比开发少很多。
答案: D
题型:逻辑单题假设题
解析: 假设题。注意到理由结论之间的概念变化:理由中 overall annual tax 考虑的是待开发 land 本身,
结论中 overall annual tax 潜在来源需要考虑的是整个县,不仅要考虑待开发 land 本身,还要考虑待开
发 land 周围地区。D 选项弥补了理由结论间的潜在概念漏洞。
Passage81-Passage90 解析

passage 81
文章结构分析:文章首先描述现象:内战后,很多前奴隶成为 sharecroppers 或者 tenant farmers。第二
句介绍主流历史学家的观点:把两个黑人改革的代表人物( RLS,JEC )看作 anti-racists。之后是作者的
评论:两人主要是务实的改革家,尽管有反种族主义的成分。文章末句总结:尽管如何认识他们的影
响有争议,他们确实作出了实际的贡献。
1.答案: C
题型:目的主旨题
解析: 该题针对文章主旨。通过文章结构分析可知 C 选项最准确:作者的评论举证反驳了主流历史学
家的观点。文章并没体现历史学家对改革者的毁谤,所以也不存在恢复名誉,A 错;文章没有同时代
人对这两个改革者的批判,B 错;文章没有讨论改革者为什么会出名, D 错;文中也没有详述 farm
tenancy system 对南方经济的影响,E 错。
2.答案: D
题型:细节题
解析: 题目问作者对两位改革者的评价。文章最后一句典型体现作者对他们的评价:While the extent of
these reformers' influence requires more study, early their organizations provided a voice for African
American farmers seeking to improve their positions in the agrarian South. D 选项最准确。

passage 82
文章结构分析:律师辩护理由:TEB 已经根据自己的规定奖励了举报人员,开除了 wrongdoers。律师
辩护结论:TEB 做出足够的努力来防止欺诈政府的行为,指控是错误的。
答案: A
题型:逻辑单题削弱题
解析:针对理由和结论之间可能的概念跳跃,如果说 Wrongdoing 的定义是危害 TEB 公司自身的行为,
那么即使 TEB 公司开除了 Wrongdoers,也不能成为"TEB 没有做出足够的努力来防止欺诈政府的行为
"辩护的理由,因为指控所定义的过错和公司所定义的过错是不一样的,因此 A 选项正确。

passage 83
文章结构分析:文章首句及第二句指出特殊事实:当代北极植物是最近 65 million 年才迁移到北极的。
针对该事实的一种观点解释:这些植物起源于中亚和北美高山,随气候变冷迁移到北极地区。However
转折后,作者指出支持前述观点的证据缺失。
1.答案: B
题型:推理题
解析:该题需要根据选项定位原文。B 选项可定位到原文:the routes by which these plants expanded their
ranges during their colonization of the Arctic remain unknown。
2.答案: C
题型:词汇题
解析: 根据上下文:Only toward the end of that period does the fossil evidence show that certain
present-day Arctic plants were established and widely distributed throughout the Arctic. Many Arctic plants
are thought to have originated in the high mountain ranges of central Asia and North America, to have
spread northward to the Arctic..... 根据下句 originated, spread 可确定 distribute 是分散、传播的意思。

passage 84
文章结构分析:文章首先通过和德国的比较,描述挪威工业化相关的特点:历史悠久的相对平等的阶
层关系。之后分析该平等的原因。
1.答案: D
题型:内容主旨题
解析:该题问文章主旨,根据文章结构分析可知 D 选项最准确。
2.答案: A
题型:细节题
解析: 根据关键词定位到原文:a strong aristocracy and landowner class did not emerge in most of Norway.
There were some exceptions to this pattern, especially in the southern regions where a landowner class did
exist,可知 A 选项。
3.答案: C
题型:细节题
解析: 选项定位。C 选项可定位到最后一句:Norway's early social and economic history engendered
egalitarianism, although, as has been pointed out by several observers, it was an equality of poverty。

passage 85
文章结构分析:文章是现象解释结构。现象:同样开始收费,Rivera Art Museum 游客下降比其他本地
博物馆更严重。原因解释:博物馆靠近政府办公楼,所以游客主体为政府工作人员。当博物馆收费后,
这些人选择不在午餐休息时间逛博物馆导致了短时参观人流减少。
答案: C
题型:逻辑单题削弱题
解析: C 选项采取求异法对比思路:非工作日(周末)的长时参观数量下降一样多甚至可能更多,否定了
原文针对短时参观的解释。

passage 86
文章结构分析:文章背景信息:金星大气中二氧化硫由火山活动产生,由钙吸收。文章理由:金星当
前大气中有大量二氧化硫。文章的结论是:Venus 上现在有火山之类的产生 sulfur dioxide 的源头。
答案: A
题型:逻辑单题假设题
解析: A 选项取非,金星过去二氧化硫比现在显著更高,意味着现在的二氧化硫有可能是过去遗留的,
现在只有吸收,没有释放来源。

passage 87
文章结构分析:文章针对火星北部低平表明讨论原因解释。首段和第二段开头讨论解释 1:类似地球
的不同地壳导致的板块移动最终导致该低地。第二段 however 转折后,McGill 指出早于版块移动的地
质构造,质疑版块解释说,提出新机制说。第三种解释是外部冲击。
1.答案: D
题型:目的主旨题
解析:该题针对文章主旨,根据文章结构分析可知 D 选项最准确。
2.答案: A
题型:推理题
解析: 根据关键词定位到原文首段:On Earth's surface, higher-and lower-lying areas have different types
of crust: one, thin and dense, is pulled toward Earth's center more strongly by gravity, and the planet's water
naturally comes to sit over it, creating oceans.板块动力来自行星内部,可知选 A 选项是最准确概括。
3.答案: B
题型:细节题
解析: 问题考察 McGill 的理论与别人不同的地方。回到原文 McGill instead believes that through some
novel mechanism the ancient surface sank to its current depth as a single unit 可知他的理论并没有明确指
出到底是什么作用导致 the ancient surface sank to its current depth,只是模糊的说 through some novel
mechanism,而第一种理论说 Some researchers do see signs of tectonic activity surrounding the northern
basin that suggest that it was created through the formation of new crust, like ocean basins on Earth ,即假设
这种作用是引力, 第三种理论说 The third possibility is that the northern low lands result from impact,假
设这是陨星撞击的作用。因此,选 B。

passage 88
文章结构分析:文章针对一种现象(CH 东部 abalone 渔业的衰败)的原因解释(过度捕捞)提出替代性
解释( Rock lobster 的引入),然后描述 Rock lobster 导致 abalone 下降的生态链条机制。
1.答案: B
题型:细节题
解析: 该题是细节题,根据时间线索和关键词 sea urchins 定位到原文 an invasion by rock lobsters during
the early 1990s ...... prey on sea urchins, and increase drock lobster densities coincided with significant
decreases in urchins。可知从 early 1990s 开始,由于龙虾入侵,海胆数量减少,所以选 B。
2.答案: C
题型:细节题
解析: 细节定位题。C 选项能定位到原文:urchins feed largely by trapping drift kelp, and in doing So
provide juvenile abalone with both protective shelter and nourishment. Without urchins' presence, juvenile
abalones are less likely to survive to adulthood 可知海胆对于幼年期的 abalone 的生存和成长非常重要,
所以选 C。

passage 89
文章结构分析:文章推理是典型目的手段结构。目的:阻止偷猎象牙到达市场。手段:象牙进口国家
禁止进口象牙。
答案: A
题型:逻辑单题假设题
解析: 文章必须假设手段能够实现目的。A 选项取非,非法渠道仍然能够到达出口市场,严重削弱结
论:如此,则手段不能实现目的。

passage 90
文章结构分析:文章是典型现象解释结合对比结构。文章直接明确特定现象的原因:早期英国殖民地
失败(尽管资源丰富)的原因就是:定居者的思维定势使得他们不能学习当地印第安人的关键生存技能
一一移动。 后文对比展开欧洲人和印第安人不同的对居住的理解。该题需要选项定位。
1.答案: B
题型:细节题
解析:
A 选项与原文信息相反。
B 选项正确。原文说:....failed amid the New World's abundance was that the settlers' mindset prevented
them from living like the native inhabitants.....but they did not adopt the real key to success:
mobility....changing food resources....
C 选项不能从原文得出。原文认为欧洲人学习了本地印第安人的技术:the settlers learned such skills as
building brushwood weirs to trapfish...
2.答案: D
题型:目的题
解析: 根据关键词定位到原文,结合文章结构可知 D 选项。
Passage91-Passage100 解析
passage 91
文章结构分析:针对特定现象(怀孕老鼠没有对婴儿老鼠产生通常会产生的蛋白拒绝。解释 1 是:环
绕婴儿老鼠的 placenta 阻止了母亲的免疫系统。解释 2 是:placenta 产生的 IDO 起到关键的阻止作用。
答案: E
题型:逻辑单题评价题
解析: 题目要求比较两种解释的有效性。利用求异法,检测两种解释的关键区别(有无 IDO)来检测是
否有免疫反应的关键结果。E 选顶最好体现该思路。

passage 92
文章结构分析:文章是典型的观点论证结构。首句表达作者判断观点:科学的推广流行化坏处比好处
多。后文给出论证解释。作者指出,科学的最重要特征不是这个那个特定结果。相反,科学的成就在
于把各种现象联系起来,把各种预测、解释联系起来。而科学的推广流行恰恰系统性地破坏科学渐进
式推理的性质,破坏科学对于各种原则、习惯、实验、规则的联系。坏的科学写作破坏科学最有趣的
特征,把科学的长线的有联系的论证分裂成孤立的特定结果。黑洞被说成是巨大的漏斗,量子电子轨
道被比喻为消散的云团。
答案: D
题型:目的题
解析: 根据关键词定位到文章末举例,知其是 bad science 的 isolated metaphor。

passage 93
文章结构分析:文章建议手段是:使用海底电缆的方法,文章结论是:该新手段能保护游泳者,保持
旅游业兴旺。
答案: E
题型:逻辑单题削弱题
解析: 如 E 选项所说,作为吸引旅游者主要资源的海洋哺乳动物会因为海底电缆而被驱逐,则该手段
导致主要目的不能实现,削弱论证。

passage 94
文章结构分析:文章首句是总体判断:幼年个体在与熟悉同种成员的互动中比与非熟悉同种成员的互
动中学到更多。二句(K 与同事们)与三句(V 与同事们)的实验结果都被用来支持首句作者判断。
(At the time)句指出,前两句的实验结果在当时出乎预料,与 G 及其同事们 1984 年的报告矛盾。
末句(however 句),G 等人的证据支持了熟悉性效果。
答案: C
题型:推理题
解析: 关键词定位,结合文章结构可确定 C 选项,他们得出的结论是作者反对的观点。

passage 95
文章结构分析:与 Linda Nicholson 不同,Katherine Lynch 认为女性志愿者活动让女性联系到一起形成
了一种非正式的公民权,形成了社会的联系,有了公共生活,而不是被限制在家庭中。
1.答案: B
题型:细节题
解析:题目问 Lynch 认为这些志愿者活动是怎么样的。文章中主线是 Lynch 说女性志愿者活动让女性
与社会联系,不局限于私人领域。当然也有让步说志愿者活动不能替代正式的政治生活。
A.错误,Lynch 认为志愿者活动是公共活动的一部分,但是并不能替代政治活动。并没有说志愿者
活动是政治生活的初期阶段。
B.正确,Lynch 认为志愿者活动对女性的公共生活有作用,需要更多的重视,被 Nicholson 之类的
人低估了。
C.错误,这是和 Lynch 相反的观点。
D.错误,没有说过女性志愿者活动能改变政治结构。
E.错误,没有说过这种活动是不是非典型的。
2.答案: B
题型:细节题
解析:问 Lynch 认为正式的政治生活怎么样。文章中让步说正式的政治生活不能被志愿者活动替代;
最后一句也说了公共生活并不只是政治生活,可以推出政治生活和志愿者活动都是公共生活的组成部
分。
A 错误,没有提到政治生活越来越重要。
B 错误,没有提到女性政治生活被低估。
C 错误,政治生活并不是社会联系所必须的,志愿者活动就可以帮助女性形成社会联系。
D 正确,文中说了公共生活不止是政治生活,但也说了志愿者活动不能替代正式的政治生活。
E 错误,没有说政治生活和 shadow citizenship 之间的关系。

passage 96
文章结构分析:
文章背景信息:两个欧洲团队(荷兰商人和法国传教士)试图发现中国人制造瓷器的秘密。
理由:法国传教士团队的杂志,直到 1717 年才发表相关成果,比欧洲瓷器制造晚了好几年。
结论:欧洲瓷器制造者是通过实验掌握瓷器制作技术的,并非抄袭中国技术。
答案: E
题型:加强题
解析: 题目要求支持论证。
A 选项是削弱选项,如果欧洲和中国陶瓷技术相同,则更可能是欧洲学习中国了;
B 选项依然是削弱,欧洲陶瓷和从中国引入的陶瓷相似,那么很有可能证明是学习了中国;
C 选项依然是削弱,如果这些手稿是在 1717 年从中国寄回去的话,那么则很有可能说明欧洲陶瓷就
是利用这些手稿来学习中国技术;
D 选项属于无关选项,陶瓷材料之前没有被使用过不能证明欧洲人是自己研究出的陶瓷工艺;
E 选项正确,一个和中国陶瓷没有任何往来的陶瓷制造商确实能加强欧洲陶瓷是资助研发这一结论。

passage 97
文章结构分析:文章首先指出现象:编辑们因为国际新闻页面的减少而被批评。编辑关键是:读者应
该为减少担负全责,因为读者对于国际新闻的兴趣减少导致相关报道销量减少(首句黑体) ;但是作者
认为:编辑仍然应当承担部分责任,理由是:编辑可以通过提高报道频率的方式来增加读者的兴趣。
答案: A
题型:高亮句子功能题
解析:由结构分析可知,第一个高亮句是支持编辑的理由,编辑的立场是作者反对的立场;第二个高
亮句是作者支持自己立场的理由,也是批评编辑辩解的理由。A 选项最准确。

passage 98
文章结构分析:H 这个东西通过吸入沉淀来摄取微生物,而重金属也经常吸附在这些沉淀上,所以 H
可能会被沉淀上的重金属中毒,所以我们可以用一个地方的 H 的含量来看这个地方重金属的含量。
答案: D
题型:逻辑单题假设题
解析:那么这个结论就假设了 H 一定会去吸入沉淀来获取微生物,所以 D 选项符合答案,因为如果微
生物被别的污染物杀死了的话,那么 H 就不会去吸入这些沉淀了,也就不会被重金属中毒了。

passage 99
文章结构分析:吃哺乳动物的鲸聚小群体,吃鱼的鲸聚大群体,然后原因是吃鱼的鲸聚集大群体是因
为他们要合作捕鱼,吃哺乳动物的鲸如果聚集太大就会被哺乳动物发现。
答案: A
题型:逻辑单题削弱题
解析:按找他因思路即可,A 选项符合,因为说吃鱼的鲸自从出生就聚在自己的群体中,而吃哺乳动
物的鲸聚小群体是因为一旦群体变大他们就会出去组新的群体。

passage 100
文章结构分析: 雄性狮子的鬃毛有不同的特征。S 学者认为鬃毛的作用是保护雄狮的身体。且颜色更
深的鬃毛要好过浅色的作用。但是目前这个结论无法验证。
1.答案: B
题型:高亮句子功能题
解析:根据句子的句内关系,高亮句子的后半句在此基础上提出了一个推测。
2.答案: A
题型:细节题
解析:定位 but direct tests of this prediction are difficult。之后的句子里面回答为什么这个预测的直接验
证是困难的,在于观测的难度。
Passage101-Passage110 解析
passage 101
文章结构分析:文章首段介绍对森林角色的新认识:森林砍伐增加二氧化碳释放,加速全球变暖。
第二段二氧化碳释放来源的角度讨论森林在全球变暖问题中的重要性。三个二氧化碳释放的主要来
源(1 石油燃料燃烧,2 树木腐败或燃烧,3 中高纬度森林突然有机物的腐败)之中,有两个和森林相关。
第三段重点纠正对森林重要性认识的误区:过于关注热带森林生物多样性。作者指出,中高纬度森
林很重要,森林本身支持生命的能力很重要,应该认识到森林对土壤与大气的重要性。
第四段总结强调森林的重要性:对于陆地生命,对于稳定大气构成都很重要。
1.答案: A
题型:内容主旨题
该题针对文章主旨,根据文章结构尤其是第四段总结,可确定 A 选项。
2.答案: A
题型:细节题
解析: 根据关键词 biodiversity 定位到第三段。A 选项可定位到该句:Forests are far more than simple
collections of species, however, it is unfortunate that the discussion of biotic or living resources has been
focused on biodiversity rather than on the actual ability of the land itself to support life.
3.答案: E
题型:细节题
解析:同上题一样定位到第三段。E 选项定位到该句:But forests, it should be emphasized, have a similar
role in every latitude they contain the largest numbers of different kinds of plants and animals of any
community on land and might be considered the most highly developed of the terrestrial communities from
the standpoint of complexity of structure and diversity of life and life forms.
4.答案: C
题型:目的题
解析: 题目针对第三段主旨,根据第三段结构分析可知 C 选项最准确。D 选项原文有依据:Forests are
far more than simple collections of species。但该内容是支持段落主旨的理由。

passage 102
文章结构分析:本文时间线索较复杂。
首句总体陈述:审美和技术考虑的对立是地图绘图界的重要矛盾。
从 19 世纪(1800 后),绘图者多数把自己当作技师,不强调视觉效果,而另一些把自己当作地形画
家。直到 1980 年,正如 B&H 称谓的“旧的是美的典范”,学者们关注 1800 年之前的地图,惊奇于
其艺术美,哀叹前技术时代艺术性的衰落。早期地图制作被当作艺术,而现代绘图属于实用工程技能
领域。末句,作者引用 Alpers 观点:艺术与技术的边界区分会让 17 世纪(1600 年-1700 年)的地图
制作者困惑,因为他们把自己当作“视觉工程师”(强调艺术和技巧的统一)。
1.答案: A
题型:细节题
解析: 由“paradigm”定位原文,可知该观点强调艺术性。
由 Alpers 定位最后一句,17 世纪的制图者并不存在美学和技术的分水岭,17 世纪的制图者们认为
自己既考虑美学因素也考虑了技术因素。
Alpers 所属 17 世纪是“prior to 1800”,对比以上各自可知两者矛盾不一致。
2.答案: B
题型:推理题
解析: 通过时间词 1980s 定位,Until the 1980s, in what Blakemore and Harley called "the old is Beautiful
Paradigm," scholars largely focused on maps made before 1800, marveling at their beauty and sometimes
regretting the decline of the pre-technical age.
文章里交代的是 20 世纪 80 年代之前的状况,题干问的是 20 世纪 80 年代之后的情况。Until 和题
干里的 beginning 告知我们这个推断题专门考查取反逻辑,时间互相取反,那么句子里的动作也应当
同时取反。再结合句内提到学者们开始注意到 1800 年以前的地图非常精美,有艺术成就,那么我们
合理推断他们是有对照物的,才能得出前者更美的结论。因此答案选 B。

passage 103
文章结构分析:文章首先提出待解决问题:类人猿是否有青春期?
1950 年代 JG 的数据缺乏支持证据。
最近对 JG 数据的再研究中,EW 发现黑猩猩四肢在接近性成熟期的加速生长。
多数生物学家怀疑 EW 发现的是类似人类的青春期:人的青春期生长很明显,影响整个身体,黑猩
猩的不明显。HS 发现,两者开始的时间对应的头盖骨发育比例也不同。
1. 答案: D
题型:内容主旨题
解析: 虽然猩猩经历青春期发育的理论有了一方支持,但是多数生物学家对此仍保留怀疑态度,并提
供了两个怀疑的理由。
2. 答案: B
题型:细节题
解析: 定位 While the human adolescent growth spurt is physically obvious and affects virtually the entire
body, the chimpanzee's increased growth rate is detectable only through sophisticated mathematical analysis.
人类的青春期发育肉眼可见,且影响周身,相反黑猩猩的这种发育加速只能通过复杂的数学分析确
认。差异性比较可知,不如人的明显。
3. 答案:C
题型:推理题
解析:定位 While the human adolescent growth spurt is physically obvious and affects virtually the entire
body, the chimpanzee's increased growth rate is detectable only through sophisticated mathematical analysis.
原因同上题,黑猩猩的发育加速只可通过复杂数学分析确认。

passage 104
文章结构分析:首句指出文化人类学者的新观点,新趋势:拒绝 1970 年之前的经验主义科学框架,
继而把所有客观科学知识当作社会主观构造的。二句三句展开文化人类学的观点:经验主义时代的文
化信息典型地来自于预设偏见的人类学者。文化人类学不可避免是主观的。作者提出建议:人类学应
该转向更加人性化的文化解释,停止对于文化行为的数据建设。新的研究框架认为:与其继续实验的
方法,调查之前的偏见可能更有意义。
1.答案: C
题型:推理题
解析:定位文章第一句,现在的文化人类学家开始逐步反对 1970 代之前的文化人类学家的观点了。
之前的是经验主义的。注意那个时间状语是在从句里,不可以迁移到主句中去,很多同学错误理解了
这个句子的时间而导致错误。
2.答案: D
题型:细节题
解析:现在的文化人类学家认为在研究上应当停止建立数据库,而应该去检查之前的内容,要给予这些
内容更人文的解读。

passage 105
文章结构分析:文章首先介绍 LH 注意到的矛盾现象:19 世纪的女性小说家都用笔名或匿名,4 位女
性旅行作家用自己真名发表作品。文章后文对该现象给出解释评论:女性小说家意识到自己的革命性,
女性旅行作家拥护传统价值,回避激进性,回避女权运动。女性小说家批评社会,而女性旅行作家满
足现状,实现个人自由。作者认为女性旅行作者生活于矛盾中,个人孤立状态服从于权威,排除了集
体行为或女性团结的可能性。
1.答案: 选 E
题型:细节题
解析:定位 contradiction,前后文都有线索,女旅行家用真名写作,都是独自旅行,不符合当时的女
性行为标准,已经挑战男性权威,行为很先进。相反她们的意识都支持传统价值,远离女性运动,相
对落后。所以作者说,她们活在矛盾中。
2.答案: 选择 AB
题型:细节题
解析:定位句 For the subjects of Harper's study, solitude in both the private and public spheres prevailed—a
solitude that conferred authority, hitherto a male prerogative, but that also precluded any collective action or
female solidarity. 解析同上题。
C 选项,它缓解了必须遵从传统价值的,来自周围人的压力。无中生有。

passage 106
文章结构分析:文章首句给出现象及可能的原因解释:19 世纪女性小说当时很流行,但被 20 世纪的
精英读者冷落,说教是可能的原因。二句 however 转折,作者提出对首句解释观点的反驳:美国文学
有说教传统。在 19 世纪,忽视道德说教的小说被批评,有说教的被赞扬。作者最后给出自己认为正
确的原因解释:是二十世纪的美国变成了世俗社会,是二十世纪的评论者,把审美标准放置在所有其
他标准上面。
1.答案: B
题型:高亮句子功能题
解析:定位句 In the nineteenth century, critics reviled Poe for neglecting to conclude his stories with pithy
moral tags, while Longfellow was canonized for his didactic verse.用句子结构 A,while B 构成一个对比的
例子,体现出 19 世纪人们的偏好。定位句是 19 世纪的举例,对应之前出现第一句话里的矛盾。
2.答案: D
题型:词汇题
解析:词汇题,conveying“传送;通知”,imparting 告知,把..分给;imparting knowledge 固定用法,
表示传授知识。

passage 107
文章结构分析:首句描述现象:更新世的大象矮化。作者继而指出该现象可能存在普遍性。几种可能
解释:1 岛上无天敌促进矮化,2 岛上缺乏资源导致矮化。
末句指出有趣的相反现象:小型哺乳动物,在岛上存在大型化现象。
1.答案: D
题型:目的主旨题
解析:为大象矮化这一现象提供了多种可能解释。
2.答案: B
题型:细节题
解析:定位句 Interestingly, the island rule is reversed for small mammals such as rodents, for which
gigantism is favored under insular conditions.岛屿上,相对而言,对于小型动物体格大更有利于生存。

passage 108
文章结构分析:20 世纪早期,杂志及其刊载的新式图像塑造了先锋派的风貌,预示着进步的理念和不
同于传统的风格。
1. 答案: D
题型:细节题
解析:定位句 because it dispensed with the cardinal rule of graphic design: to take an idea and make it
visually clear, concise, and instantly understood.
2. 答案: A
题型:细节题
解析:提问先锋艺术家们创作出的无明确意义的图像目的是 This overturning of convention, this
assailing of standard graphic and typographic formats, was part of a search for intellectual freedom.
BC 均是未在文章中出现的内容,无中生有

passage 109
文章结构分析:文章主要介绍 H&S 关于“nation-state”的观点及其意义。
作者首先指出,H&S 认为 “nation-state”是个错误术语,理由是典型的单一语言,单一文化的国家
从未存在,即使在欧洲也不存在。他们认为,现代欧洲国家,是通过特定民族对其他族群的控制实现
的。 “nation-state”的概念,他们认为淡化了这些国家的内部文化语言多样性,应该称为“征服国
家”更恰当。
作者继而介绍该观点更正的现实意义:多样族群在 “nation-state”的复活,不是对主权国家的威胁,
而是开创了新的可能性:被征服族群与征服族群享受平等权利的机会。
1.答案: A
题型:目的主旨题
解析:HS 通过对学术用语(民族国家)提出质疑,来讨论与某种政治组织形式相关的问题。
2.答案: B
题型:目的题
解析:注意问法,问的是作者引用 U 和 S 的话,目的是什么。找到定位句“potentially threating to the
sovereign jurisdiction of the state”作者引用这句话,纯是为了总结对方的观点。
A 选项,尽管文中出现了 sovereign jurisdiction 字样,但作者没想讨论扩展民族国家这个概念所带来
的法律后果。
C 选项,没有 external threats 的事,也看不出作者 shift focus 了。
D 选项,两种观点不同,没有什么 similarities 。
E 选项,文中没具体说过种族中心政策的影响。
3.答案: B
题型:细节题
解析:提问 H 和 S 两位学者如何定义 nations 这个概念。缩写 i.e.= that is 明显提示了答案的所在位置。
他们认为不存在所谓的 “ 民族国家 ” 即单民族国家,所以选 B,所有国家都是不同族群的集合。
A 选项错在 similar concerns。
C 选项,不可能 homogeneous。
D 选项,语言不同,的确是国家的一个特点,但不是 H 和 S 认定的决定因素。
E 选项,有国界即称为国家。同上,这是国家的特点,但不是决定因素。

passage 110
文章结构分析:文章首先描述现象:1910 年至 1913 年,美国女性选举权主义者组织了游行。传统上,
游行是男性表达团结、破坏秩序规则的活动。女性游行挑战了传统对女性的概念,对女性行为的限制。
While 句介绍最近学者们对女性游行负面作用的理解:女性要求团结的努力导致媒体和公众的怜悯同
情,也招致反女权主义者的反对。
1.答案: E
题型:细节题
解析:定位句 From 1910 to 1913, women suffragists in the United States... ...women' s very presence in the
streets challenged traditional notions of femininity and restrictions on women's conduct.
可见男女游行都有这个特点。
2.答案: D
题型:推理题
解析:组织和参与游行的人,与 proponents of the“rules of social order”观点对立。
Passage110-Passage120 解析

passage 111
文章结构分析:晚期研究 JA 的著作,将重点放在动荡的时代背景中。
1.答案: C
题型:目的题
解析:传统的男性行为在 JA 的时代改变了。
2.答案: D
题型:推理题
解析:仍旧没有远离文章的主题,传统的男性行为在 JA 的时代改变了。

passage 112
文章结构分析:YLE 给自己设定的目标是重现菲律宾裔美国人的生活史。
1.答案: D
题型:细节题
解析:Espiritu' s subjects document their changing sense of Filipino identity in the United States, much as
Bulosan did as a member of the first substantial wave of immigrants.
2.答案: E
题型:词汇题
解析:同义词 vividly。

passage 113
文章结构分析:T Dilworth 的 New Guide to the English Dialogue 被广泛用于美国的阅读教材。其入门
读物强调孩子对所读材料的理解。
1.答案: A
题型:细节题
解析:定位句 In contrast, many earlier authors assumed that, just as introductory Latin texts taught children
an unknown language, introductory English texts should teach English as if it, too, were an unknown
language.
2.答案: B
题型:推理题
解析:作者同意以下哪种比 D 更早的英文入门读物作者的批评,这个文章里面没有直接写到,需要推
断。依据是 D 不同于早先的作者,并且 D 更注重培养孩子把书面印刷信息转化为口语来理解。因此
选 B。

passage 114
文章结构分析:许多学者认为,二战期间美国政府在南方对于制造业的投资直到战后仍然刺激着本地
的经济高速发展。但这些投资多被用于特定的工厂,其中许多不适合战后的生产。
1. 答案: B
题型:目的主旨题
解析:Many scholars have argued that government investment in manufacturing in the southern United
States during the Second World War spurred a regional economic boom that lasted into the postwar period.
其后的内容,都是作者反驳这种观点。
2. 答案: AB
题型:细节题
解析:选 AB 原文第九句和第八句对应两个题目。
3. 答案: E
题型:目的题
解析:举例子支持前一句。

passage 115
文章结构分析:12 世纪 C 遗址的研究,发现这种在 18,19 世纪相当繁盛的动物在 12 世纪的时候踪
迹难觅。
1.答案: E
题型:目的题
解析:定位句在最后一句,根据前句和再前句的两次转折,可以知道考古学家内部出现了分歧,分裂
成了两派,并且作者 defend 了前者。
2.答案: E
题型:削弱题
解析:提供他因,旅鸽不会在某地栖息太久,可能在 1 2 世纪旅鸽未在 C 地区周围生活。

passage 116
文章结构分析:女权主义学者,倾向于把选择单身的十九世纪美国女性视为争取女性独立的斗士,以
及对于婚姻作为一种压迫性制度的批判者。作者认为,这种说法扭曲了女性单身的真实意图。
1.答案: D
题型:细节题
解析:The nineteenth century saw the elevation of marriage for love as a spiritual ideal.
怀疑自己的婚姻,不能达到自己对理想婚姻的期待。
此处 Doubt 的理解不再是 challenge,而是 not sure。
2.答案: A
题型:细节题
解析:要记得选择作者的理解,把单身的选择归因于一种特定的,对婚姻的态度。

passage 117
文章结构分析:首句提出很多研究者认同的老观点:鸟和哺乳动物很多的生理和行为的相似性来自它
们共同的内热机制。二句之后介绍 Farmer 的反驳新观点:父母关照是对鸟和哺乳动物相似性的更关
键解释。传统上认为的内热机制的优势:栖息地扩展和承受剧烈运动。但 Farmer 认为新陈代谢提高
才是栖息地扩展的关键。内热机制与承受剧烈运动也没有生物意义的因果关系。Farmer 认为,内热
机制是作为孵化温度控制能力演化出来的。承受剧烈运动的能力是独立演化的,作为提高父母为幼仔
觅食的能力独立演化的。
1.答案: A
题型:目的主旨题
解析:少数派 Farmer 的解释驳斥了多数派 many researchers 的解释。
2.答案: A
题型:组织结构题
解析:考察各论点论据展开的时间先后。
该文章从针锋相对的两个解释展开,第二个解释后面提供了依据做论据。
3.答案: E
题型:细节题
解析:定位句 According to Farmer, while endothermy provides an explanation for a few similarities, such as
the presence of body insulation.

passage 118
文章结构分析:文章是典型的驳论结构。
首段介绍关于睡眠功能理论的主导观点:M 的消极不动假设,认为睡眠不是作为恢复功能,而是作
为安全保护功能的,不受捕食威胁的动物睡眠最安全。
二段 however 转折作者提出反驳,M 的假设有不能解释的睡眠现象:睡眠缺乏之后的睡眠反弹,睡
眠分为不同状态,每种状态就有不同的功能。
1. 答案: AB
题型:细节题
解析:
A 选项定位句 Neither does the hypothesis explain the existence of various states of sleep, which
themselves may be associated with different functions.
B 选项定位句 However, that hypothesis cannot easily explain why one often observes a rebound in sleep
time or intensity following a period of sleep deprivation。
2. 答案: B
题型:高亮句子功能题
解析:Reasoned that 表示对前面所做表述的解释,是常见的引导论据支持论点的说法。所以是 the basis
of a hypothesis。
passage 119
文章结构分析:文章是典型驳论结构。
文章首先解释事实:1919 年,英国经历了最大的工时缩短,每周工时降低为 48 小时。
D 关于该事实的观点:是英国经济低迷的主要原因。However 句,G&O 反驳说,一战才是更主要原
因。S 论证说,D 忽视了工时效率的提高。作者直接反驳 D:多数工业国家当时都有工时缩短,破坏
任何国家间工业生产力相对降低的解释。
1. 答案: E
题型:加强题
解析:因为每个人的产出效率提高了。一道围绕数量关系展开的逻辑题,要看清出文章里给的逻辑。
2. 答案: C
题型:推理题
解析:1920s 就是对应 1914—1918,定位句为 However, Greasly and Oxley found that the First World
War(1914-1918) constituted a more powerful negative macroeconomic shock to Britain’s competitiveness.

passage 120
文章结构分析:首句提出待解释现象:小行星的融化岩石的高温。传统上,小行星被认为比行星散热
快,不应该有这样高的温度。
R 的解释是:小行星从相互间撞击获得热量。
批评者说,单次撞击提供能量有限,小行星会在撞击间散热。
作者为 R 辩护:最近发现小行星是多孔状的。固体相撞产生很多碎片散失热量。多孔天体产生更少
碎片,产生更多热量,碎片掉落,又造成隔热层。
1.答案: B
题型:细节题
解析:However, these objections assumed that asteroids are dense, solid bodies.
2.答案: C
题型:细节题
解析:R 认为热量来自碰撞,质疑者认为不正确。还是在讨论 collisions 产生对 asteroids 的温度影响。
Passage121-Passage130 解析
passage 121
文章结构分析:文章首先提出老观点:富兰克林被认为是自学成才的典范。二句引用爱默生的反问说
法:谁能教富兰克林呢?In fact 句开始,作者提出对老观点的反驳。作者指出,富兰克林广泛学习,
很多工作都是集体合作的。作者举出很多说明他得到帮助支持的例子。文章最后,富兰克林作为个体
研究者形象的原因,部分是由于“偷懒”(shorthand)现象:当个人发表群体的发现,历史有时候
把功劳归因于个人。
1.答案: C
题型:高亮句子功能题
解析: history sometimes grants singular credit for collective effort 说明是历史中的普遍现象
2.答案:B
题型:目的题
解析: 定位句在第二句,和第一句是个解释支持的作用。作者的第三句和第一句对立,因此三句的关
系都体现出来了。

passage 122
文章结构分析:首句支持现象:墨西哥湾的死亡带(dead zone)现象已有很长历史。科学家通过研
究沉淀物寻找原因,发现 F 生物可能提供死亡带成因。最早在 1823 年,F 在密西西比河中大量繁殖,
暗示丰富营养会导致低氧死亡带。末句高亮黑体句指出,F 在 1950 年后最多,这也是农民使用营养
丰富肥料的时候。末句,研究者总结观点,认为:是肥料使用的增加导致更严重的死亡带现象。
1. 答案: E
题型:高亮句子功能题
解析:提问举例的目的。例子在前,观点在后,是倒着总结的,因此选 E。
2. 答案: A
题型:推理题
解析:选项 A 如果减少化肥的使用量,dead 会减轻;选项 B 错误继续使用化肥,会加重 deadzone;
选项 C 停止了化肥,会减轻, 但是不会消失,毕竞洪水期也会有。

passage 123
文章结构分析:
文章首先介绍已知老信息:一些海藻通过制造 DMS 气体产生云彩,DMS 气体与上空的氧反应,形
成固体颗粒,从而提供水蒸汽凝固成云的依附物。
L 认为,海藻造云的过程是全球气候控制系统的一部分。他认为,温暖条件增加海藻活动,产生更多
DMS,形成更多云,从而阻挡阳光,起到为地球降温作用。当气候变冷,海藻和 DMS 活动降低,进
入升温循环。
在应对怀疑者关于生物自立活动如何有利于全球气候的时候,L 指出,降温对海藻有利:导致更多营
养循环。云导致的降雨也能带来更多氮营养。
1.答案: C
题型:细节题
解析:海面的营养物质得到补充。the circulating water carries nutrients upward from the depths below.
2.答案: B
题型:类比题
解析:thermostat 是自动调温器,与文章内所介绍的藻类通过 DMS 造云来控制 cooling process 的循环
机制相同。
3.答案: A
题型:目的题
解析:L 的结论是藻类通过 DMS 控制天气,也是地球自我调节过程的一部分。质疑者说藻类当然应
该进化出对自己的生存有益的行为。L 已经证明了,藻类的行为不但有利于地球,也有利于自己,这个
双赢的逻辑正确。

passage 124
文章结构分析:46 亿年前地球刚形成时,太阳燃烧发出的亮度值相当于今天的七成。然而地质记录中
直到 23 亿年前オ首次出现广泛的结冰期的证据。后面是研究地球暖化的内容。
1.答案: B
题型:高亮句子功能题
解析:提供一种假设,作为一个现象的解释。
2.答案: E
题型:推理题
解析:如果 methanogens 比 cyanobateria 出现的晚,作用就不那么重要了。正确,
因为后者会杀死前者,前者的暖化作用就被削弱了。

passage 125
文章结构分析:一些研究者认为鯨目动物——鯨和海豚——拥有文化,研究者把它定义为互相学习的
能力。
1.答案: B
题型:高亮句子功能题
解析:举例说明前一句话中提到的 deduction。
2.答案: E
题型:推理题
解析:最后作者认可,确实 They exhibit tendencies that suggest a capacity for the kind of behavior that
qualifies as cultural.

passage 126
文章结构分析:比太阳更热也更大的恒星,称为"straggler",超蓝巨星,后文提供该恒星的形成原
因。
1.答案: C
题型:目的题
解析:He suggests that a pair of stars already orbiting each other presents a larger target for a third star or
another pair。他认为,一对已经互相吸引彼此环绕的恒星,对于第三颗恒星或另一对互绕的恒星,是
更大的目标。
2.答案: B
题型:推理题
解析:定位句 Hotter and more massive than the Sun,, stars called "stragglers"are puzzling to astronomers
because such rapidly burning stars would not be expected to persist in ancient star clusters 错误选型都是因
为比较对象不符合原文内容。比较对象要看仔细。
3.答案: E
题型:细节题
解析:Peter Leonard theorizes alternatively that in low density globular clusters, where mergers between
single stars occur too infrequently to account for the observed quantity of blue stragglers, these stragglers are
created instead by a group of stars.
Too infrequently to account for the observed quantity of blue stragglers 因此 PL 对前句的观点质疑。

passage 127
文章结构分析:
文章首先介绍老观点:在大地震之后,地质断层会在几百到几千年的时间内保持安静。
Recent evidence 句,新证据否定老观点。地质断层对临近断层导致的微小压力有出乎意料的反应。地
震中释放的压力会沿地质断层移动,在附近聚集。微小的压力增加会导致另一该断层或附近断层又一
次地震。
该新假说已经解释了好几个破坏性地震的位置和频率。
1. 答案:选 E
题型:细节题
解析:Most seismologists assume that 提出观点之后这句话就被驳斥了,Recent evidence of subtle
interactions between earthquakes may overturn this assumption, however.
多数地震学家提出观点,然而最近发现的一些,地震之间存在细微互动的证据,可能推
翻这种假设。
2.答案:AC
题型:推理题
解析:根据第一句,他们同意作者攻击的那种观点,即认为断层经过一次地震后,要安静一段时间,
等积蕾了足够力量才会再次爆发。B 选项因为是作者支持的假设,因此不选。
passage 128
文章结构分析:动植物的种类少,貌似需要人类努力适应。句 3 列举了两拨学者的观点,都是为了支
撑句 2 的推论。然而从句 4 开始,作者话锋一转,用句 4-7 反驳了这种观点。
1.答案:C
题型:目的主旨题
解析:根据文章结构可知选 C 。观点被后面的内容质疑 Reduced biodiversity in an island context is
likely to require significant adaptation on the part of colonizing human populations.For human communities,
however, this limitation may potentially be offset by other factors.2.答案:ABC
题型:细节题
解析:根据句 6、7:
A small island such as Malta or Melos allows all communities direct access to the sea, providing an
important nutritional “safety net,” as well as an element of dietary diversity, which may actually give
island communities an advantage over their landlocked counterparts.
Islands may also have specific nonbiological resources(such as obsidian on Melos), which may be used
in exchange with communities on other islands and adjacent mainlands.
3. 答案:A
题型:推理题
解析:Evan 认为生物多样性和岛上有的资源是相关的,从后面反驳可以看出作者认为 Evans 过于强
调,或只强调 terrestrial resources 了,没有看到海洋资源的因素。

passage 129
文章结构分析:文章是关于 MM 成就的评论。
首句是关于 MM 成就的老观点:她被当作文章家而非小说家记住。
二句 But 句转折后,作者提出自己对 MM 的评论观点。作者认为,支持 MM 最好文章的,还是她的
虚构天才。她是用生动的描述、吸引人的形象、微妙的特征描写等表达观点。作者认为,MM 最重
要的贡献是,淡化了各种写作文体之间的区别,淡化了小说和文章的区别。
1. 答案:ABC
题型:细节题
解析:
选项 A 对应原文“what powers McCarthy's best essays are her fictional rather than strict intellectual
gifts”;
选项 B 对应原文“McCarthy's greatest contribution was to blur the distinctions between different kinds
of prose writing”;
选项 C 对应原文“And for all her exacting sense of fact”。
2. 答案:D
题型:词汇题
解析:
Discursive(using logic reasoning but not intuition)东拉西扯的,推论的,证明的;
prolific 多产的
sophistic 诡辩的,似是而非的
rambling 闲逛的,蔓延的,不连贯的,(文章等)长且离题的,东拉西扯的
analytical 分析的
circuitous 迂回的,曲折的
纵观全文,没有任何证据说明 M 的杂文罗嗦,主题不鲜明,东拉西扯,所以文中的 discursive 只
可能是 analytical 分析的,推论的,跟文中 polemical 辩论的,近义。

passage 130
文章结构分析:该文是典型的驳论文。
首句介绍一些考古学家的观点:美国可能在 4 万到 2.5 万年前被殖民。
However 句之后,作者提出反驳。
理由 1: 需要解释该时段人类居住点证据缺失的问题。澳洲比美国小,但有很多支持其在 4 万年前被
殖民的证据。按说澳大利亚保存证据的条件更差。
理由 2: 澳大利亚丢失了同等比例的陆地(反驳陆地损失导致证据缺失)
理由 3: 美国连接内陆的河流丰富,人类栖息地局限于海岸的说法不合理。
1.答案:A
题型:目的主旨题
解析:第一句的观点被负面点评,后面北美缺失能证明存在定居人类的证据,暗示这个观点是被作者
驳斥了。
2.答案:E
题型:推理题
解析:句 5 明确指出 because of postglacial sea-level rise。
选项 A,根据常识可以否定,面积变小,形状大致不变的情况下,周长应该变短而不是变长。
选项 B,未知信息。文段没有讨论美国境内河流的变化。
选项 C,用澳大利亚 25000 年前的人口密度比美国现在的人口密度,毫无意义,虽然前者确实肯定小
于后者,但这不是作者想在文中 imply 的内容。
选项 D,未知信息。而且对作者的论证也没任何帮助。
3.答案:D
题型:推理题
解析:作者提到缺少证据指明北美在距今 40000-25000 年前有人居住,暗示他认为在这段时间美国
没人居住。
Passage131-Passage140 解析

passage 131
文章结构分析:文章首先提出待解释现象,热量摄入比正常水平低 1/3 的动物通常活得更长,也通常
更健康。
第一黑体高亮句提出老观点解释:新陈代谢减慢导致相关毒素生产减少。
In fact 句,作者对高亮句解释提出质疑,酵母,虫子(第二黑体高亮句)都是反例。
Some scientist 句是应对质疑的新解释观点:热量缺乏导致更复杂的防御反应,该防御反应在哺乳动
物中引起细胞防御修复的变化。
1.答案:D
题型:高亮句子功能题
解析:对应第二个划线句子,前文说客观事实是少吃更健康,科学家们早期解读是因为新陈代谢慢,
产生的有毒产物增殖也慢。后来发现少吃也不并不减慢代谢,科学家对限制食量使动物更健康的机制
的看法改变了。划线句子本身是个案例,对此进行支持。
2.答案:A
题型:推理题
解析:本文中的 empirical findings 指句 4 提到的,实验观察到,新陈代谢不因进食减少而改变;
presumption of the explanation 指句 3 提到的,进食受限新陈代谢变慢有毒物质累积变慢,变慢是一种
假设,没有被证实过。标记部分暗示二者矛盾。

passage 132
文章结构分析::文章整篇是对相反观点的对比介绍。文章开头是 B&H 关于 deindustrialization 表明
的观点:1960 年代后期美国制造业的外流导致"hollowing out"效果,国内大量工厂关闭造成工人进
入不稳定的低收入工作,或者直接失业。二句是批评者相反的观点:新的服务和高技术部门最近创造
了大量工作。三句( however 句)指出的是反对二句的证据: hourglass effect(收入的两极分化,技
术精英只为前产业工人带来低收入工作机
会)
1. 答案:B
题型:目的主旨题
解析:目的题。根据文章结构可知,B 是最准确概括:针对一个经济现象
(deindustrialization)呈现不同观点。
A 选项, contributed 没有依据。
C 选项, the short-term and long- term effects 没有依据。
D 选项, a dominant theory 文章没有体现。作者介绍相反观点,没有说哪种更 dominant。
E 选项, new evidence 文章不明显。
2.答案:D
题型:削弱题
解析:题目要求找最能削弱 B&H 的 deindustrialization 的选项。由首句可知,B&H 核心观点是:坏影
响,空洞化,导致工人低收入失业。直接说产业工人高收入的选项应该是最直接的反驳削弱。D 选项,
大量产业工人经过训练成为高技能,高收入的计算机工人。
A 选项, since the mid-1960,时间点明显不对。
3.答案:A
题型:推理题
解析:定位"critics""hourglass"关键词到章倒数一、二句,概括倒数一、二句可得 A 选项。

passage 133
文章结构分析:文章整体是比较典型的驳论文。文章开头是一些历史学家对"widely held belief 的质
疑:这些历史学家认为 19 世纪末到 20 世纪中期的教育进步没有美国南部黑人获得权力。之后是这些
历史学家用来支持自己观点的理由证据。到 Nonetheless 句,文章作者直接跳出来反对这些历史学家:
作者认为教育进步对黑人的经济政治权力有贡献。之后是作者的理由。
1.答案:C
题型:目的题
解析:根据引用内容定位到文章第 2 句冒号后。C 选项 unevenness 是对冒号之前 never steady 的同义
改写。
2.答案:E
题型:高亮句子功能题
解析:文章高亮句出现在文章中间,之前是一些历史学家的观点及理由。之后是作者质疑反对一些历
史学家的观点理由。E 选项"qualifes"理解为"限定”。
3.答案:C
题型:推理题
解析:根据关键词"Civil Rights movement"定位到文章末句。根据末句及整体文章结构(转折后说
教育进步的正面贡献)可得 C 选项。

passage 134
文章结构分析:文章是比较典型的驳论。开头是老观点:对资源的竞争决定物种在特定生态系统中的
位置。 However 转折后,作者用红松鼠和灰松鼠的例子,来说明资源
竞争之外的因素。红松鼠在灰松鼠引入之前就有周期性族群数量循环。灰松鼠携带
疾病,都是作者指出的竞争外因素。
1.答案:D
题型:目的题
解析:题目问作者提到灰松鼠觅食效率的目的。定位到倒数第 2 句,根据 Although
可知是作者的让步。
2.答案:C
题型:推理题
解析:题目问作者对于早期观点的评价。根据文章结构可知应偏向负评价:关于物
种在特定生态系统中的位置,作者认为其忽视了资源竞争之外的其他因素。C 最准确。

passage 135
文章结构分析:文章是关于 1970 年代旧金山 Chicano 族群壁画艺术的评论。首段主要是将其与之前
壁画的对比:早期是政府主导的,1970 年代是人民自发的。早期是艺术家个人的,1970 年代是艺术
家和社区群众一起的。二段把 Chicano 族群壁画艺术放到一个大的历史背景下: Chicano 族群壁画艺
术是 Chicano 人民民权运动的手段,壁画艺术促进了民权运动。
1.答案:E
题型:目的主旨题
解析:题目问文章的主要目的,通过文章结构分析可知 E 最准确:二段的当代政治背景可理解为历史
背景,首段与早期比較也可理解为历史背景。
2.答案:E
题型:细节题
解析:通过关键词定位到最后一句。该句"They"指代前句"Artists",E 是该句概括改写。
3.答案:B
题型:结构功能题
解析:题目问两个段落之间的关系。通过文章结构分析可知 B。

passage 136
文章结构分析::文章是 Phelps 作品的评论。首句暗含作者的观点是:Phelps1882 年的小说 Doctor Zay
之前的作品也应该被关注到,后文可以看作是支持首句暗含观点的理由:这些作品促进了内战后女性
医生地位的提升。
1.答案:B
题型:推理题
解析:题目问作者会同意的关于 Phelps 作品的观点。
选项 A,错在后半句,关于小说 Doctor Zay 作用文章没有提到。
选项 B,是概括文章可得的观点。句 3 有 largely overlooked 可以对应选项中的 underestimate。
选项 C,比较从文章得不到。
2.答案:ABC
题型:细节题
解析:题目问文章中有依据的陈述。
A 选。公众的态度是不接受女医生,见句 2 in the face of pernicious resistance 。
B 选。P 早期作品如何促进了美国女医生的利益,句 2 主干(These writings underscored the
achievements)就回答了 how 的问题。
C 选。Doctor Zay 有名,文学批评界总是提到,早期作品不出名,句 1 可知 little-known。
passage 137
文章结构分析:文章开头提出观点:鸟的羽毛妨碍我们对鸟生理机能的理解。后文通过和哺乳动物的
对比说明观点。哺乳动物肌肉运动直观,和人的很像,好理解。乌有羽毛遮挡,我们看不到内在运动。
1.答案:ABC
题型:细节题
解析:题目问关于鸟的羽毛的效果。
A 依据首句。
B 依据首句。
C 依据首句。
2.答案:E
题型:推理题
解析:依据关键词定位到倒数第 3 句“With a mammal...”依据本句和前句,结合与鸟的对比,可知 E
选项。

passage 138
文章结构分析:文章是对于 Gerstmann 在 1920 年代发现的评论。Gerstmann 发现:大脑特定区域的损
伤可能与手指控制能力及数学理解能力有关。作者二句 but 转折后倾向支
持 Gerstmann 的发现。作者引用 2005 年的实验支持 Gerstmann。在该实验中,实验群
体大脑特定区域被磁场刺激干扰,他们手指控制能力和数字配对能力(Facility)的受到
损害
1.答案:C
题型:高亮句子功能题
解析:高亮部分出现在二句 but 之后,由文章结构和句义理解可知 C 最准确。
2.答案:D
题型:目的题
解析:由文章结构分析,可知 D 最准确体现作者目的。

passage 139
文章结构分析:文章是典型的观点论证结构。首句是作者观点:分析美国印第安人在政府岗位的比例
重要。之后是层次清楚的几个理由:
First,改府岗位比例反映人口结构重要,能维护传统上弱势的印第安群体的政策利益。 Second,政府
岗位还能提供经济利益。
Third,了解比例很重要。
文章最后是作者引用 Peterson and Duncan 的观点。
1.答案:D
题型:目的主旨题
解析:D 最准确反映文章结构,首句的 Analyzing...可理解为对特定社会研究的倡导。E 不准确。文章
的重点不是倡导某种社会政策,而是对社会现象的研究。
2.答案:C
题型:高亮句子功能题
解析:高亮句在文章最后,由文章结构可知是 Peterson and Duncan 指出的某种社会趋势可能反映的意
义。C 相对最准确,projection 可理解为推测,推测社会趋势的意义。
A 文章最后一句和前句是并列关系,不是解释关系。
B 文章最后一句是观点,不是 evidence。
D discrepancy 文章没有体现。
E 参见 A 选项对最后两句关系的分析。
3.答案:C
题型:推理题
解析: 由文章 Second 句体现的经济好处可知 C 选项, 在 First 句提到的 policy interests 之外
( apart from)
还有其他好处。

passage 140
文章结构分析:文章首句指出现象: attine ants 利用 fungus 的现象。第二句提出对该现象的常规理解;
蚂蚁控制菌类。第三句 But 后作者提出质疑,菌类也可能处于主导地位。
1.答案:A
题型:细节题
解析:题目问文章指出的支持蚂蚁控制菌类的证据。A 选项依据第二句: Because the ants play the
behaviorally active role...
2.答案:E
题型:词汇题
解析: manipulate 的意思是控制,这个地方表达的意思是形成一些复杂的机理去控制一些行为,这个
控制就是去支配,去战胜,所以 E 选项最为合适。
Passage141-Passage150 解析
passage 141
文章结构分析:文章首段陈述一个问题:理论家和大型计算机气候模型都不能预测云系统对气候变暖
的作用。不同的研究之间结论不ー致。第二段进一步通过一个包括十四个模型的研究细述问题,并没
有给出解决方案。
1.答案:B
题型:目的主旨题
解析:文章首句可以看作 statement,后文都是细述、支持该陈述。A 选项 theory 不准确,theory 通常
有一个明确观点,C 文章没有 new information。D 文章无预测。E 文
章有对比,但是对研究结果的对比,不是对观点的对比。
2.答案:A
题型:推理题
解析:根据题干的 fourteen models 可以定位文章第二段的第一句。题目问,fourteen
models failed to agree,即与这些模型不一致的点。在第一句中,我们能看到,clouds
represented the weakest element in climate models 这个现象已经被这些模型解释了。所
以,C 选项说,这些模型基于 effects of clouds 的不同假设,明显就与原文违背了。
3.答案:A
题型:推理题
解析:A 选项依据首段:some studies suggested that...On the other hand...矛盾的关键是考虑的云的种类
理解不同。要想取得一致预测,首先就要取得对云的种类考虑的一致。

passage 142
文章结构分析:文章介绍以 Herbert Shaw 为代表的理论家对当代地质理论的质疑。文章开头指出该质
疑:地球内部能量是否足够推动板块运动。之后文章以 Herbert Sha 为质疑派代表,认为要考虑天外
能量影响。作者对比 Herbert Shaw 和正统地质理论:大部分地质学家认为天外因素是分开独立的,
Herbert Shaw 认为天外因素不可分割。作者进而指出现代板块理论的背景基础:Newton
1.答案:D
题型:目的主旨题
解析:D 最准确, explanation 可理解为 Herbert Shaw 对板块运动原因的解释。 Theoretical context 可
理解为十九世纪的古典地质学,牛顿的思想。
A 选项偏向后半段,无法概括 Herbert Shaw 的观点。
B 选项核心概念 extraterrestrial impactors 偏。
C 选项 dubious evidence 文章无依据。
E 选项偏向前半段,无法概括之后的讨论。
2.答案:E
题型:目的题
解析:根据关键词定位。可知上下文强调 Herbert Shaw 的观点和传统观点的对比。

3.答案:C
题型:推理题
解析:C 选项依据文章后半段。现代板块理论,像古典地质学一样,借用牛顿的前提假设。

passage 143
文章结构分析:文章开头指出问题:人的因素在 Holocenel 期的影响难于处理,因为气候和人为因素
难于分开。黑体句是作者对前文的否定,作者人为人的影响小,主要还是自然气候因素。
1.答案:E
题型:高亮句子功能题
解析:根据文章结构分析,可知 E 最准确。作者说人的影响小,等于降低之前讨论问题的复杂性。
2.答案:B
题型:细节题
解析:B 选项依据文章中段提到的例子。

passage 144
文章结构分析:文章介绍 Meggers 的观点:土壤环境决定史前农业社会的规模,亚马逊地区因为土壞
筼瘠不可能支持大规模族群。即使加上鱼和乌龟资源,河边村落也不可能达到上万的人口。Omagua
的观念支持 Meggers 的观点。
1.答案:A
题型:高亮句子共同题
解析:前句是 M 对于亚马逊地区的论断,本句是关于农业与环境的一般性论断,为前句提供一般性
理论支持。
2.答案:D
题型:推理题
解析:D 选项定位 Meggers accepted that fish and turtle resources of the Amazon made possible the long
riverside villages...Omagula 属于 riverside villages.

passage 145
文章结构分析:文章是关于 blenny 选巢倾向的研究报告。文章开始介绍选巢的两种因素,石头的大小
和巢的方向。巢的东南朝向比石头大小更能解释卵的多少。水流速度不是选择巢的朝向的原因,因此
朝向本身更是择巢原因,尽管背后的原因不清楚。
1.答案:B
题型:目的主旨题
解析:由文章结构分析,两种因素中,朝向更是择巢的重要因素,可知 B 选项。
C 选项 nest size 概念不准确,即使理解为 stone size,文章说,卵的多少不能被巢所石
头 completely explained,并没有完全否认 stone size 的因素。
2.答案:C
题型:目的题
解析:在文章所述最近的研究中,朝东南的巢和其他朝向的巢的水流是一样的,可知水流不是朝向的
关键因素。

passage 146
文章结构分析:文章是作者对于 editors 传记起源于浪漫主义时期观点的驳论。 Editors 的观点是:传
记起源于浪漫主义时期。但作者指出起源的过程是缺失的。作者进而指出,传记同样可以被认为是起
源于前浪漫主义的 scandalous memoirs.
1.答案:A
题型:目的题
解析:由文章结构分析可知,作者指出 scandalous memoirs 是表明传记起源的替代性来源。
2.答案:C
题型:细节题
解析:由关键词"attained great significance"定位到末句"became an object of major ideological
significance",C 选项“widespread”是“sheer ubiquity”的同义改写。末句“it”指代前句“The Romantic
period”。

passage 147
文章结构分析:文章关于 S 回巢现象的解释分析。文章首先指出关于回巢的一个现象:距离越长,回
巢成功率越低。提出两种可能解释:
1) 能力问题,距离太远,没有能力回。作者在第一个 However 句否定了该解释。
2) 是否值得问题,能量消耗和卵被捕食与回巢成功孵化之间权衡利弊得失。作者在第二个 However
句指出,有大窝卵的 S 在长距离回巢率更高(支持了解释 2,大窝卵更值得消耗能量回巢)。
1.答案:B
题型:目的主旨题
解析:B 是对文章结构的最准切概括。D 选项只概括了后半段。
2.答案:D
题型:假设题
解析:高亮句在第一个 However 转折,题目问暗含的假设。作者在该句指出,在每个距离上都有 S
个体成功回巢。作者用此证据否定第一种解释:太远没有能力回巢。D 选项取非,成功回巢的个体比
其他个体有更强的回巢能力,等于说其他个体没有回巢能力,就不能否认第一种解释:没有能力回巢。
3.答案:A
题型:推理题
解析:题目问第二种解释对于远距离没有回巢的 S 暗含的观点。第二种解释认为是”是否值得的问
题",引用有大窝卵的 S 回巢率更高的证据作为支持,认为有大窝卵更值得回来,暗含的逆否命题观
点就是:不值得回来的是因为有小窝卵。A 选项由此推理可得。

passage 148
文章结构分析:文章是对于作者 Elizabeth Gaskell 的作品 Mary Barto 的介绍加评论。
首段引入 Mary Barton:关于 1840 年代英国产业工人的遭遇。作品关于工人阶级的日常生活,带有
documentary 性质。
首段末句是对 documentary 性质的大正小负的概括评价。
第二段开头指出作品一个不足:局外观察纪录者的视角。但之后转为正评价:genuine
imaginative,convincing. 第二段末句又是一个大正小负的概括评价。
第三段开头介绍作品一个章节( Old Alice's History)所表现的从乡村转变为都市工人的
痛苦经历。
段末提到工人合作的传统。
1.答案:B
题型:推理题
解析:题目问哪个选项能体现从末段例子中概括出的工业化经历。B 选项依据原文末段首句: workers
brought from the villages and the countryside to the urban industrial centers.末段提到的几个人物的例子
也都体现了这一特点。
2.答案:E
题型:推理题
解析:本题是典型正改善题。首先找到文章作者对 Mary Barton 的负评价:第二段开头:As a member
of the middle class, Gaskell could hardly help approaching working-class life as an outside observer and a
reporter。取非后可得 E 选项。
3.答案:C
题型:态度题
解析:由关键词 documentary record 定位到首段。首段末句是文章作者大正小负的评价。C 选项,“有
保留的赞许”体现最准确。
4.答案:D
题型:类比题
解析:类比题。由人名 Job Leigh 定位到第三段。由段落两处提到的 Job Leigh 可概括出与 D 选项类似
的特点:人在工业都市中,迷恋于与身处环境不协调的乡村事物。

passage 149
文章结构分析:首段开头指出问题:机动车快速增加,很多美国城市污染物超标。随后,作
者指出相对可行解决方案:天然气等替代性燃料。
第二段首先指出替代性燃料的优点:更小更简单的分子,更少碳碳组合键,释放的碳氢物质更少生成
臭氧。
作者通过对比,指出传统燃料的缺点:更大分子,更易不充分燃烧,更易产生未燃烧的光化学性能活
跃的碳氢化合物。 On the other hand 之后,作者指出替代性燃料的缺点。
第三段特意讨论乙醇(Ethanol)和甲醇(Methanol)。相对其他替代燃料,乙醇甲醇有
优点:单位能效更高,需要较少改进现有燃料供给系统。但也有缺点:乙醇比甲醇更
贵。甲醇另外的优点:大量减少导致臭氧的释放。
第四段是作者对替代性燃料的辩护。针对替代性燃料能效低的批评,作者认为,有效的设计能弥补缺
点。
1.答案:B
题型:细节题
解析:题目问的是汽油的缺点,优先定位到第二段。根据关键词"incomplete
combustion"定位可得 B 选项。involves an intricate series of reactions 是原文 involves a more complex
series of reactions 的同义改写。
2.答案:C
题型:类比题
解析:概括首句描述的问题可得:单位排放减少,但总量增长太快,导致总排放增加。C 选项最体现
该矛盾。
3.答案:E
题型:推理题
解析:根据关鍵词"designed to use methanol"优先定位到第四段。第四段先承认使用 methanol 能效
低。转折之后说,为 methanol 设计的车辆使用更少燃料,减少都市空气污染,可知是能效提高了。E
是说能效提高了。
4.答案:A
题型:推理题
解析:题干没有可定位信息,只能用选项定位。A 选项可依据首段:There is a
growing realization that the only effective way to achieve further reductions in vehicle emissions...只能采用
替代性燃料,说明传统燃料没有提高空间了。

passage 150
文章结构分析:文章首句引入主题,关于 Alice Walker's The Color Purples 的评论:评论界的负评价,
希望成为现实主义,但各种非现实主义的因素违反现实主义传统。
第二段首句带有总结递进性质:The Color Purple 是否本质上就不想成为现实主义。 Gates 认为, The
Color Purple 中与上帝书信交流的设计来自黑人文学传统。
第三段是文章作者对 Gates 评论观点的批评。作者认为 Gates 对 The Color Purple 的评论
不准确:忽路 The Color Purple 的传统结构,过去关注书信本身,忽略书信的环境。作
者认为:需要 evaluative paradigm 来表明 The Color Purple 的故意的反传统意图。
1. 答案:B
题型:推理题
解析:题干所说的细节出现在第一段。作者在第二段就提出包括这些细节在内的因素说明 The Color
Purple 是否本质上就不想成为现实主义的问题。在第三段作者的评论中,作者进一步明确 The Color
Purple 的反传统性质。B 选项的 certain novelistic conventions 可理解为 Anglo-European 的现实主义小
说传统。
2. 答案:C
题型:目的题
解析:在第二段,在对 Gates 的评论介绍中,可知美国黑人文学可能是 The Color
Purple 的模仿对象。
3. 答案:B
题型:细节题
解析:由关键词 an evaluative paradigm 定位到第三段最后。由最后一句可知 B 选项。
4. 答案:E
题型:推理题
解析:题目问 Gates 和首段提到的评论者的共同点。首段评论者注意到 The Color
Purple 的非现实主义性质,第二段 Gates 认为它借鉴了美国黑人文学的因素,可知 E 选项正确。
Passage151-Passage160 解析
passage 151
文章结构分析:首段开头是 Oliver 的观点:布鲁斯难于转化成文字的诗歌,However 转折后,作者认
为,Hughes 成功地把布鲁斯转化为诗歌。
第二段首先介绍 Hughes 认为重要的体裁区分:folk blues 和 classic blues。传统评论家
认为, Hughes 成功转化的是 classic blues,他转化 folk blues 不成功。
第三段是作者的评论。作者认为 Hughes 的布鲁斯诗歌表现了 folk blues 的精髓。作者借
用 Arnold Rampersad 的观点:Hughes 的布鲁斯诗歌表达的是普通民众的声音。
Young Gal's Blues 是这方面的代表,Golden Brown Blues 则是机械模仿古典布鲁斯的反面代表。
1.答案:E
题型:目的主旨题
解析:根据文章结构分析,作者在第三段反驳了第二段介绍的传统评论家的观点。
2.答案:B
题型:目的题
解析:根据首段结构,结合句义理解,可知高亮句起到强调写作布鲁斯诗歌的作用。
3.答案:A
题型:推理题
解析:根据引号内容,进行定位,然后能够知道:Young Gal’s Blue 更加倾向的是 common folk; 而
Golden Brown Blues 则是 remote from the common,所以,可知,
Young 的更加 colloquial。此题的难点在于选项中各个生词的认知。neglect= lacking
4.答家:B
题型:细节题
解析:由首段 Hughes 解决的问题,第二段体现的 classic blues 的特点,可概括出相同点。

passage 152
文章结构分析:首段通过对比引出问题现象;和 19 世纪的其他几个革命相比,我们缺乏对 1848 年 2
月革命社会背景、动机、过程的了解。
第二段第三段,作者对首段问题给出两种解释:1)被 6 月革命所淡化。2)成功革命缺乏可靠历史记
录。
1.答案:D
题型:推理题
解析:D 选项可定位到第二段第 2 点解释。该解释强调,和失败革命相比,成功革命缺乏历史记录,
可知革命之间的历史纪录不同。
2. 答案:B
题型:推理题
解析:作者在末段的结论是,像 1830 年 7 月和 1848 年 2 月这样的成功起义难于描画,难于理解社会
根源。B 选项里讲,1830 的 background and motivation have been identified,因此为正确答案。
3.答案:D
题型:目的题
解析:由文章结构,可知第二段解释的是首段问题现象。
4.答案:E
题型:推理题
解析:作者首段提到 participant's background,第二段又提到 socioeconomic groups,综合可知,作者
认为社会经济背景对于革命的理解重要。

passage 153
文章结构分析:文章首段先介绍 19 世纪对 17 世纪荷兰绘画的传统评论观点:现实主义视角。However
转折后引入最近的研究:象征主义视角。超现实主义的解读几成共识,但对超现实、象征程度的理解
仍是难题。
第二段借 de Jongh 表达观点:评估多重性很困难。作者反对 de Jongh 的跟随者对作品的
纯粹象征主义的处理。
第三段首先借用 B 的观点:绘画会有单一象征意义之外的各种可能含义。作者反对对 17 世纪荷兰绘
画的公式化简单化处理。
1.答案:D
题型:目的主旨题
解析:根据文章结构,作者主要是在对最近的关于荷兰 17 世纪绘画的研究做介绍和评

2.答案:D
题型:目的题
解析:定位到第二段末。根据所在句可知 D 选项。
3.答案:B
题型:功能题
解析:根据文章结构,作者在末段(第 3 段),仍然在对最近的研究的简单象征主义倾向做批评。caveat:
警告,批评。
4.答案:E
题型:推理题
解析:根据首段开头,19 世纪艺术批评家对 17 世纪荷兰绘画做现实主义理解。首段后半部分,emblem
books 被作为含有不能简单做表面理解(现实主义)的素材的来源。可知 E 选项。

Passage 154
文章结构分析:文章开头是老观点,18 世纪后期文化权威的共识:历史阅读比文学阅读地位更高。之
后,包括黑体句和 likewise 句都是从作家塑造女主人公形象的角度支持老观点的例子:喜欢历史的往
往是正面主人公,喜欢文学的往往是负面形象。Yet 句是文章作者的评论观点:该问题比表面看起来
的更复杂。之后是文章作者用来支持自己立场的理由:不喜欢历史的有正面意义,喜欢历史也有负面
作用。
1.答案:C
题型:目的主旨题
解析:根据文章结构,尤其是 Yet 转折句可知 C 选项。
2.答案:B
题型:细节题
解析:定位到该句:While Hume's history was generally seen as being less
objectionable than his philosophy, there were widespread doubts about his
moral soundness even as a historian by the time that Hanway was writing. 从该句可知,Hume 的哲学被负
评价,他的历史也被负评价。
3.答案:D
题型:高亮句子功能题
解析:根据文章结构分析,可知高亮句是支持文章首句的例子。首句和该句都体现的对历史阅读的判
断是:历史阅读地位更高。
4.答案:C
题型:目的题
解析:根据关键词可定位到该句:While Hume’s history was generally seen as being less objectionable than
his philosophy, there were widespread doubts about his moral soundness even as a historian by the time that
Hanway was writing。由前文 Yet 转折前可知, Hanway 把阅读 Hume 历史当作正面形象塑造的手段。
但该句中,Hume 的历史有负面评价。综合可知, Hanway 没想到阅读 Hume 的负面影响,否则不会
当作正面塑造的手段。

passage 155
文章结构分析:Columnist 的论证中,P 和 V 两种同样功能的处方药品。
P 的广告用更多的处方数量当作 P 更优秀(第一句黑体句,P 广告的观点)的证据。It can be argued
之后是 Columnist 的评论。
第二句黑体句是 Columnist 的观点:P 广告的观点很荒谬,理由是:P 上市的时间比 V 更长(是时间
长导致处方数量多,而不是质量更高)。
答案:B
题型:高亮句子功能题
解析:由结构分析可知,第一句黑体句是 P 广告的观点,也是 Columnist 反对的观。第二句黑体句是
columnist 的核心观点。

Passage 156
文章结构分析:本文在一开始就说到:有证据证明 Garden of Eden 这幅作品是 van Eyck 创作的。但是
接下来,出现了第一个高亮句说:尽管如此,这幅画也有可能是别人创作的。而后后面开始举例说明,
为什么这幅画很可能是别人的作品。
答案:C
题型:高亮句子功能题
解析:逻辑单题中出现句子作用题,最常见的考法就如本题所示,问你哪一句是结论,
哪一句是支持结论亦或是反对结论。
由文字结构分析可知,第一个高亮句是 main conclusion;而第二个高亮则是 provide evidence in support
of that conclusion。答案选 C。

Passage 157
文章结构分析:本文介绍了两种方法来研究 relatedness of species,其中,传统的 anatomy 的方法已经
被 ascertained,而新的方法 infer(未被证实=is not borne out)...
答案:A
题型:逻辑单题推理题
解析:本题要求我们根据原文提供的信息,选择一个最合适的结论。

passage 158
文章结构分析:
背景信息:Z 鱼被引入英国河流,破坏本土鱼类。
目的:保护当地鱼类。
手段:政府项目清除大量 Z 鱼。
矛盾结果:今年本土鱼类被 Z 鱼吃掉比去年还多。
答案:B
题型:逻辑单题解释题
解析:请注意,这道逻题的突破口在于 greater effect。特别是文中最后一句话提到:当把一大部分 zander
清除之后,剩下的 zander 对本地的鱼造成的影响比之前更大。
B 选项符合逻辑。清除的鱼,都是 zander 的成年鱼,而成年 zander 会吃年轻的 zander,这就使得之前
的影响不会那么大。现在,成年的 zander 去掉了,年经的 zander 就没有了天敌,吃得可不就比之前
欢快了嘛。

passage 159
文章结构分析:Langland 和 Chaucer 的作品在结构和主题上都有很多相似之处。一些批评家认为,在
Middle English poetry 中,有相似成份的现象并不常见,所以,他们认为 Chaucer 受到了 Langland 的
Pier Plowman 的影响。
答案:D
题型:逻辑单题削弱题
解析:
题目要求我们选一个削弱的选项,那该选项能够明 Chaucer 不ー定是受到了 Piers Plowman 的影响,
有可能是受到了其他作品的影响。所以选 D。

passage 160
文章结构分析:原文给出 argument:Sphinx 的 face 不可能是 pharaoh Khafre 的,因为....
答案:E
题型:逻辑单题削弱题
解析:
题目要求我们选一个削弱,即是选一个能够支持 Sphinx 的 face 有可能是 pharaoh Khafre 的。所以,排
除下大雨的因素(因为文章已经说了这是不可能导致 face 是 pharaoh Khafre 的原因) , 答案选 E。
Passage161-Passage170 解析
passage 161
文章结构分析:题目要求选一个能够解释:做过手术的病入得 glaucoma-related blindness 的概率低(与
没做过手术的人相比,)
答案:D
题型:逻辑单题解释题
解析:
文中已明确说明:滴眼药水和做手术是 as effective as 的,那为啥滴眼药水的病人会有
更高的概率得 glaucoma-related blindness(也即是做过手术的病人得 glaucoma-related blindness 的概
率低),因为滴眼药水的人坚持不下去,所以效果就不如做手术好了。

passage 162
文章结构分析:timberline 是植物能够生长的最高的海拔。然后通过植物的化石表明,在比当前的 Rocky
mountain timberline 还高 100 米。所以,推导出以前 Rocky Mountain 的气温没有现在低,所以才会生
长出植物。
答案:C
题型:逻辑单题假设题
解析:考虑主动攻击论证。原文推理有一个漏洞,万一这个植物是特别耐寒,耐冻的植物呢?那显然,
你再说以前的气温没有现在低,就是不符合逻辑的,所以,我们需要假设的是 C。

passage 163
文章结构分析:cotton grass 是 caribou 夏天的唯一蛋白质来源。如果 caribou 在夏天不能获得足够多的
蛋白质的话,那么在第二年,它们就不能繁殖后代。而最近,随着平均气温的上升,cotton grass 开始
消失了,所以,文章得到一个结论,if the warming trend continues, caribou 将要灭绝。
答案:C
题型:逻辑单题假设题
解析:原文逻辑结构就有一个漏洞了,如果气温的上升,使得出现一些新的植物,而这些植物能够为
caribou 提供足够的蛋白质,那它们仍然能够正常的繁殖。所以,要补充文章的逻辑,选 C。

passage 164
文章结构分析:尽管 Almaria-年级学生中,阅读水平比较高的学生的比例在 1995-2010 年期间没有
变化,但是同时期的学生中,阅读存在问题的学生的比例显著增加。表明一年级的学生的平均阅读水
平是在下降的。
答案:D
题型:逻辑单题加强题
解析:原文有一个很重要的逻辑,并没有说明,那就是 schoolbooks 的难度,到底有没有
变化,如果 schoolbooks 的难度越来越大,那就不能说明,一年级学生的阅读能力越来
越差。所以,我们需要补足这个逻辑,选 D。

passage 165
文章结构分析:Minoans 这个地方有很发达的制铜的工业(因为在这个地方发现了很多 bronze
implements and the furnaces 等制造铜的东西)。然后,又在 southern Greece 发现了很多类型于 Minoans
这个地方生产的铜制品,所以,得出一个结论,Minoans 生产的铜制品不光自己本国使用,还出口到
southern Greece。
答案:B
题型:逻辑单题加强题
解析:原文逻辑最大的漏洞就在于:本文并没有说 southern Greece 能不能生产,或者其它第三方国家
A,能不能生产。所以,原文的逻辑是不严谨的。所以,需要补足这个逻辑,选 B。

passage 166
文章结构分析:埃及的木乃伊保存普遍比其他地方的木乃伊更好。有两个原因:一是技术,二是埃及
当地干燥的气候。
答案:D
题型:逻辑单题加强题
解析:
这道题重要的突破点在于,第二个原因中的插入语,“more likely”。所以,更有可能
的是第二个原因,而现在需要我们 provide the most support,那我们的选项,就应该
体现出是干燥的问题,而不是技术的问题。
选 D。(埃及古代的坟墓中保存的尸体和木乃伊中保存的一样好,说明这并不是技术的
原因,而有可能是第二个气候干燥的原因。)

passage 167
文章结构分析:Colonia 的经济在过去的八年中,进入了衰退期。很多本该倒闭的公司都幸免于难,
因为他们员工的高效率帮助公司控制了成本。
然而,在最近的几个月,Colonia 的经济开始复苏,公司也开始招人。因此,Colonia 将会遭受到雇员
平均工作效率的下降,因为_____。
答案:C
题型:逻辑单题填空题
解析:原来雇员的效率是很高的,现在由于雇员入数的增加,导致工作效率下降,那只能是新加入的
员工效率很低或者是没有原来老员工的效率高,所以才能把平均水平往下拉。所
以,选 C。

passage 168
文章结构分析:
答案:E
题型:逻辑单题填空题
解析:最近出土的古代 Carthagian 硬币都印有人的头像,同时在脸颊的地方会有 mole。由于,艺术家
不会在 generalized portrayal of face 上包含这种特殊的细节,
所以,考古学家认为,这些硬币应该是某个真实的个人,而不是一个 idealized type。
然后,这个结论是不合理的,因为_____。
我们现在需要说明的是,这个结论不合理,也就是需要我们说明,这些硬币上的图案就
是 idealized type,那既然是 idealized type,为什么这上面。又会有 mole 呢?因为 E。

passage 169
文章结构分析:人们普遍认为最早 controlled use of fire 的是 200,000 年前的 Homo sapiens。然而,
最近发现了一个预计 500,000 年前的遗址,包括:charcoal,burned animal bone, and charred rocks。
尽管,homo sapiens 在那个时段还未出现,遗址上的这些发现,也不能说明 controlled use of fire 的时
期早于 Homo sapiens,因为_____。
答案:E
题型:逻辑单题填空题
解析:
这道题,最大的逻辑漏洞就在于,我们没有办法确定 500,000 年前,遗址上发现这个
火,是不是 controlled use of fire。所以正好选 E。

passage 170
文章结构分析:因为有比较强的 stylistic similarities,El Castillo 和 Altamira 的洞穴壁画,被认为来自
同一个时期。然而,最近的颜料分析表明 Altamira paintings 是迄今为止最老的作品。尽管如此,这个
案列也不能证明,stylistic similarities 不是一个靠谱的指标。因为_____。
答案:B
题型:逻辑单题填空题
解析:因为,stylistic similarities 还是可以用来进行 closeness 的分类的,所以选 B。
Passage171-Passage180 解析
passage 171
文章结构分析:San 这个地方的人相比于典型的工业社会的人,他们吃更多的水果、蔬菜和低盐的食
物。同时,他们中得高血压和肥胖症的概率要远远低于工业社会的人。然而,如果工业社会的人也遵
循相同的 diet,他们获得高血压和肥胖症的几率并不会显著性地降低,因为_____。
答案:D
题型:逻辑单题填空题
解析:两个地方的人都采取了相同的 diet,但是产生的结果还是截然不同,只
能说明是个体差异了(体格,基因)。选 D。

passage 172
文章结构分析:文章关于 Mionan 建筑讨论中 Palace 术语使用的不同观点。
D 观点:对于 Palace 的术语使用并没有一致意见,争论经常集中于建筑的功能。
E 观点:Palace 可用来表达不同的角色:皇家居住地,管理中心等等,时间久了,导致该术语所指
称功能的争论。
S 观点:Palace 有社会政治等级身份含义,她建议使用更中性的“宫廷中心建筑”。
作者观点(The problem 句):问题不是来自应用的建筑标签,而是来自术语使用中所暗含的假设。
作者介绍说,自己使用该术语,不是暗示功能,而是指向特定正式的建筑元素。
1.答案:倒数第二句。
题型:功能句子选择题
解析:这个题注意问的是“author”,也就是文章里面的"I",只有最后两句才是说的作者的结论,
然后注意倒数第二句直接得出结论,最后一句是用例子加强结论,所
以作者对 particular approach taken by Minoan archaeologists 的反对主要是用倒数第二句来体现。
2.答案:D
题型:推理题
解析:Schoepp 的观点是紧接在 hose functions has been called into questions 之后的,Schoep 认为 Palace
是用来描述有地位的人的而不是功能,作者在文章里面也是反对 function 的,所以 D 选项中的 without
being committed to the term’s functional implications 是非常符合作者和 Schoep 的观点的。A 选项没有提
到 function,错。B 选项和 A 选项同理,错。C 选项虽然提到了 function,但是和作者的观点是相反的。
E 选项 hierarchical social structure 是 Schoep 的观点,作者并没有反对 Schoep,所以 E 也是错误的。
3.答案:E
题型:推理题
解析:作者的观点是最后两句话,所以此题要从最后两句话里面去寻找答案,可以看到 E 选项和文章
中 refer to a group of buildings that share a set of formal elements 是同义改写,所以 E 选项正确。A 选项
convention 文章没有提到,A 错。B 选项中 replaced by a more neutral term 是错误的,因为作者是比较
认可 Schoep 的 more neutral term,但是作者认为自己的想法才是对的,所以不存在 replaced。C 选项
明显错,文章就是在说以前对于 Palace 的用法是错误的。D 选项也是明显错的,这个问题用文章原话
is widely used and deeply ingrained,所以肯定不是 recently become popular 的。

passage 173
文章结构分析:首段介绍矛盾现象:D 与 ST 两地鸟类种类重合,但 D 有更高的比例是陆地觅食(61%
vs 37%)
二段提出尝试解释:D 栖息地更开阔的地貌。However 转折后研究者质疑该解释:ST 也有开阔地貌。
三段提出另外几种假设解释。
1 未探明的地貌特征,例如树的高度,树冠生态复杂度。
2 地表食物源的不同。
3 历史上的他因影响,例如,放牧、生火的规定,外来捕食者的影响,林木采伐等等。
段落末句支持了他因说,D 受历史他因的影响更少,更短,高比例反映了相对自然完整的情况。
1.答案:B
题型:组织结构题
解析:第二段第一句先是提出了ー个观点,从 howerver 开始这个观点被反驳
了,thus 继续解释反驳内容。所以符合这个结构的明显是 B 选项。
2.答案:C【划线句子是 Dryandra has not been free of these changes,but the impact may have been less
or more recent with the result that Dryandra may retain a more natural or complete bird diversity relative to
the Tablelands.】
题型:高亮句子功能题
解析:这句话是和上一句中描述的现象有关系,做了一个进一步说明的作用,所以答案选 C 选项比较
合适。
3.答案:D
题型:推理题
解析:原文中 not revealed by casual observation 说明 canopy complexity 不是随便被观察到的,对应选
项中的 D 选项 are not always obvious。A 选项完全是无中生有。B 选项也是无中生有,文章并没有提
到这个关系。C 选项依然是无中生有。E 选项也是文章没有提到的内容。
4.答案:E
题型:推理题
解析:文章最后一句说到 Dryandra may retain a more natural or complete bird diversity relative to the
Tablelands, 其中关键词 natural,注意 introduced foxes and feral cats 不是 natural 的,所以它们对于 D
的影响比 T 更小,所以 E 选项是正确的。

passage 174
文章结构分析:
背景信息:因为天花消失,美国儿童 1970 年代早期停止天花疫苗接种。最近科学家警告天花感染的
危险。
美国各个年龄群体有类似的感染几率,因为(填入空格)。
答案:D
题型:逻辑单题填空题
解析:what many people believe 指的是前面的现象:小孩需要接种天花疫苗。
文章最后一句的观点是所有年龄段的人都需要接种天花疫苗。所以必须要找出表明
现在的成人身上已经没有天花抗体的选项。所以答案选 D 选项,因为如果天花疫苗
的效果在 20 年之后会消失的话,那就意味着现在的成年人身上没有天花的抗体了,
所以需要接种新的天花疫苗。

passage175
文章结构分析:文章介绍马达加斯加牛饲养者引入仙人掌种植对生活的影响。
首句背景信息:牛饲养者引入仙人掌种植。
二句说引入仙人掌的效果:不再迁徙,依靠仙人掌储水。
三句说该变化对生活的影响,尤其是迁徙性。
四句,五句,六句分别介绍对女性和男性工作性质的影响。
七句总结,仙人掌种植导致迁徙减少。
1.答案:Women harvest cactus for their families to eat.(第四句)
题型:功能句子选择题
解析:很明显只有这句话说到了 prickly pear 对于人类饮食的影响。注意 Male herders work cactus as a
vegetable crop for cattle.这句话说的是对牛的影响,不符合题意。
2.答案:C
题型:细节题
解析:A 选项说得过于绝对了,文章只说现在不需要通过游牧的方式来帮助 cattle 度过旱季,但不是
完全不需要帮助 cattle 了,其实还是需要把仙人掌转化成液体补充给 cattle。B 选项也说得太绝对了,
文章并没有说这些人完全放弃游牧生活,而且题目里面还用到了 pastoralists 这个词。C 选项是对的,
是文章最后一句话的同义改写。

passage 176
文章结构分析:文章关于 CS 文学观点的介绍加评论。
CS 文学观点:用文学作品对读者对实际影响而非定义经典的审美标准评估文学作品。
CS 用 LMA 的“Little Women”作为被传统标准忽略,用自己的标准发现有价值的例子。
EB 批评 CS 的文学标准,认为其标准过于主观模糊。
1.答案:D
题型:细节题
解析:定位到文章"because she believes such works have been unjustifiably
neglected by unsympathetic scholars"这句话,所以直接从选项中找到这句话
的同义改写即可。所以答案是 D 选项,unrecognized=neglected
2.答案:C
题型:目的题
解析: Louisa May Alcott's Little Women 是在第一段中被提到的,而文章的第一
段是在呈现 Stimpson 的观点,所以引出 Louisa May Alcott's Little Women 的目
的一定也是要表达 Stimpson 的某个观点的,所以先排除 AB 选项,因为文章并没有
体现作者的观点。C 选项符合文章结构。D 选项明显错误的,因为 Stimpson 和
Barnes 的观点是不一致的。E 选项中的 ambiguous 也是第二段才提到的,所以不可
能是引出 Louisa May Alcot’s Little Women 的目的。
3.答案:D
题型:推理题
解析:这个题定位到文章的最后一句话,定位到关键词"ambiguity",这个词说
明 Stimpson 在定义 goodness 和 love 的时候并不清晰,包括前文 Stimpson never
actually defines‘love’,也能说明这一点,所以 D 选项的描述是符合文章的。

passage 177
文章结构分析:首句关于特定现象:海洋动物的震荡性游泳。
后句提出解释观点:返回水面保持体温。Tuna 等都是支持该观点解释的例子。
1.答案:AB
题型:细节题
解析: A 选项是 improving their muscular efficiency 的同义改写,B 选项是 swim with bursts of speed
的同义改写,C 选项并不是 maintaining an internal body temperature above that of surrounding waters 的
作用,排除。
2.答案:A
题型:词汇题
解析:这个题关键词其实是 oscillatory,这个词体现的是摆动,并没有带攻击的意思,就是普通的游
上游下而已,所以答案选 A 最合适。

passage 178
文章结构分析:
首段对比新老观点。
二十世纪老观点:民族概念早于民族国家。
新观点:先有民族国家,再有民族概念。
二段介绍以 Em 为代表的朝鲜学者对朝鲜案例的研究。他们的研究支持新观点:民族国家早于民族观
念。
三段是作者对朝鲜案例的观点评论。作者概括区分两个阵营:
前现代主义者,认为民族概念出现更早。
现代主义者,认为现代民族国家促成民族概念。
作者各打五十大板,认为两者都有问题:
七世纪分裂的朝鲜难以有整体民族意识。
朝鲜的历史长于西欧学者的研究对象,可能有共同身份意识。
1.答案:A
题型:目的主旨题
解析:这篇文章的最后一段是对文章的总结,所以最后一段的第一句可以看出这篇
文章的主要目的,是在评估 primordialists 和 modernists 两种说法各自的问题,所以答案选 A 选项。
2.答案:第三段第三句
题型:功能句子选择题
解析:younger Koreanists 的观点中有 common sense of identity,第三句话中的 universalistic orientations
能够支持。
3.答案:B
题型:推理题
解析:从文章第一段能够得知这两人对于国家的理解中 a strictly modern phenomenon 这一点,然后文
章最后一段中"On the other hand,the inhabitants of the Korean peninsula had a much longer history-well
over one thousand years-as a unified political collectivity than did the peoples studied by Gellner and
Anderson"这句话明显是在说这俩人的调查不严谨,所以答案选 B 选项最合适。
4.答案:C
题型:细节题
解析:文章提到欧洲,拉丁美洲,东南亚是在文章第一段说 Gellner 和 Anderson 的观点的时候,那个
地方说这些地区和韩国一样,都是“modern phenomenon”“common sense of identity”,所以不同的地
方必须要出现在反驳那俩人的观点的地方,所以定位到"On the other hand,the inhabitants of the
Korean peninsula had a much longer history -well over one thousand years-as a unified political collectivity
than did the peoples studied by Gellner and Anderson."提取关键信息"much longer history as a unified
political collectivity",所以答案选 C 选项。

passage 179
文章结构分析:
背景信息:鸟吃 CC 辣椒种子,提供传播方式。
加勒比海岛种植 CC 辣椒,岛上 CC 辣椒应该是由人类引入的,因为(填入空格)。
答案:A
题型:逻辑单题填空题
解析:要证明 Domesticated varieties of C.chinense 是由人类引进岛屿,也就是
要证明不是由鸟类引进的,所以 A 选项符合逻辑。如果鸟类无法吞住 Domesticated
varieties of C.chinense 的话,那么只能是人类才能把这个品种带到岛屿上去。

Passage 180
文章结构分析:首句总体观点:农业社会的数据表明,陶瓷的商业生产是用来农业产出的不足。H 对
于美国西南部的调查发现,B 居住点有多于家庭需要的整村的陶瓷生产材料(为首句观点提供支持)。
1. 答案:D
题型:句子功能题
解析:文章的最后一句话表明 WB 这个地方的陶瓷比正常家庭使用要多,所以能推出这里的陶瓷有一
些是用来进行交易的,所以也就是证明了和文章首句的正面支持关系。所以答案选 D 选项,WB 这个
地方并不是消费陶瓷的地方,而是生产陶瓷由于交易的地方(证据支持首句观点,反过来,首句观点
也支持后句研究发现)。这个题 C 选项有干扰性,注意 WB 这个地方并不是 agricultural surpluses.而
是 pottery surpluses。
2. 答案:D
题型:词汇题
解析: 有文章结构, marginality 所在句支持前句观点,
marginality 同义重复前句 s inadequate , insufficient.
可知 D 选项最准确。
Passage181-Passage190 解析
Passage 181
文章结构分析:首句:一些学者拒绝把 K 当作独立体裁。
However 句,作者提出异议,K 有独特的声音特征,能区分于其他体裁。K 有犹太音乐共有的元素。
1.答案:B
题型:推理题
解析:根据文章的第一句就可以推出答案,文章第一句说尽管 Klezmer 来自于很多
不同的音乐传统,但是 Roma(Gypsy),Greek,and Romanian 却变成主导了,所
以 Klezmer 也一定受到了 Roma( Gypsy),Greek,and Romanian 的影响,不然的
话这里的让步的意思就有问题。所以这个题答案选 B。A 选项无法从文章推出,C 选
项通过第一句可以推知和文章描述是相反的,D 选项和 E 选项都无法从文章中推出。
2.答案:C
题型:推理题
解析:A 选项文章没有提到过,所以不可能是正确答案。B 选项也不对,is enough
to preclude 这完全是无中生有,文章里面从来没有这么肯定的语气。C 选项是对的,就是取文章最后
一句话 These and other elements typical of Klezmer are also found in other forms of Jewish musical
expression, including cantorial music 的同义改写。

passage182
文章结构分析:
背景信息:灰尘阻挡阳光。
事实信息:冰川期发现大气中有更多灰尘。
观点:灰尘促进延长了冰川期。
答案:A
题型:逻辑单题削弱题
解析:题目的推理是空气的灰尘多阻挡阳光直射,然后导致气温下降,然后延长冰
河时期。所以削弱选项必须要体现灰尘并不会导致气温的下降。
A 选项正确,因为如果灰尘能够防止地球上的热量逃离的话,那就说明灰尘能够帮助地球增加温度,
所以对题目的推理起到了削弱作用。
B 选项错误,题目研究的就是 ice age 时候的灰尘的影响,选项说的是两次 ice age 之间的时候,起不
到削弱作用。
C 选项也错误,火山爆发是除了灰尘之外的其他因素。
D 选项错误,D 选项和 B 选项犯了同样的错误,研究时期的不同对题目起不到削弱作用。E 选项错误,
E 选项其实加强了文章的结论,因为阳光是主要热源,所以灰尘就是会导致温度下降了。

passage183
文章结构分析:
段首是气候学家 1964 年的新观察及观点:应该是墨西哥暖流导致 1910 和 1920 年代的北大西洋表面
增温,北大西洋增温与 1940 年全球破记录的暖温一致。
However 句,当时的温室气体不可能与人类相关变暖有关。
末句总结,温室效应反对者认为,最近的变暖也可能是自然原因,而非人类原因引起。
1.答案:C
题型:推理题
解析:contrarians 认为任何最近的变暖都可能是自然的而不是人为的,所以我们在
ABC 选项中找等价的信息就行了。A 选项错误,因为并不是在讨论人为还是自然这
个问题。B 选项错误,错误原因和 A 选项完全一致。C 选项正确,C 选项这句话其实
就是 contrarians 后面的 claim 的同义改写。
2.答案:C
题型:推理题
解析:A 选项错误,原文并没有说 1910s 和 1920s 的气候变化和 1940s 的气候变化有直接的关系。B
选项也错误,1990s 其实也没有确定 1940s 的变暖原因。C 选项正确,原文原句 However,no one believes
enough greenhouse gases had reached the atmosphere by then to cause much of a human-induced warming
就说明我们无法确定是否这次变暖是不是由人类产生的温室气体引起的。

passage184
文章结构分析:
首句二句引入文学权利应用到集体合作材料到问题,作者权利和文学继承人的确定出现疑问。
后文举例说明问题及问题产生的原因:把基于欧洲的个人权利概念用到基于口头传统的文化上。
1. 答案:A
题型:目的功能题
解析:注意文章举了两个例子,一个是 oral tale-个是 chemist,两句话用 just as
连接,表示两个例子的功能相似。然后注意后面一句话"This instance on private
rather than collective ownership, derived from the nineteenth-century notion of the autonomous, creative,
authorial voice, flies in the face of those who come from an oral tradition"这句话中的 this instance 就是上
面的两个例子,所以这两个例子的功能就是把 collective ownership 变成了 private,所以正确答案选 A
选项。
2. 答案:D
题型:内容主旨题
解析:这个题非常简单,整篇文章都是在说到底 authorship 属于个人还是属于集体,所以描述这种关
系的选项是 D 选项。
3. 答案:C
题型:细节题
解析:此题对应到文章这句话"This instance on private rather than collective ownership, derived from the
nineteenth-century notion of the autonomous, creative, authorial voice. flies in the face of those who come
from an oral tradition",所以在选项中和此句话同义改写的是 C 选项。

passage185
文章结构分析:
事实理由:在 S 地方,就单个事故导致的受伤人数,出租车总是比其他车辆多。出租车乘客倾向于不
用安全带。
结论:要求出租车乘客用安全带之后,就单个事故导致的受伤人数,出租车就会不比其他车辆多。
答案:D
题型:逻辑单题削弱题
解析:文章的推论是乘客坐出租车系好安全带能够使得车祸中受伤人数不高于其他车的车祸受伤人
数,所以我们要找的选项必须是使得出租车车祸受伤人数多于其他车的。
A 选项错误,连出租车都没有提到。
B 选项错,发生事故多和平均受伤人数是完全两个概念。
C 选项也错,错误原因和 B 选项类似,车辆多和平均受伤人数没有关系。
D 选项正确,如果出租车平均乘客比其他车多,这就很可能导致平均受伤人数多。
E 选项也错,我们需要的结论是出租车平均受伤人数多,E 选项也不能实现这个目的。

passage 186
文章结构分析:文章的大致意思是:葡萄在温室的高温和高二氧化碳的环境中长得好,vineyard 不容
易提供温室高温环境,但是可以提供高二氧化碳的环境,所以葡萄的产量也会上升。
答案:A
题型:逻辑单题削弱题
解析:削弱的选项必须是尽管在高二氧化碳中,但是葡萄的产量可能不会上升,甚至可能会下降。
A 选项是正确答案,A 选项说在温室生长的时候,容易控制害虫,现在没有了温室,所以害虫可能导
致葡萄产量的下降。
B 选项没有提到葡萄产量的下降,所以错误。
C 选项偷换概念,葡萄的产量和葡萄的质量是不一样的概念。
D 选项和 C 选项错误原因很接近,都是偷换概念,没有提高葡萄产量这个要素。
E 选项属于偏加强的作用,温度上升之后有利于葡萄的产量上升。

passage 187
文章结构分析:
首段,作者提出对 NN 摄影成就的评估方法:把 NN 的方法应用到其他摄影作品,能否产生其他媒介
不能实现的效果。尽管对比的手法挑战观众对于大小的传统概念,油画也能实现类似效果。
二段,作者提出应用 NN 第二标准(带有创造者的痕迹)、第三标准(持续观看后仍能吸引观众兴趣)
检测。作者认为,是观众自己判断是否 NN 的作品引发后续观看的新体验(否定了 NN 的第三标准)。
1. 答案:C
题型:细节题
解析:在第二段 indeed 句是作者的让步:Newhall's work bears her unmistakable imprint in the
juxtaposition of the photos and in the rhythmic prose accompanying them,可得 C 选项。Newhall's
work bears her unmistakable imprint=contains elements identifying it as a particular person’s work
2.答案:E
题型:细节题
解析:定位到"Does this photographic essay achieve results impossible in other media?",这句话说明
Newhall 只相信在摄影中存在。所以答案选 E 选项。

passage 188
文章结构分析:
背景信息:很多种类的蜘蛛会变色为所在花朵的颜色。
理由:蜘蛛捕食者能识别该伪装。
结论:蜘蛛的变色行为不会带来重要的生存好处。
答案:C
题型:逻辑单题评价题
解析:此题的逻辑推理是蜘蛛的捕食者能够去区分蜘蛛变色,所以蜘蛛的变色对于蜘蛛的生存不太可
能有什么好处。题目要让我们选一个能够评估这个推理的选项。
A 选项错误,因为 A 选项没有提到蜘蛛的变色对于生存的影响。
B 选项错,蜘蛛的自然颜色也不影响它的生存。
C 选项是正确答案,因为 C 选项提到了蜘蛛的变色和生存之间的关系,如果蜘蛛变色能够让蜘蛛捕食
猎物有帮助的话,那就符合问题的要求。
D 选项也错,文章捕食在研究蜘蛛变色和花的关系。
E 选项错,文章没有研究其他动物的变色。

passage189
文章结构分析:
多数女权主义劳工历史学家的观点:1930 年代的工业化工会对于组织大量妇女参加工会正面作用巨
大。他们认为,工业化工会致力于招收非技能工人,放弃种族和性别的封闭性吸引了女性工人。1910
年制衣业女工的成功组织不是否定,而是预示了工业化工会的形式。
答案:C
题型:细节题
解析:此题相对比较简单,定位到文章最后一句话,从 since 开始直接解释了原因“the garment unions
welcomed women workers of every skill level”, 所以 C 选项是正确答案。

passage190
文章结构分析:本文主要是改正了之前的一个错误观点:美国黑人的 newspapers 对于美国的民权运动
的影响小。
1.答案:E
题型:目的主旨题
解析:从文章的分析来看,实际上美国黑人的 newspapers,对于美国民权运动有见解的推动作用,所
以答案选 E 选项。
2.答案:D
题型:细节题
解析:此题对应到文章的这句话"using a compelling form of advocacy journalism rather than the standard
objective newspaper style, allowed those who in the 1950s began pushing nationally for equality to start at a
far higher level”。直接从选项中找同义改写,所以对应到 D 选项。
Passage191-Passage200 解析
passage191
文章结构分析:这篇文章主要是在说人类的一些习惯导致了我们一直会和蚊子共存,即便我们不喜欢
和蚊子生活在一起。
1.答案:C
题型:内容主旨题
解析:最准确的答案是 C 选项:arguing that social traditions can perpetuate people’s problems with
mosquitoes.
2.答案:C
题型:组织结构题
解析:这篇文章先是提出了一个言论:人类不能消灭蚊子。然后列举了一些理由告诉我们为什么人类
不能消灭蚊子。最后再一次强调之前的言论,所以正确的文章结构应该是 C 选项。
3.答案:第二段第一句
题型:句子功能选择题
解析:题目问这篇文章可能对作者的立场的一个误解。文章只有第二段第一句作者在澄清自己的立场,
也就是提出了读者可能对他的误解。

passage192
文章结构分析:用了 BPA 的老鼠的后代比没有用 BPA 的老鼠的后代要大 10%,所以推出 BPA 对于
老鼠的体型有影响。
答案:B
题型:逻辑单题假设题
解析:假设是 B 选项,即两组老鼠在做实验之前体型没有区別,如果在做实验前老鼠体型本来就有差
异的话,那么就削弱了这个实验的结果了。取否之后对推论有削弱,符合文章的假设。

passage 193
文章结构分析:
1995 年,狼在 70 年后被重新引入黄石公园。
没有狼的时期,以杨树为食的鹿群不仅对杨树群,而且对其他以来杨树的动物群体造成灾难。
狼重新引入后,鹿群行为恢复正常,保护小树和生态环境。
1.答案:B
题型:细节题
解析: 定位到文章"During the wolf-free era,heavy browsing of aspen trees by elk populations spelled
doom not only for trees themselves but for a host of other creatures dependent on them, such as beavers...",
其中最后的 such as beavers 就说明 elk populations 对于 beavers 有着非常大的负面影响。所以符合这样
的意思的是 B 选项。
2.答案:B
题型:推理题
解析:定位文章“When wolves returned,grazers and browsers resumed normal patterns of behaviors,
preferring safer, open areas over the dense cover and streamsides where carnivores can lurk. Keeping elk
wary”。意思是狼回归之后,elk 等动物就更喜欢安全开放的区域,不太喜欢 streamsides,因为那里有
肉食动物。所以符合这个描述的是 B 选项。

passage 194
文章结构分析:
老观点以为,动物纤维天然营养均衡,肉食动物不用特地平衡饮食。
But 句化学分析新发现,非脊椎动物物种间和物种内被捕食者的营养不均衡。
G 新解释观点:捕食者根据营养状况选择食物。
J 等人的实验支持 G 观点。
1.答案:D
题型:推理题
解析:A 选项错误,文章只描述了 sit-and-wait invertebrate predators,没有描述其他的 predators,所以
A 选项属于推测。
B 选项错误,文章并没有说为什么在营养组成上要有区别。
C 项错误,dry mass 只是 wolf spider 那里举的例子,不代表所有。
D 选项是正确的,文最后一句话就是在描述这个内容,和 D 选项是等价的。
E 选项也是文章没有说到的,只能推测。
2.答案:C
题型:组织结构题
解析:文章先是提出了一个传统的观点,然后后面反驳了这个观点,最后再用了些证据去支持反驳的
理由。符合这个文章结构的是 C 选项。

passage 195
文章结构分析:
首段介绍背景信息:D 猴子被豹和黑猩猩捕食,对他们有不同的应对策略。
对豹,D 猴子大声吼叫;对黑猩猩,D 猴子不叫或小声叫。
二段说明 D 猴子应对策略的复杂性。黑猩猩也被豹捕食,被捕食时发出响亮叫声。
猜测假设:D 猴子应该能区分应对两种情况:只有黑猩猩的情况,和黑猩猩报警豹出现的情况。
三段介绍 Z 的实验。Z 的对比实验说明 D 猴子有不同的应对反应。
四段介绍 Z 的另一组实验。该实验也说明 D 猴子有基于因果关系的反应策略。
1. 答案:A
题型:细节题
解析:定位到文章“in contrast,upon detecting a chimpanzee,male Diana monkeys do not vocalize at all,
while females give only a few quiet alarm calls and flee quickly to hide in the forest canopy”,意思是雄性
猴子不会发出声音,雌性猴子会发出很小的声音,所以答案选 A 选项。
2.答案:E
题型:目的题
解析:定位到文章“function both to warn others and to signal to the predator that it has been detected”,
所以答案选 E 选项。

passage 196
文章结构分析:
首句是作者对于 M 的艺术的总体判断:难于分析。
二句引用 HS 的说法,支持首句对 M 的判断。
对比 P 和 B, P 和 B 相对更容易分析。
文末总结,M 的方法太个性化,直觉是他仅有的体系。
1.答案:A
题型:推理题
解析:此题定位到“But the solutions that Matisse arrives at are always idiosyncratic and tend to be unrelated
to any system of ideas.”因为 Matisse 和 Braque’s cubism 是相反特征,所以可以推出 Braque’s cubism
的特征就是 system of ideas,所以正确答案是 A 选项。

2.答案:AB
题型:细节题
解析:题目定位到这句话“But the solutions that Matisse arrives at are always idiosyncratic and tend to be
unrelated to any system of ideas.”通过 idiosyncratic 推出 Matisse 的 unconventional 的特征,所以 A 选
项是对的。unrelated to an system of ideas 则说明 B 选项是对的。

passage 197
文章结构分析:
首句引入困难问题:定义狩猎采集群体的困难。
后文解释困难的原因。
根据普通定义,狩猎采集者是依据狩猎野生动物,采集野生植物生存的人。但是(Yet 句),人作为
消费者的存在,模糊了野生和驯养的界限,狩猎和放牧的界限,采集和种植的界限。Moreover 句指出,
在消费方式上采用放牧和种植的群体如何归类是不清楚的。
1.答案:B
题型:内容主旨题
解析:文章第一句话"Attempts to identify New Guinea's hunter-gatherers face the well-known difficulty
of defining what constitutes a hunter-gatherer group“就表明了这边文章的主要目的,就是定义 New
Guinea’s hunter-gatherers 有困难,所以答案选 B 选项。
2.答案:E
题型:词汇题
解析:这里 common 就是指“普通的”,所以 E 选项 conventional 最接近这个意思。

passage 198
文章结构分析:文章说作者关于女性写作研究问题的判断分析。
仅仅从 1960 年代末开始,文学学者才开始准确系统地建立女性小说家的历史。之前的历史学家都受
限制于“伟大传统主义”,局限于杰出作家的研究,忽视女性小说家的多样性。
问题:1960 年代前期的文学学者完全回避妇女的性别身份问题,仅仅关注于妇女写作的形式风格,
忽略了妇女写作与她们法律经济身份变革的关系。
1.答案:C
题型:细节题
解析:此题需要定位到文章这一句话“it was not surprising that some literary scholars in the early 1960s
evaded the important issue of women’s sexual identity entirely”,所以这些学者做的事情就是回避了女性
的性别身份的问题,所以在选项中最贴近的是 C 选项。
2.答案:C
题型:细节题
解析:定位到文章这一句话“These histories excluded the minor novelists, who are the links in the chain
that binds literary generations together”句中的 these histories 指的就是 Great Traditionalism 那一段时期
的历史。这段历史主要就是忽略了和其他小说家之间的关系,所以答案选 C 选项最合理。
3.答案:C
题型:态度题
解析:定位到“Such an approach, while insightful and very valuable, did not
consider the crucial connections between women's writing and changes in their legal and economic status.”
其中 such an approach 指的就是 some literary scholars in the early 1960s 的立场,所以比较等价的评价是
C 选项:respectful but critical。就是先承认它好的地方,然后再说不好的地方。

passage 199
文章结构分析:文章的推理是说像木星这样大小的行星形成的时候,它的轨道一定比地球绕太阳的轨
道要长。
答案:E
题型:逻辑单题加强题
解析:题目问哪个选项能够提供支持,尽管发现了一些例外的情况。
A 选项是无关选项。
B 选项也不能支持推理,排除。
C 选项说的是地球,题目需要说的是像木星一样大小的行星。
D 选项没有提到行星轨道,也排除。
E 选项是正确答案,因为如果行星老了之后就会离母恒星更近,那就能够支持行星刚产生的时候轨道
比较大的这个说法。

passage 200
文章结构分析:
首句提出待解释问题:引起 B 工蜂个头大小差异的原因是什么?
二句三句提出解释:成年蜂决定幼虫大小。
四句给出详细解释:幼虫蜂巢不是像蜜蜂的一样整齐规则排列,同一窝中,边缘的幼虫受到更少关照,
导致十分之一的个头质量变化产生。
末句总结:个头差异有适应功能,蜂群整体获益。
1.答案:D
题型:推理题
解析:文章定位到这句话"the colonies benefit from rearing workers of a range of sizes”,很明显答案对
应的是 D 选项,benefit from=advantageous。
2.答案:C
题型:词汇题
解析: 文章说成年蜜蜂 determine 幼蜂的体型, 再结合前面一句话“In pollen storing species larvae develop
in individual nests cells and are fed directly by adults.”能够推出这里是说成年蜜蜂能够控制幼虫的题型,
所以答案选 C 选项。determine=control。
Passage201-Passage210 解析
passage 201
文章结构分析:
首句事实信息:PB 建筑,有 4-5 层,600 房间。
Traditional 句老观点:PB 是民居建筑。
But 句,W 挑战老观点:缺乏炉子考古证据,首层有 59 炉子发现,上层只有 3 个。
However 句,作者解释炉子发现少的原因,为老观点辩护:上层坍塌,掩埋炉子,早期考古方法没有
揭示炉子。
1. 答案:E
题型:推理题
解析:定位到“But Windes recently challenged this view by pointing out the paucity of...”,可知 E 选项
正确。
A 选项错,irrelevant to the examination 无中生有;
B、C 选项错,crucial Evidence 无中生有;
D 选项错,a false notion about the significance 无中生有;
2. 答案:C
题型:推理题
解析:定位到“which revealed only 3 upper-story hearths, in contrast to 59 ground-floor hearths: habituation
rooms would have required hearths for cooking and heat.”,可知 C 选项正确。
A 选项错, storage areas 无中生有;
B 选项错, remodeled 无中生有;
D 选项错, Early excavation of...disturbed
E 选项错,文中并无此比较关系。
3. 答案:A
题型:推理题
解析:定位到倒数第二句“especially given Judd's disinclination to destroy later structures
and features to expose earlier ones.”可知 A 选项正确。
C 选项错,无中生有;
D 选项错,resulted in 因果关系无中生有;
E 选项错,confirmed traditional population estimates 无中生有。
4. 答案:E
题型:细节题
解析:传统观点定位文章开头。根据二句“traditional interpretations…residential”可知 E 选项正确。

Passage 202
文章结构分析:
首段,1970 年代,长江流域发现早于已知水稻种植 1000 年的高级水稻种植社会。
在此发现之前,老观点认为水稻种植开始于南方。该观点基于野生和自由生长水稻的地理范围,以及
低级水稻种植的考古发现。
水稻南方起源说的支持者指出,长江流域发现的是高级社会,缺乏低级早期阶段。他们坚持认为,最
早的狩猎采集者发展农业一定是在更南方,在当前野生水稻地理范围内。
二段,作者介绍 1984 年的发现,长江流域在野生水稻的地理范围内(用来反对南方起源说,支持长
江流域起源说)。
1.答案:C
题型:削弱题
解析:定位到最后一句 two northern outlier populations were also discovered,而 C 选项说,最近才
migrated to the north,故而 C 选项与原文矛盾,能够 undermine the conclusion。
2.答案:B
题型:细节题
解析:定位到 Proponents of ...point out that... and that evidence for the first stage of rice cultivation is
missing.可知 B 选项正确。
3.答案:D
题型:推理题
解析:根据 This scenario was based on both on the...and on archaeological records of very early domestic
rice from Southeast Asia and India(not known to be not so old as first reported)从括号中的表述知道,这
些 records 是有问题的。D 选项正确。

Passage 203
文章结构分析:
首句老观点:S 地方海牛的消失被归因于高密度捕猎。
But 句质疑老观点,时间短,不太可能杀掉大族群海牛。
However 句之后,作者支持老观点:猎人捕杀海獭,导致海胆扩张,吃 kelp 海藻压力增大。海牛完
全依赖 kelp 海藻。S 曾记录海牛营养不良证据。
1.答案:A
题型:推理题
解析:定位到倒数第二句。 Sea cows were totally dependent on kelp for food,and within a decade of the
onset of otter hunting.可推知,sea cows 被捕杀的就少了,从而 kelp forests 的数量将减少。A 选项正
确。其余选项均在文章找不到对应依据。
2.答案:B
题型:细节题
解析:定位到倒数第三句。 Fewer otters would have allowed sea urchin populations on
which the otters preyed to expand and the urchins grazing pressure on kelp forest to increase
可知,如果 otters 数量减少,那么 urchins 的数量将增加,对 kelp forest 的压力也将增加。
B 选项正确。
A 选项错,sea cow 与 sea urchin 并无该关系。
C、D、E 选项错,无中生有。

Passage 204
文章结构分析:
关于美国革命的老观点:美国创始人强调人的平等权利。
反对观点:有美国创始人有强烈财产欲望,仅仅把平等当作工具。
Both of these stories 句是作者评论。作者认为,两种观点都假设美国创始人是同质化的,都使用同样
的权利平等表述。作者指出,核查当时的公共表述,可发现,当时的领导人不是同质化的,他们总体
上对于自由,财产的强调要远高于平等权利。
1.答案:B
题型:推理题
解析:定位到最后一句,makes it clear that the leadership...;and that overall the relative 由后半句,可知,
revolutionary leader 更关注的是 liberty and property 而不是 equality.
B 选项正确。
A 选项错,与原文相反,was anything but"恰恰不是"homogeneous
C 选项错, provide freedom and wealth for all colonial citizens 无中生有;
D 选项错, was cynically manipulated 无依据;
E 选项错,无法得知。
2.答案:D
题型:细节题
解析:关于第一个 account,原文说到 unsurpassed in its promise of human equality;关于第二个 account,
原文说到, as having wielded promises of equality merely as deceptive tools.D 选项符合。
3.答案:E
题型:削弱题
解析:要想质疑文中高亮部分,我们可以强调,他们同样重视,甚至是更重视 equality.E 选项正确,
是关于很多领导人一致致力于平等权利的证据。

Passage 205
文章结构分析:ZNH1942 年的作品 Dust Tracks on a Road 被批评为太碎片化。
作者为 ZNH 辩解,说批评是基于对传记的传统理解,传统传记的结构和正式组织。而 Dust Tracks on
a Road 的碎片化结构是为其人物个性特征塑造服务的。
1.答案:B
题型:内容主旨题
解析:B 选项的描述最合适,supposed deficiency 就是那种“被信以为真的缺陷",实际上是没有缺
陷的。
2.答案:A
题型:推理题
解析:从 But Dust Tracks portrays a persona that resists reduction to a coherent unity 开始,文章就在描述
这个小说为什么会 fragmentary,然后说很多东西是因为作者必须要描绘一个不同的 persona,所以就
不符合传统的标准了,所以这个题答案选 A 选项,因为文章最后是在为这个 Dust Tracks 的作者做辩
护了。
3.答案:D
题型:组织结构题
解析:文章的结构从前两个题就很容易看出来,就是先提出了传统的观点,然后再反驳了传统的观点,
所以最合适的选项是 D 选项。

Passage 206
文章结构分析:
美国黑人画家 MGJ 成长于都市,但 1934 年描绘了佛杰尼亚 B 城的小城风光。
一些评论者观点:赞扬 MGJ 表现乡村黑人社会风光。
作者评论:评论者默认黑人画家只会记录直接经验。
作者认为,MGJ 在纽约就形成了 B 城乡村画的想法,B 乡村画反映 MGJ 受 C 风格影响,把乡村当作
现代工业都市的理想替代。
1.答案:A
题型:内容主旨题
解析:
A 选项正确。
B 选项错,以偏概全,这是一个细节;
C 选项错,social significance 无中生有;
D 选项错,本文讨论的是 Johnson 的作品,而不是 African American painters;
E 选项错,largely overlooked 无中生有;
2.答案:A
题型:句子功能题
解析:高亮句 tendency to present rural life 是对前一句 Johnson’s conceptualization of South was largely
formed before he left New York 的 expand。B、C 选项文中无依据。

Passage 207
文章结构分析:
首句介绍背景前提:识别个体能力重要,对复杂社会行为演化有重要影响。
研究者认为,实验室核实的鱼的个体识别能力,在自然鱼群中也存在。
In fact 句,作者指出自然环境中的证据不充分。作者认为,实验室证据可能是人工设计结果,自然状
态妨碍个体识别能力发展。
1.答案:第四句:The possibility...
题型:功能句子选择题
解析:根据 speculates about the effect,可以对应到第四句。
2.答案:C
题型:推理题
解析:A 选项错,原文中没有对应依据。文章说到 which enforce prolonged interaction 对应 continued
exposure.接着提到 prevent the diluting effects...可对应 inflated,故而 B 选项正确。文章提到 evidence of
individual recognition in free-ranging fish populations equivocal.可知 C 选项正确。

Passage 208
文章结构分析:
背景信息:Hard Times 是狄更斯发表于 1854 年的最短的小说,是他最不受欢迎的小说。
One critic 猜测:不受欢迎应该归因于编辑剪裁。
However, JJ 指出,该小树发表于狄更斯自己控制的杂志社。
更可能的原因:
1 空洞虚无的价值观。
2 缓慢空洞的结构:前三分之二的描述对话重复啰嗦。
1.答案:E
题型:推断题
解析:利用 last section 进行定位,定位到文章最后一句。最后一句提到,the book' s tone changes abruptly
in the last section,同时,注意到前一句提到,...during the first two thirds of the book are repetitious.由
此,我们可以推断出 E 选项正确。(注意本题是推断题,E 选项会被很多同学误排除,认为文章没有
提到。)
A 选项错,文章并没有就 last section 和 first part of the book 进行 literary quality 的比较;
B 选项错,文章并没有将 last section 和 Dickens 的其他作品就 tedious fashion 进行比较;
C 选项错,文章并没有对 characters 是否 optimistic 进行比较;
D 选项错, last section 是否 atypical of Dickens’ general storytelling style,文章未提及。
2.答案:C
题型:信息目的题
解析:根据题干定位到文章第二句。利用这些作品的成功(这些作品分别是在 Hard Times 之前或之后
发表),目的就是想突出主句中 Hard Times seems to have been Dickens' least popular novel 的观点。
可知,C 选项正确。
A 选项错, helps to explain the public and critical reaction 没有依据。
B 选项错, other works 是否 initially commercially successful 未提及。(当然此处,
也可以理解为 not initially commercially successful 与文中的 sensational success 矛盾)
D 选项错, quality of Dickens’ writing 未提及;
E 选项错,文中并未 question 这些作品的成功。
3.答案:D
题型:EXCEPT 型否定事实信息题。
解析:注意,本题需要选的是哪个选项不是 Hard Times 不受欢迎的原因。
A 选项错,对应 Jacobs also suggest...,此为 Hard Times 不受欢迎的原因。
B+C 选项错,对应 Part of the reason...这两个选项均为 Hard Times 不受欢迎的原因。
D 选项正确,对应最后一句中 When 引导的时间状语从句,可知此时 book' s tone
changes abruptly,可知该选项属于 Hard times 的亮点部分,而非不受欢迎的原因。
E 选项错误,对应文中最后一句 the reader must first...,可知 way that the novel
establishes its character 是 reader 需要忍受的 tedious way, 此为 Hard times 不受欢迎的原因。

Passage 209
文章结构分析:
首句提出问题,资源压力加剧是否为纽芬兰岛上 DPE 和 RI 人对于人口压力的应对?
Not exactly 句,作者否定首句说法,理由是,纽芬兰岛的人口压力不是人口持续增加的结果,而是新
来人口带来的。该岛的狩猎采集人群通过细分化应对另一族群的压力。DPE 人选择了海豹狩猎,RI
人选择了内陆航海来彼此适应。
1.答案:B
题型:事实信息题
解析:正确答案是原文定位句的同义替换。本题利用题干中的 arrival of the Recent Indians 可以定位到
倒数第二句。(当然此处,同学们会在其他地方找到 Recent Indians,但是我们还应注意到题干中的
resulted from,可以与倒数第二句中的 in response to 对应)
(我们在做此题时,只需考虑倒数第二句,这是本题的答案区间)
A 选项错,compete with the Recent Indians for a limited supply of seals 未提及;
B 选项正确, intensified their harvest of seals 与 spent more time harvesting seals 同义替换;
C 选项错,defending their territory 未提及;
D 选项错,adopt new subsistence strategies 原文未提及;
E 选项错,population came to halt 未提及。
2.答案:D
题型:句子作用题
解析:高亮句和前一句是正向的关系,故此,可以排除 E。
A 选项错,law 无中生有;
B 选项错,need for additional research 无中生有;
C 选项错,underscores the importance 无中生有;
D 选项正确,符合高亮句与前文的句间关系。

passage 210
文章结构分析:
首句陈述通常情况:选择多,消费者愿望匹配的概率更大。二句支持首句,更多样化的选择提高愿望
被匹配的几率。
In practice, 作者指出实际情况的不一致:选择增多,实现更好匹配增加的幅度很小。
后文解释理由:选择多未必提高丰富性。选择工具也可能缺失。
Therefore 总结推论:选择多增加愿望不匹配的几率,导致消费者更不满意。
1.答案:C
题型:假设题
解析:阅读最后一句话,要从前半句推出高亮句,存在从 expectations 到 satisfaction 的概念跳跃。作
者是默认两个概念的一致性的,明确两个概念的一致性,就是作者的假设。本题选 C。
A 选项错,原文说 larger assortments can increase the likelihood,表述的是一种可能性,而 A 选项直接
说 depends almost exclusively on the size of the range, 与原文矛盾。
B+E 选项错,本题的逻辑和是否 quickly 无关。
D 选项错,与 the preferences they had beforehand 无关。
2.答案:A
题型:句子作用题
解析:从文章结构分析可知,首句是作者结论(最后一句)得到的前提。
A 选项正确。
B 选项,容易误选,但是请注意题干问的是 as a whole 的作用;
C+D 选项错,第一句是前提作者并没有证明其 illogical 和 dispute;
E 选项错,文章并未解释首句,而是以此为前提展开后续内容的讨论。
Passage211-Passage220 解析
Passage 211
文章结构分析:
文章引入 Founder mutation 概念:来自祖先的引起疾病的基因突变。
特别现象:Founder mutaion 发生概率高于一般基因突变。
解释:Founder mutation 多是“隐性”的,单基因不会发病,只是传递给后代。单基因甚至有优势,
例如,HHM(高亮黑体)提高铁吸收率。
1. 答案:AB
题型:推理题
解析:本题如果利用题干 founder mutations 进行定位的话,将会发现全文都是相关的内容。所以此题,
最佳的做法是利用选项来定位,并验证是否正确。
A 选项正确,根据选项能够定位到 Furthermore,the single copy of a founder mutation often confers a
survival advantage on carriers.
B 选项正确,根据选项能够定位到原文 This apparent anomaly is...becomes ill.
C 选项错误,文中并没有就 founder mutations 和 other mutations 谁更容易 passed to offspring 进行比
较,属于无中生有。
2. 答案:D
题型:信息目的题。
解析:本题属于简单题,问 hereditary hemochromatosis mutation 的作用,该信息是利用 For example
引导出的,作用很明显就是为上一句的观点举例(前一句正好说到 advantage on carriers),故本题选
D。ABCE 选项均可归为无中生有。

Passage 212
文章结构分析:
首段否定一个新发现:最近声称发现的从 P 生成的行星基于错误数据。超新星会毁坏任何老行星,新
P 没有时间生成新新行星。
二段说新证据的另外价值:另外一种 P 可能带行星。MP 可能从附近恒星活得气体物质。剩余物质可
能成为行星。MP 也可能气化伴随恒星,为行星形成提供新物质。
1.答案:C
题型:主旨题
解析:对于本题,大家需要关注到:如果一篇文章有两段及以上,并且这篇文章有主旨题,那么该题
考察的是文章的段落关系。
由第二段首句,But newer evidence of a different pulsar with planets is more promising,可知第一段
是在介绍旧的 evidence,由此,可知本文的主旨 assess the credibility of recent findings,加之,首段提
到了 theorists would have had difficulty accounting for the existence of such a planet, 可知 unverified
existence of a class of objects,对应正确。本题选 C
A 选项错,an example of the dangers 无中生有。
B+E 选项错,文章虽提到一句 theorists would have had difficulty accounting for the existence of such a
plant, 但并未具体提及这种 difficulty,也并没提及 how difficult。
这两个选项容易误选。
D 选项错。Seemingly contradictory phenomenon 无中生有。
2.答案:A
题型:组织结构题
解析:本题与主旨题相似,由第一题的文章结构思路,可知此题选 A。
B 选项错,rejected 无中生有;
C 选项错,a set of observational data 无中生有;
D 选项错,analysis of the same material 无中生有;
E 选项错,an approach to further investigation is recommend 无中生有。
3.答案:D
题型:推断题
解析:定位到首段最后一句。由 it rotates relatively slowly,implying that it has not interacted with any
nearby star since the supernova,推知 D 选项正确。其余选项均无推断依据。

passage 213
文章结构分析:
作者首先介绍背景信息:在翻译 DI 二十年前,HWL 自己认为要学意大利语。接着提出问题:他自
己践行诺言了吗?
However 转折前,作者说 HWL 意大利语表达水平有限,转折后说,他的翻译准确,注意语义细节。
他的翻译表达出杰出的精确、丰富、变化。
1. 答案:D
题型:目的题
解析:作者引用 HWL 表述的目的。结合文章结构,依据后句“How true was Longfellow to his own
dictum?”可知,作者用来当作检查 HWL 自己作品的标准。D 选项最准确。
2. 答案:A C
题型:细节题
解析:作者用来支持对 HWL 意大利语水平的的证据。
A 选项正确,依据:Judging by the problems he had in composing a simple inscription, his ability to
express himself in Italian was probably modest.
B 选项不正确,scholarly writings 概念错。原文说的 richness and variety 是指 HWL 的翻译作品。
C选项正确,The Italian nature of his translation=末句“the literalness of his translation”。

passage 214
文章结构分析:本文是典型的对比类文章结构。文章开头提出一个对比现象(海洋生态系统比陆地生
态系统 less permanence),后文对这个现象进行对比解释讨论。
陆地生态系统界限相对固定,只在千年的维度显著迁移变化。即使考虑人类活动因素,人类对海洋系
统的改变也更加间接。人只能通过对大气影响影响海洋生态系统。
1.答案:C
题型:主旨题
解析:由文章结构分析可知 C 选项正确。
A 选项错,differences in the threats 无中生有;
B 选项错,以偏概全,这并不是本文的主旨,只是文中的一个细节;
D 选项错,debate 无中生有;
E 选项错,certain consequences 无中生有;
2.答案:E
题型:推断题
解析:根据"ecological boundaries of the sea"定位到首段倒数第二句。然后,首段最后一句也提到
accomplish this,出现指示代词,可知该句也是答案句。We can accomplish this only indirectly by 与 less
susceptible to direct human intervention 对应。由此,可推知 E 选项正确
A 选项错,increasingly sensitive 无中生有;
B 选项错,是否会显著影响未知;
C 选项错,整体无中生有;
D 选项错,该比较关系在定位区间能现。

Passage 215
文章结构分析:文章描述 Bois 与斯宾塞派社会学家的矛盾。
他在巴黎的黑人美国历史与文化的展览吸引社会学家注意。但是,Bois 认为,斯宾塞派社会学家只关
注理论,只关注理论化,不关注社会进步。通过展览,Bois 希望把社会学研究从抽象概括转向个例
观察,关注个人对社会结构功能的贡献。
1.答案:A
题型:推断题
解析:根据 Du Bois 和 Spencerian sociologists 可以定位到原文第二句。本题实质上是考察长难句分析。
their (Spencerian sociologists)work examined not deeds but theories and because they(Spencerian
sociologists)gathered data not to affect social progress but merely theorize。可知 Spencerian sociologists 认
为 theorizing 更重要,综上,A 选项正确。
B 选项错,vast generalization 非答案区间;
C 选项错,relatively unimportant 比较关系的无中生有;
D 选项错,cultural artifacts 非答案区间。
E 选项错,无中生有。
2.答案:AB
题型:可多选题+推断题
解析:根据 Du Bois 和 sociology 可以定位到文章最后一句。本题仍然考察长难句分析。本句的核心结
构为:shift from A to B and C,B 和 C 所代表的部分是本题答案
observation of particular, living individual elements of society 可推出 B 选项正确。
working contributions of individual people to a vast functioning social structure 可推出 A 选项正确;C 选
项错,无中生有。

Passage 216
文章结构分析:文章是典型观点论证结构。
首句整体判断:有利环境未必导致植物种植发展。
举例支持首句判断:南中国比北中国和长江流域更温暖潮湿,野生水稻丰富,但是,南中国直到六七
千年前才种植谷物,还是得益于与长江流域的接触。
总结:环境因素重要,但不是绝对关键因素。在资源最丰富地区,游猎采集可能是更有效率的生活方
式。
1.答案:A
题型:推断题
解析:本题利用 natural resources in South China 进行定位,可以定位到文章第二句和最后一句。
原文最后一句说到, cultivation in South China may not occur,因为 foraging might be a more
efficient way of life,A 选项正确。
B 选项错,as...as 同级比较,无中生有;
C 选项错,cultural contact 非答案区间,出现在第四句;
D 选项错,been downplayed 无中生有;
E 选项错, 可以从两个方面来理解: 一、 该选项和 A 选项矛盾, 不可能是 little influence;ニ、 influence
on the type of plants 无中生有。
2.答案:B
题型:推断题
解析:利用题干定位到原文 2、3、4 句。原文说到,cereal cultivation did not occur in this region(South
China)until approximately 7,000 to 6.500 years ago.同时提到,This cultivation was likely a result of
cultural contact with and expansion from the Yangtze Basin。由此,可知 Yangtze basin 的 cultivation 会更
早,prior to 6,500 years ago.B 选项正确。
A、C、D 选项,文中无依据。
E 选项错,因果倒置。 This cultivation(South China)was likely a result of cultural contact with and
expansion from the Yangtze Basin。而不是相反。

Passage 217
文章结构分析:文章论证野猫的驯化方式。
野猫是食肉动物,自我消化新陈代谢能力弱。
野猫是独居动物,依恋特定地盘。
野猫对人类没有特定实用价值,人类无动机主动驯养。
结论:野猫利用人类环境,人类容忍野猫,导致野猫驯化。
1.答案:D
题型:推断题
解析:本题的定位信息并不好使,一是时间,一是 cats,而这两个东西贯穿全文,故而本题建议从选
项入手。
A 选项错,该比较无中生有。
B 选项错,ferrets 是能够 preforming directed tasks;
C 选项错,该比较无中生有;
D 选项正确,even as mousers,terrier dogs and ferrets are preferable
E 选项错,与首句矛盾。
2.答案:C
题型:句子作用题
解析:A 选项错,高亮句并没有 challenge-个观点;
B 选项错,long puzzled researchers 无中生有;
C 选项正确
D 选项错,高亮句是一个观点,而不是 facts;
E 选项错,formulates a new hypothesis 无中生有;
Passage 218
文章结构分析:本文是一个典型的驳论观点介绍。
首句体现 GM 观点:把 C 当作国家有三个问题(反对当作国家)。
下文具体展示这三个问题(GM 反对的理由)。
第一, C 的发现与其它部落的发现类似,除了动土量大。
第二, 居住的人少,不需要广泛的税收,贸易,进贡。
最后,考古发现不能说明 C 政治集中,经济专业分工,也不具有扩张特征。
1. 答案:B
题型:主旨题
解析:由文章结构分析可知 B 选项正确。

2. 答案:C
题型:推断题
解析:根据 political centralization 可以定位到文章最后一句。
文章说到 this does not indicate that Cahokia was politically centralized,并且文章开头说到了,Cahokia
应该是 chiefdom 而不是 state(开头说到,把 Cahokia 归为 state,会有三个主要的问题),所以,可
知 politically centralized 应该是 state 的特征。C 选项正确。
A 选项错,无中生有,和人数无关;
B 选项错,distinguished from 无中生有;
D 选项错,result from 因果关系无中生有;
E 选项错,无中生有。

Passage 219
文章结构分析:
文章开头老观点:热带树木生长年际速率固定。
However, 在 LS, CR 的研究发现,热带树木在热年慢,在凉年快。
引申新发现意义:
树木生长是光合作用与呼吸之间平衡,人为二氧化碳释放会导致热带树木释放更多二氧化碳,加剧气
候变暖。
该发现否定了热带森林作为二氧化碳海绵吸收功能的理解。
该发现与 K 的模型一致,该模型认为,二氧化碳在热带地貌的吸收凉年增加,热年减少,与大气中
二氧化碳的年际变化一致。
1.答案:C
题型:主旨题
解析:由开头两句,可以判断本文属于驳论文。
故 A 错,该选项属于立论表述。
B 选项错,underlying similarities 无中生有;
C 选项正确。a particular study 即 a study at La Selva,Costa Rica.
D 选项错,Well-documented phenomenon 无中生有;
E 选项错,reconcile two competing theories 无中生有。
2.答案:A
题型:逻辑单题-归纳
解析:本题根据 trees in the La Selva study 可以定位到第二句;同时,我们可以注意到五个选项中都
提到同个时间 1984-2000,这正是第二句的时间段,更加确定定位正确。
between 1984 to 2000,...with trees adding twice as much wood in some cooler years as they did in the
scorching El Nino year of 1997-1998。可归纳出 A 选项正确。
其余选项并无对应逻辑,此题属于简单题。
3.答案:D
题型:推断题
解析:根据 temperatures rise 和 trees in the tropical regions 可以定位到第三句: rapidly rising global
temperatures, driven by human-generated emissions of CO2, may be pushing tropical forests to release more
CO2, thereby intensifying global warming.
可知,D 选项正确。(呼吸作用更多,光合作用减少,CO2 的排放就会更多)其余选项并无依据。
Passage 220
文章结构分析:
首句事实介绍:接触欧洲人之前,北部平原土著人群体依靠捕猎野牛。
二句引入矛盾事实:野牛春季变瘦,降低营养价值,土著人群体不补充吃脂肪含量高的鱼类。
M 等人的尝试解释:习惯了瘦肉,吃鱼得病。长期食用瘦肉导致人体不能消化脂肪。
1.答案:D
题型:信息目的题
解析:此类题型往往考察的是文章的逻辑结构(句间、句内关系)
题目所问的信息在倒数第三句,而倒数第三句和第二句都是在举一个具体的例子,目的就是为了
得到最后一句的假设:native hunters' fish avoidance. D 选项正确。
A 选项错,无中生有;
B 选项错,与倒数第三句信息矛盾,原文说到 had adapted tolean-meat diets;
C 选项错,非答案区间,这是首句的内容。
E 选项错, 方向错。本题问到的信息,与上下文都是正向的关系。call into question 错。
2.答案:文章第二句
题型:句子功能选择
解析:本题需要寻找全文解释的现象,根据 Malainey et al.find a possible explanation,可知文章第二
句,正是 the phenomenon that the passage is concerned with finding an explanation for。(为啥他们不吃
鱼?)
Passage221-Passage230 解析
Passage 221
文章结构分析:
背景信息:欧亚大陆温带森林的迁徙鸣禽依赖昆虫,尤其是毛虫的夏季大量滋生。
新问题:全球变暖,毛虫滋生变早。
理论上解决方法:提早从过冬地回来。
困难:依据日长等固定线索,无法调节。
尼德兰的证据支持困难假设。
1.答案:B
题型:句子作用题
解析:A 选项错,refute the legitimacy 无中生有;
B 选项正确。
C 选项错,distinction between two theories 无中生有;
2.答案:C
题型:事实信息题
解析:本题定位到最后一句,the emergence of insects relative to the timing of songbird migration,可知 C
选项正确。
本题最具干扰的选项是 E 选项, 文章确实说到 a significant decline in populations of pied flycatchers,
但是文中并未说到这是 populations of insects 上升原因。

Passage 222
文章结构分析:问题解决结构。
新问题:全球气候变化使得局部保护无意义。
解决方法:人为帮助迁徙。
1.答案:B
题型:推断题
解析:定位到文章倒数第二句,由 places that humans have made impassable.可推导出 B 选项正确。
A 选项错,disappear 无中生有;
C 选项错,比较关系无中生有;
D+E 选项,亦在文中找不到依据。
2.答案:D
题型:推断题
解析:文章开头说到了 Widespread climate change challenges traditional notions that preserving specific...
由此可知,D 选项正确。

Passage 223
文章结构分析:
新现象发现:灰松鼠数量下降,红松鼠迁徙到印第安纳碎片化森林。
事实:灰松鼠分散储存,红松鼠集中储存,灰松鼠是更高效种子传播者。
新现象带来问题:红松鼠不能补偿灰松鼠种子传播功能。
1.答案:A
题型:推断题
解析:本题问 red squirrels 和 gray squirrels 的区别。文中出现多处这种对比,本题无法用题干进行准
确定位,可用选项来定位。
A 选项正确,对应最后一句。 red squirrels successfully colonize...,they may not
compensate completely for the loss of gray squirrels as seed dispersers。
B 选项错,无比较;
C 选项逻辑错,gray squirrels are scatter hoarders;
D、E 错,无比较。
2.答案:倒数第二句
题型:句子功能选择
解析:题目要求选择支持另外一句声称判断的说明性句子。倒数第二句是对于红松鼠、灰松鼠扩
散种子能力的对比说明,用来支持末句判断:红松鼠不能完全补偿灰松鼠的种子传播功能。

Passage 224
文章结构分析:文章开头引入个问题:How good an artist was Rockwell?之后,开始对其 technique
进行评估。
缺点:competent but undistinguished / anatomical competence limited
优点:fastidious, attention to detail
1.答案:C
题型:主旨题
解析: C 选项正确。
A 选项错,change 无中生有;
B 选项错,overrated 无中生有;
D 选项错,placing...in its historical context 无中生有;
E 选项错,contrasting 无中生有。
2.答案:E
题型:推断题
解析:His technique...undistinguished; his anatomical competence was limited; However, Rockwell’s work
does exhibit a genius in...可知 E 选项正确。
A 选项错,evaluated fairly 与首句矛盾;
B 选项错,because 无中生有;
C 选项错,artistic merit 无中生有;
D 选项错,rejected by 无中生有。
3.答案:首句
题型:句子功能选择题
解析:首句“Norman Rockwell was the most popular illustrator in the United States during the
mid-twentieth century”是对其当时艺术名声的说明。

Passage 225
文章结构分析:
问题:生态系统研究面临 porous frontier,无法控制边界。
解决方法:研究岛屿生态,自然的隔离实验室,然后应用到大陆。
1.答案:C
题型:推断题
解析:porous: easy to pass or get through。可知,C 选项正确。
2.答案:D
题型:词汇题
解析:islands 与前面 porous frontiers 对立,故而本题应选择 not easy to pass or get through,即 D 选项
relatively inaccessible(此处更像是考察 inaccessible 这个单词)。

passage 226
文章结构分析:
事实信息:pen flourishing 作为图书边缘装饰 1400 世纪在北尼德兰流行。
意义价值:flourishing 的地区差异能用来判断手稿的年代和地区。
判断年代好使:印刷书籍年代准确,可用来判断使用同样 flourishing 的手稿的时间。
判断地点不准确:书印好后往往不装饰,由买者在异地完成装饰。
1.答案:A
题型:句子作用题
解析:A 选项正确
B 选项错,restate 无中生有;
C 选项错,correct a misconception,高亮句与前文是正向关系;
D、E 选项,无中生有。
2.答案:D
题型:事实信息题
解析:根据 pen flourishing in manuscripts,valuable information 可以定位第二句。
useful tool for localizing and roughly dating manuscripts.可知 D 选项正确。

passage 227
文章结构分析:文章主要按照时间线索介绍史实。
20 世纪早期,旧金山 B 海岸在西部是非洲美国人爵士乐的中心。
1921 年,禁酒运动关闭 B 海岸,洛杉矶成为主要爵士乐中心。
关闭之前,洛杉矶已经开始吸引爵士音乐家:它有最大的西部黑人社区,成长的电影工业,萌芽的录
音工业。1917 年新奥尔良关闭 S 地区之后,洛杉矶已经吸引爵士音乐家。
1.答案:E
题型:主旨题
解析: 首先,本文是立论。故而,B 选项,alternative 驳论型排除;D 选项 refute 排除。
A 选项, parallel effects 过于细节,排除
C 选项, importance of Californian cities 无中生有
E 选项正确。
2.答案:E
题型:细节题
解析:本题问 20 世纪早期,为什么 San Francisco 会成为 Jazz center.定位到文章第二句,many of them
African American。同时,注意到文章最后一句说到,随着 African American 的增加,Los A Angeles was
already a magnet for jazz2 可知 African American 的数量是影响一个地方能否成为 center 的因素。E 选
项正确。
3.答案:B
题型:推理题
解析:利用 1921 可以定位到原文,指出 government closed the Barbary Coast. This closure
was the decisive event that established Los Angeles as the premier center for jazz
可知 B 选项正确

passage228
文章结构分析:评论最近的艺术展览。
最近的展览是关注社会意识形态理论的视觉形象反制。
以 G 的绘画为典型举例:很长时间被学者边缘化 EM 的学徒,展览显示 G 的才华。
展览的缺陷:陪展展品让人失望。艺术家同时代的评论文章没有历史艺术价值。评论文章观点单一,
回避学术辩论机会。
1.答案:第三句
题型:功能句子选择
解析: Long marginalized by scholars as little more than an apprentice of Edouard Manet
Gonzales is revealed by the exhibition as possessing remarkable, if uneven, talents.这句话很明显是说
以前 G 这个人只被当成一个学徒(贬义),二现在是 remarkable talents(褒义),所以答案是这句话。
2.答案:C
题型:词汇题
解析;此题較难,必须把握整个文章的结构才能推出来。后文都是在说这个 exhibition 有很多欠缺的
地方,而文章的观点是再说 G 这个人是正面评价的,所以正好对这个 exhibition 是起到个补充的作用,
所以 complement,是最合适的选项。

passage 229
文章结构分析:本文的结论是最后一句: Industry must have instituted highly effective energy
conservation measures 削弱结论,就变成了 Industry must have not instituted…
答案:E
题型:逻辑单题削弱题。
解析:找他因思路,并不是 industry 采取了什么有效的能源措施,而是其他原因,使得 1980 的能源
消耗低于 1970。
A 选项错,如果使用低价油以后,1980 的能源消耗不一定会低于 1970
B 选项错,本文讨论的 industrial use of energy,而和 Residential energy consumption 没
有关系。
C 选项错,无关。1970 之前,不关注能源保护,和我们讨论的 between1970 to1980 无

D 选项错,原文已知细节的补充,并不影响结论
E 选项正确, industrial use of energy 的降低,是因为能源密集型的企业产量减少,而不是因为 effective
energy conservation measures。找他因成功。
passage 230
文章结构分析:这篇文章的推理过程大致是:①松树不能活在水里(只能活在干旱期)→干旱结束后
J 松树一定会死亡;②J 松树只活了 120 年:(①+②)→干早持续时间少于 150 年。这个推理过程存
在的问题是,它无法确定干旱开始的时间,因为可能在 J 松树开始生长前干旱期就早已开始。
答案:E
题型:逻辑单题假设题
解析:A 选项错误,1200 年后其他树种是否开始生长并不影响文中推断,因为文中提到的年代测定方
法是针对 J 松树的,不会受到其他树种影响,即使有其他耐水树种始终存活,也与本文讨论的问题无
关。
B 选项错误,更深的湖底中是否有树存在不影响文中推断,因为树木生长的湖底区域深浅与否充其量
只能说明干旱的程度,不影响对干早是否存在的判断。
C 选项错误,是否有树活过整个 120 年不影响文中推断,因为文中提到的年代检测方法已经证明 1200
年至 1320 年都有 J 松树生长。即使每棵树只活了一年,也不影响“J 松树存活于 1200 至 1320 年间”
这一推断。
D 选项错误,后期是否有干早再次发生不影响文中推断,因为后期是否还有干早发生与这时期的研究
无关。同时,通过年代检测方法已经证明松树只存活于 1200 至 1320 年的连续 120 年间,即已经可以
说明文中特指的这一干早期结束于 1320 年。
E 选项正确,因为文中没有证据说明干星何时开始,如果在 J 松树开始生长时干早已经持续了超过 30
年,那么再加上 J 松树生长期的 120 年,总干早期就会超过 150 年,与文中结论相违背。
Passage231-Passage240 解析
passage 231
文章结构分析:题目说飞机上被推荐的电影因为画质差,所以 C 不喜欢,然后被嘲笑的电影 C 也不
会喜欢,所以 C 不会在飞机上喜欢任何电影。
答案:C
题型:逻辑单题假设题
解析:题目有一个遗漏的地方,就是既没有被推荐,但也没有被嘲笑的电影 C 是有可能看的,所以如
果要题目的结论成立的话,那么必须要求不存在既没有被推荐,但也没有被嘲笑的电影。所以正确答
案是 C 选项。

passage 232
文章结构分析:文章的意思是说近期海豚因为传染病死了很多,在海豚体内发现能够削弱抵抗传染病
的化合物,化合物的来源是船画,而且海豚自己除化合物,所以只要船画被禁止了,海豚的死亡率就
会下降。
答案:C
题型:逻辑单题加强题
解析:题目需要加强这个结论。C 选项能够起到加强的作用,因为如果化合物被海豚排除之后然后又
降解成为无害的物质的话,那么水中就不会有这种化合物,那么海豚死亡率自然会下降。A 选项是个
削弱选项,错。B 没有说海豚,错。D 选项也是没有提到海豚,错。E 选项也没有直接提到禁止船画
就能减少海豚死亡率的逻辑,错。

passage 233
文章结构分析:文章的意思是说 iridium 是很稀少的东西,高浓度的 iridium 有两个来源:①火山爆发;
②陨石,科学家发现了高浓度的 iridiun 出现在恐龙灭绝的年代,所以推断出陨石是导致恐龙灭绝的
原因。
答案:A
题型:逻辑单题加强题
解析:题目要我们找一个加强结论的选项。A 选项正确,因为这个题无非就是有人怀疑万一是火山爆
发导致恐龙灭绝呢?A 选项把这个可能性基本排除了,所以起到了加强结论的作用。B 选项时间和题
目不一致,肯定无法加强。C 选项和题目陨石导致恐龙灭绝是矛盾的。D 选项杀死了其他生命不能加
强陨石使恐龙灭绝的结论。E 选项选项完全是无稽之谈,时间和逻辑方向全错了。

passage 234
文章结构分析:文章说蝴蝶离开出生地产卵,然后毛毛虫(蝴蝶小时候)吃出生地的植物,吸收
glycosides,而且不同地区的 glycosides 是不一样的,所以可以根据蝴蝶体内的 glycosides 来确定蝴蝶
的出生地。
答案:A
题型:逻辑单题加强题
解析: A 选项其实是文章的一个假设,就是说毛毛虫变成蝴蝶之后不能再吃含有 glycosides 的植物
了,要不然体内就混合了不同地方的 glycosides,所以 A 是一个加强选项。B 选项看到 toxic 直接排
除。C 选项对文章结论没有任何加强作用,排除。D 选型说到其他东西,不能加强本文的结论。E 选
项和 D 选项同理,排除。

passage 235
文章结构分析:
M 蝴蝶离开出生地几百英里产卵,然后毛毛虫(蝴蝶小时候)只吃出生地的 M 植物,吸收 G 物质。
蝴蝶长大后保持作为毛毛虫吸收的 G 物质。
研究发现特殊情况:成年蝴蝶的 G 物质组合总是匹配出生地 M 植物的独特 G 物质组合。
答案:D
题型:逻辑单题推理题
解析:题目要求从一个特殊现象(成年蝴蝶 G 物质组合总是匹配出生地 M 植物的独特 G 物质组合)
得出合理推论。可以主动设想该特殊现象成立的条件,或者设想何种情况下,该特殊现象就不能成立。
如果蝴蝶离开出生地,继续吸收储存其它地方的 M 植物的 G 物质呢?显然特殊情况成立的可能性大
大降低(由 distinctive 可知不同地区 G 物质组合不一样)由此可知 D 选项正确。该选项否定了破坏特
殊情况成立的可能性,反过来,就是该特殊情况的合理推论。

passage 236
文章结构分析:文章关于对比实验结果。
实验 A:压力下,ag( adrenal glands)会增加 c( corticosterone),然后 wb( white blood)减少。
实验 B:压力下,没有 ag,Wb 没有减少
结论:ag 产生了 c 所以导致了 wb 减少。
答案:C
题型:逻辑单题评价题
解析:该对比实验有一个地方不严谨:wb 不ー定是由于 c 引起的,很可能是 ag 产生了另外一种激素,
而这种激素引起 wb 减少。所以正确答案选 C 选项: ag 是否产生 c 之外的其它激素。

passage 237
文章结构分析:文章很明显是两个直接对立的观点,前面的观点认为孩子的 psychology 是不一样的,
后面的观点则认为孩子的获得行为特征的心理方法( psychological mechanism)是一样的。
答案:D
题型:细节题
解析:他们对于 psychology 的理解是不同的,所以答案选 D 选项。

Passage 238
文章结构分析:文章开头说到,学者未能把 Addams 的 ideas 和其他 activists 和 public intellectuals
做足够的对比。然后在文章最后,又提到了 such contextualizing effort 是能够带来好处的。
1. 答案:D
题型:目的主旨题
A 选项错, extent of her debt to other social activists and public intellectuals 无中生有。
B 选项错, misunderstood 无中生有
C 选项错, debate...regarding Addams’ approach 无中生有
E 选项错,过于细节,以偏概全。
2.答案:E
题型:目的题
根据高亮信息定位到原文,可知从句提到相关的信息,目的是为了服务主语提到的 her contribution
emerges as far more cooperative than groundbreaking.E 选项正确

Passage 239
文章结构分析:
研究方法的问题:使用史前洞穴中大型哺乳动物骨骼研究当时气候变化有问题。
原因:一些物种适应性强;骨骼被移动很远距离。
建议方法:使用小型哺乳动物化石更准确。
理由:小性动物对环境敏感,迁移距离短。
1.答案:A
题型:细节题
解析:文中说到, using large mammals' bones can be problematic.可知,答案将在下一句产生。由 both
carnivores and humans can have lager hunting territories and…可知这是其中的一个 obstacle,A 选项正
确。B 选项错误,文章最后一句说到了 small mammals providea better index of climate change.C 选项
正确,在举例是,提到了, such as red deer
are very adaptable-…这也会到 reconstruct past climate changes 造成障碍。答案:A
2.答案:E
题型:举例作用题
解析:举 red deer 为例,目的是为了说明 using larger mammals’ bones 来做预测,是有问题的,是不
好的。E 选项正确。A 选项对应原文事实信息,不是作者举例的目的。

Passage 240
文章结构分析:
针对使用非本地物种恢复火后森林的实践,作者主要持怀疑否定态度,认为其会妨碍本地物种,引进物种难于
根除,能加重未来火灾的严重性。

1.答案:A C
题型:细节题
解析:根据关键词“replanting an area with dense stands of fast-growing conifers”定位原文。
A 选项对应:truncates the biologically rich early stage of ecological succession。
C 选项对应:increase the severity of future fires。

2.答案:B
题型:推理题
解析:根据题干关键词定位文章开头。B 选项依据二句:Moreover, native seed sources are almost
always sufficient for early nature reestablishment of native species…
Passage241-Passage250 解析
Passage 241
文章结构分析:

文章首句介绍新发现:“火星环球调查者”发现了磁化的残存物。二句是作者推测:这些残存物

可能是全星球磁场的遗迹,该磁场位于 38 亿年前的陨石坑的附近。三句及后句是作者的推理。

三句(if 句)说,即使火星曾经存在磁场,该磁场应该是在陨石坑形成前消失了。末句是作者基

于前句推理所做的总结结论:火星磁场在其历史的早期就消失了。

1.答案:D
题型:细节题
解析:A 选项错,无中生有
B、C 选项错,文中并无此种比较关系
D 选项正确,对应, ancient craters in the Terra Sirenum region, astronomers believe
these craters were formed some 3.8 billion years ago from the impact of chunks of debris.
E 选项错、非答案区间。下ー句并不是本题的讨论范国。
2.答案:C

题型分类:细节题
解析:题目问末句结论句(火星磁场早就消失了)的依据。根据文章结构可知末
句前的推理是其依据。C 选项依据末句的前句,意思是说,如果撞击的时候有磁
场,那么就会在冷却的时候重新磁化。潜在逆否命题是:没有重新磁化的磁场,
所以撞击的时候没有磁场。这是末句结论的关键推理。
3.答案:E
题型:目的题

解析:根据文章结构,高亮内容出现在作者介绍完发现后,对其他天文学家关于这些陨石坑

是如何形成的观点的介绍。Craters 可理解为“features of the surfaces of Mars ”

Passage 242
文章结构分析:文章结构分析:
几个范畴概念:
1 people who speaks T
2 people who live in T
3 people who speak several languages
文章在描述完几个范畴概念之间关系后,末句得出一个意外结论,要求填入合理原因解
释。
答案:C
题型:逻辑单题填空题
解析:

92
由文章对几个范畴概念描述,可用韦恩图大致表示其关系:1在2范围内,2又几乎在3范
围内。合理的推理应该为:1几乎包括在3之内。但文章结论为:1和3几乎没有交集。哪
个选项能让1和3交集最少,又不违背文章已有信息呢?如C所说,2之中属于1的很少。
这样,既满足了“1和3几乎没有交集”(能说几种语言的T居民几乎不是说T语言的人),
又满足了文章已有信息:1在2范围内,2又几乎在3范围内。

Passage 243
文章结构分析:
文章主要介绍19世纪对更大音量钢琴的需求引起的后果影响。很多艺术家反对铁制的钢
琴结构,认为其破坏钢琴的音色。
1. 答案:A C
题型:推理题
解析:根据选项定位:
A选项,依据二句“long after”, 末句“debate”,仍在考虑是否接受,仍有激烈争论,
可知钢琴制作仍掌握在艺人手里。
B选项,不能确定。末句说导致了激烈争论,没有明确结论。
C选项正确。文章介绍的需求变化。钢琴的铁制结构发生在工业革命后,依据时间对比
取非,之前都是木制结构。
2. 答案:C
题型:词汇题
解析:由文章讨论的场景,和louder(音量大小)相关的应该是音符的质量、音色,C最合
适。

Passage 244
文章结构分析:
首句体现作者总体观点:18世纪妇女在英国政治生活中发挥重要作用。二句是支持首句
的事实性信息。三句(Whereas句)介绍历史学家的新观点:认为妇女的作用是连续的
常态的。四句、五句、六句展开支持三句历史学家新观点。文章最后是作者观点,表达
了对历史学家观点对质疑。
1. 答案:C
题型:推理题
解析:由定位点之后作者本人观点,可知C选项。
2. 答案:A
题型:细节题
解析:题目问当前历史学家与之前历史学家对于18世纪妇女政治角色问题的区别。
A选项“commonplace”依据原文:normalcy.
3. 答案:A
题型:推理题
解析:由文章最后作者的质疑,可知A选项正确。

Passage 245

93
文章结构分析:
文章是比较典型的现象解释结构。首先描述T蚂蚁寻找确定巢穴点的特别行为。Some
researchers believe句是他们的解释观点。末句是支持研究者观点的观察证据。
1. 答案:B

题型:推理题

解析:定位“researchers”观点句,可知他们结论的潜在前提条件是之前“Experiments”

的发现:能区分个人和同伴的信息素。B选项是对该能力功能的明确阐述。

2. 答案:A

题型:句子功能题

解析:由文章结构,可知末句高亮句是用来支持前句 researcher 的观点的观察证据。A

选项含蓄表达了支持作用。

Passage 246
文章结构分析:典型的现象解释结构。

首先指出矛盾现象:月球和火星早期有磁场,如今都不再拥有。水星大小在月球和火星

之间,但仍有活跃磁场。按说水星的核也应该是固体的,不应有磁场。

文章最后解释:水星硫元素与铁相混合,降低了熔点,使得核心处于融化状态。

1. 答案:A

题型:细节题

解析:根据上下文,此处“by all right”应该理解为“本应,应该”之意。此处判断推

理的依据是前文指出的水星的大小。

2. 答案:B

题型:词汇题

解析:根据上下文,此处“dead”指的是没有活跃磁场,可知 B inert“惰性的,不活跃

的”最符合要求。

A 选项 latent(潜伏的),C 选项 dormant(休眠的)都含有“暂时不活跃,仍有可能变得活

跃”之意。

Passage 247
文章结构分析:

94
首句是主旨句,总体判断 17-18 世纪应对女性独特需求的历史重要性。
后文安装时间顺序介绍该发展:先有图书关注女性顾客(高亮句),后有家具突出女性
需求。
文章最后是作者对该事件的评论:强化了女性社会角色,“女性”被狭义定义。
1. 答案:E

题型:推理题

解析:题目问文章作者的立场。由文章结构可知,文末是作者的立场观点。E 选项同义

改写末句。

2. 答案:A

题型:细节题

解析:根据时间线索“1760‘定位,后句“Of course”让步可知 A 选项。

D 选项,由“in the extension of the range of…”不能明确推出之前“small”。

3. 答案:C

题型:句子功能题

解析:根据文章结构,文章重点讨论以家具为代表的“object”商品的对女性顾客需求

的强调的历史现象,并讨论其影响。高亮句对图书印刷品女性特殊化的介绍主要是作为

历史背景。C 选项概括最准确。

Passage 248
文章结构分析:典型的文学评论。
首句让步:WW 的“An Evening Walk”吸取了 18 世纪的描述传统。
二句(高亮句)是支持首句的证据。
三句是作者核心观点,是对主流研究的评论:对 W 的研究过于狭窄。
Because 句是支持上句核心观点的理由。

1. 答案:C

题型:句子功能题

解析:由文章结构分析,可知 C 选项概括最准确。

2. 答案:A C

题型:推理题

解析:题干关键词“mainstream twentieth-century critics”定位结合选项定位。

95
A 选项正确。由文章结构,首句、二句是让步内容,作者认为没有争论,主流评论者也

会同意:… has dominated critical reaction to the poem…

B 选项不正确。由…wax and wane…可知不能形成一致意见。

C 选项正确。由…salutary revolt against the sedate norms of eighteenth-century culture…可

知主流评论者更高评价对于传统的否定破除。

Passage 249
文章结构分析:典型的图书评论。
首句引入主题:J 的新书关于女性对废奴运动的参与。
二句(While 句)指出 J 书的独特价值:不同意传统观点的特定方面。
For example 句及后句展开二句:女性在废奴运动中没有边缘化,找到持续开展的方式;
黑人女性的作用也更直接,更重要。
1. 答案:C

题型分类:主旨题

解析:根据文章结构,作者在介绍 J 新书成就过程中,主要采取将其传统、主流研究对

比的方式,C 选项准确概括。

2. 答案:D

题型分类:细节题

解析:根据关键词定位,可知 D 选项是定位句内容的同义该写:circulation of petitions.

3. 答案:B

题型分类:目的题

解析:由文章结构,可知高亮例子是 J 用来支持自己立场的例子。J 反对传统观点认为

的女性在废奴运动中被边缘化。

Passage 250
文章结构分析:典型的分总论证结构。
首句是理由 1: 人们基于人与食肉动物的相遇来判断食肉动物保护项目的价值,人与郊
狼的相遇不利于郊狼引入项目。
二句三句是理由 2: 食肉动物的价值在于控制鹿群,该观点也不利于狼引进项目。
Thus 句是文章结论:郊狼成为重要的保护问题(有争论)。

1. 答案:A C

题型分类:细节题

96
解析:

A 选项依据理由 2:This perception also decreases public support for wolf reintroduction

because it suggests that additional predators are unnecessary for controlling populations of

prey species.

C 选项依据理由 2: At the same time, coyotes are commonly, though not necessarily correctly,

credited with lowering the density of the deer population in parts of the region.

2. 答案:D

题型:词汇题

解析:结合上下午,可知“controlling”此处是降低、减少鹿群数量的意思,D 最准确。

Passage251-Passage260 解析

Passage 251
文章结构分析:典型的现象解释结构。
首句提出需要解释的问题:火星为何有如此稀薄的大气层?
二句黑体句和后句提出老解释:化学反应导致早期浓稠大气变成固体。火星体积小,冷
却快,火山不活跃,不能循环气体。
However 句和 Moreover 句提出对老解释的质疑:发现碳化岩石证据少,不能解释氮和
惰性气体少。
Escape 句提出作者支持的新解释:气体遗失到太空。

1. 答案:D

题型分类:推理题

解析:根据质疑老观点的证据取非,可得 D 选项。

2. 答案:C

题型分类:细节题

解析:火山出现支持老解释的第三句:…cooled quickly and its volcanoes---which might

have released dissolved gases back into the atmosphere---stopped erupting. 可知 C 选项正

确。

Passage 252

97
文章结构分析:
首句指出好趋势:造林能导致颗粒物减少。However 句提出潜在的限制因素。
因素 1: 病虫害,干旱,火灾等干扰因素。
因素 2: 气候变化。
因素 3: 私人林场主经营行为。
1. 答案:B
题型分类:主旨题
解析:根据文章结构分析,文章开头的趋势可理解为 strategy, 文章之后讨论的是该策略的限制
性因素。
2. 答案:B
题型分类:目的题
解析:关键词“agriculture”出现在因素 3: 私人林场主经营决策行为。可知农业是影响林
场主决策行为的因素。
3. 答案:A
题型分类:推理题
解析:关键词“Alberta”出现在因素 1:病虫害,干旱,火灾等干扰因素。结合原文定位可
确定 A 选项。

Passage 253
文章结构分析:
首段介绍老理论:性别社会化理论认为,获得性性别偏见影响工作倾向性。Earlier
research 提供支持证据。
二段 However 句转折,提出对老理论质疑。Heckert 发现与老理论不一致的证据。
1. 答案:C
题型分类:推理题
解析: Heckert 的研究发现出现在第二段,根据文章结构可知是对于老理论的质疑。C 选项最准
确:H 的发现不能用老理论明确解释。
2. 答案:A
题型分类:推理题
解析:题目问早期研究与 H 研究的区别。结合文章结构,对比可发现 A 选项。

Passage 254
文章结构分析:
首段首句是主旨句:一直到 19 世纪初,化学处于边缘地位。首段后句基本都是支持主
旨句的历史证据事实。
二段描述当时化学的应用情况,化学被当作实用学科,而非基础理论学科。
三段从科学界的角度,描述化学不被重视的情况。
1. 答案:C

题型:细节题

解析:HB 出现在首段让步内容中,可知 HB 是例外。

2. 答案:B

题型分类:推理题

解析:“universities” 主要出现在首段,优先从首段寻找定位依据。

98
B 选项可定位到首段末句:Chemistry, unlike medicine, did not constitute a profession in its

own right.

3. 答案:E

题型分类:细节题

解析:在二段中,化学主要被当作:practical application, practice, tool。pragmatic 是其同

义词。

4. 答案:B C

题型:推理题

解析:高亮关键词出现在二段。

B 选项依据:…military chemists worked in many nations on the improvement of gunpowder.

C 选项依据:…to solving problems of water quality, street lighting, sewage disposal, and

more

Passage 255
文章结构分析:

文章开头是事实性介绍:以 Mozart 命名的一个音乐掷骰子游戏。

Some music historians 句是这些音乐史家的观点,以及文章作者对于他们观点的评论。


作者同意音乐史家的观点: Mozart 不依赖于这样的系统去作曲,但作者不同意音乐史
家,作者认为 Mozart 也用过类似的东西。
In fact 句及之后,是作者论证自己观点的理由:
Mozart 喜欢游戏,手稿中存在游戏类似的片段。
Mozart 同时代的音乐家用过类似的作曲手段。

很多 Mozart 同时代的作曲家设计过掷骰子游戏。

末句 however 转折,作者认为,游戏本身的琐碎特征说明不是 Mozart 本人的设计。

1. 答案:B

题型分类:细节题

解析:作者用来否定 Mozart 是该游戏作者的证据出现在末句。B 选项,the quality 对应

triviality of the music.

A 选项是用来支持 Mozart 也用过类似的作曲工具的证据。

C 选项对应内容出现在文章开头事实介绍部分。

2. 答案:E

99
题型分类:细节题

解析:题目要求找出与作者观点最一致的选项。

E 选项最符合 Some music historians 句及之后内容体现的作者观点:Mozart 很可能使用

过类似的工具,但他不是以自己名字命名的 musical Dice Game 的直接作者。

3. 答案:E

题型分类:推理题

解析:根据关键词“music historians”定位到该句:…their view that Mozart was too creative

to have used such a device is inappropriately based on a nineteenth-century Romantic concept

of composers. 可知 E 选项正确。

Passage 256
文章结构分析:典型的问题描述型结构。
文章开头引出水获得、水管理问题对玛雅低地人民的重要性。However 句强调水问题对
于多数低地人民的严重性。In response 句提出应对解决办法。Even 句之后仍然说水问题
的严重性。
1. 答案:B

题型分类:目的题

解析:高亮内容出现在末句。B 选项对其功能描述最准确。

Hypothesis 对应 many researchers to suspect that…

高亮句内容是对支持该观点证据来源的说明。

2. 答案:C

题型分类:推理题

解析:依据关键词“Tikal”定位,知其是解决水问题实践的例证。根据文章结构,Even

句及之后继续说水问题的严重性。可推知 C 选项,Tikal 这样的设施不能应对严重的干

旱。

3. 答案:A

题型分类:推理题

100
解析:根据关键词“cenotes”定位到文章第二句。根据文章结构,However 句及之后说

其他地区水位低,需要建造人工水库,可推知使用 cenotes 的地区不用建造人工水利工

程。

Passage 257
文章结构分析:
研究者钻井深处发现之前认为不可能存在的生物。尽管之前在同样深度发现的微生物实

际上是从地表带过去的,这次的发现不太可能是从地表带去的,是因为什么。

可知空格应该支持本次发现确实是来自地层深处。

答案:A

题型分类:逻辑单题填空题

解析:如果确实如 A 选项所说,这次发现的微生物不能在地表生存,支持来源于地层深

处的说法,降低是地表带过去的可能性。

Passage 258
文章结构分析:
文章首句提出观点:来自马的竞争对于历史上野生动物的分布及数量有影响。

后文用 Columbia River basin 的野牛和马的例证来支持首句观点。文章最后把马对野牛

的竞争总结为两点:1 直接食物竞争。2 提高狩猎效率。

1. 答案:C

题型分类:句子功能题

解析:根据文章结构,高亮句用来展开支持直接前句 First 句。

2. 答案:B C

题型:细节题

解析:依据选项定位。

B 选项依据:……that is, sometime after about 1720.

C 选项依据文章倒数第二句(高亮句)。

Passage 259
文章结构分析:
文章首先提出需要解释的现象问题:火星月亮的来源。

101
One theory 的解释是:火星月亮形成于小行星带,后被火星引力捕获,依据的理由是火
星月亮表面与小行星表面的相似性。
However 句提出质疑:不能解释火星月亮的轨道特征:近乎圆形,接近赤道平面。
末句提出另一种解释:火星月亮由碎片盘形成于火星轨道附近。
1. 答案:B

题型分类:推理题

解析:however 句反驳“One theory”(认为形成于遥远的小行星带)时提出,该理论不

能解释火星月亮的轨道特征:近乎圆形,接近赤道平面。可知 B 选项正确。

2. 答案:A

题型分类:细节题

解析:定位“one theory”,可知其依据是:based on striking similarities between the physical

characteristics of the surfaces of Mars’ moons and the surfaces of numerous objects in the

asteroid belt…可确定 A 选项正确。

Passage 260
文章结构分析:
关于 A 蜘蛛的蛛网的 S 特征,被新发现具有收集饮水功能。作者认为该发现与之前认
为的 S 用来强化网络的观点不矛盾。
空格要求填入支持 S 用来强化蛛网的理由。
1. 答案:D

题型分类:逻辑单题填空题

解析:如果如 D 选项所说,用来收集饮水的蛛网要求更坚固,则调和了新旧两种关于 S

功能的观点。

102
Passage261-Passage270 解析

Passage 261
文章结构分析:
文章开头是 Cole 发现的关于蚁群的矛盾现象:蚂蚁数量少,行为呈混乱模式,蚂蚁数
量多,行为呈现秩序模式。Why 句提出密度和秩序行为的关系问题。后文解释两者之间
关系:蚂蚁之间相互作用,活跃的个体刺激不活跃的个体进入活跃状态。密度越高,刺
激的机率越高。
1. 答案:A B
题型分类:细节题
解析:关键词“interaction”出现在解释部分。
A 选项依据:…and an active ant encountering an inactive one will stimulate the latter into
movement.
B 选项依据:At low densities there are few encounters…
C 选项没有依据。
2. 答案:C
题型分类:词汇题
解析:在上下文中,“ordered”对应 rhythmic, 相互协调的,有秩序的,有规律的。C
选项,协调的,相互一致的。

Passage 262
文章结构分析:
本文是典型的时间对比结构。
1865 年前,女性杂志针对精英阶层的读者,价格高,几乎没有广告,内容偏重文学、礼
仪、时尚。
1865 年后,识字率上升,印刷术、交通物流改进,读者数量激增。广告增加,杂志价格
下降。女性杂志在保留时尚娱乐特征同时,广告消费属性增强。
1. 答案:D
题型分类:细节题
解析:题目要求找出 1865 年前后女性杂志的共同特征。D 选项对应 1865 年后对女性杂
志的让步部分:…articles on cultural affairs and other nonfiction topics…
2. 答案:A C
题型分类:推理题
解析:题目问 1865 年前,妇女杂志读者群受到的限制。AC 选项依据 1865 年后妇女杂
志读者群新发展的条件取非。
A 选项:…a larger and cheaper postal service permitted…
C 选项:…allowed publishers to reduce prices of the magazines…

Passage 263
文章结构分析:问题描述结构。
文章首先总体介绍问题:使用无区分杀虫剂的副作用:杀死捕食害虫的好昆虫,间接降
低好昆虫的生育率,导致有害昆虫恢复快。

103
后文详细展开问题描述:在捕食者/被捕食者系统中,捕食者控制被捕食者死亡率,被
捕食者控制捕食者出生率。当捕食者和被捕食者族群同时下降,被捕食者的出生率受影
响更小,恢复更快。这时,因为缺乏捕食者,被捕食者的有害昆虫往往达到比之前更大
的规模。
1. 答案:A
题型分类:细节题
解析:题目问捕食者和被捕食者昆虫受无区分杀虫剂影响的区别。A 选项对应原文:the
bad insects will rebound much more quickly.
2. 答案:D
题型分类:细节题
解析:定位原文,“side effect”描述,杀死捕食害虫的好昆虫,间接降低好昆虫的生育
率,导致有害昆虫恢复快。可知 D 选项正确。
3. 答案:C
题型分类:推理题
解析:根据文章对使用无区分杀虫剂“side effect”描述:杀死捕食害虫的好昆虫,间接
降低好昆虫的生育率,导致有害昆虫恢复快,甚至恢复到更高水平。
对比 C 选项和 D 选项可能的效果。
C 选项:使用特定只杀吃庄稼有害昆虫的杀虫剂。避免同时杀有益昆虫的副作用,有益
昆虫朗多肉少,提高捕食有害昆虫的效率。
D 选项,交替使用无区分杀虫剂和特定只杀吃庄稼有害昆虫的杀虫剂。无区分杀虫剂总
会杀死部分有益昆虫。不如直接只使用区分杀虫剂效率高。

Passage 264
文章结构分析:
过去,进口药品的检查问题率是 25%。今年,检查的人手和数量翻倍,进口药品的检查
问题率降为 21%。
结论:进口药品质量提高了。
答案:B
题型分类:逻辑单题评价题
解析:对正确选项的问题回答是或否,能直接影响结论是否有效,也就是针对了影响结
论的关键因素。如果如 B 选项,新增加的检查人手不如老人手有经验,那么检查出的问
题药品的比例下降可以归结为检查效率降低,而不是结论所说的质量提高。

Passage 265
文章结构分析:文章属于文学艺术评论。
首句是作者的判断观点:小说 G 不应该是 D 的作品,而应该是 FM 的作品。
作者在后文主要从风格对比的角度,提供支持首句的理由。
1. 答案:E
题型分类:推理题
解析:E 选项是大正小负判断。
小负依据:…is full of redundancies, repetitions, and non sequiturs.

104
大正依据末句:the charm of Dumas’ novel and its “narrative wisdom”…
2. 答案:B
题型分类:细节题
解析:定位到 For example 句:Georges’ characters are tiresomely earnest, while those from
Dumas’ well-known novels crack jokes and utter colorful oaths.

Passage 266
文章结构分析:
首句是作者总体判断观点:文艺复兴艺术家的写作作品能帮助了解他们的生活个性。
二句用举例对比方式,支持首句观点。
三句继续用例证方式,支持首句观点。
However 句及之后指出矛盾例外:达芬奇写作作品多,但个人生活个性仍然难于了解。
1. 答案:A
题型分类:目的题
解析:根据文章结构,可知 A 选项最准确,达芬奇是首句观点方法(写作作品帮助了解
艺术家生活个性)的例外。
2. 答案:D
题型:句子功能题
解析:根据文章结构,高亮句是支持首句(前句)的理由证据。

Passage 267
文章结构分析:
文章首段首先提出事实问题:我们看到的亚里士多德的作品不是他的正式出版物,而是
他为自己标注的笔记。首段接着指出这一事实的利弊。
利:利于专家学术研究。
弊:难懂,歧义。
二段继续描述弊端方面:他的代表作品仍有矛盾解释,关键概念仍有歧义。
1. 答案:D
题型分类:推理题
解析:根据关键词定位,可知 D 选项有依据:the Aristotelian works available to us are the
ones that intellectually sophisticated commentators in late antiquity (A.D.300-600) found
philosophically most rewarding…
2. 答案:B
题型分类:推理题
解析:根据高亮词定位,结合前句,可推知 B 选项。亚里士多德在古代世界因清晰的写
作风格被赞誉(首句)。高亮“writing”为他赢得这样的尊重(二句)。
3. 答案:E
题型分类:目的功能题
解析:根据高亮内容所处位置,可知是用来解释冒号前要点。

Passage 268
文章结构分析:文章结构分析:

105
首句(第 1 句黑体高亮句)陈述一个问题事实;过去几年该地区交通死亡数量增加。二
句归因;路况差,道路委员会基于该归因提出问题解决建议;多批钱修路。 ‘Many more’
句(第 2 句黑体高亮句)提出另一归因:交通堵塞。末句基于另一归因提出结论,也就
是替代解决问题方案:减少交通堵塞。
答案:D
题型分类:逻辑单题高亮句子功能题。
解析:根据第一句性质判断,可依据(position)排除 ABC 选项。D 选项最准确,2 句
黑体高亮句是用来反驳之前道路委员会提议解决方案的证据。

Passage 269
文章结构分析:
文章首句介绍历史事实:直到 17 世纪中期,新英格兰地区的本土美国人才开始首次饲
养牲畜。后文解释影响他们这一决策的因素:社会文化的挑战。黑体句是研究者发现的
影响本土美国人决策的挑战障碍。末句总结:障碍巨大,但他们找到了创造性克服障碍
的方法。
1. 答案:E
题型分类:句子功能题。
解析:根据文章结构,可知 E 选项最准确:该句内容是前句‘sociocultural challenge’
的具体举例解释。
B 选项‘more’概念不准确。文章讨论的是‘first’首次的问题,而非‘多少’的问题。
2. 答案:A
题型分类:推理题
解析:结合选项定位。
A 选项由黑体句前‘sociocultural challenge’及末句‘overcome’可推得。
BC 选项原文没有依据。

Passage 270
文章结构分析:文章结构分析:
文章首句,作者对经济学家的研究方式正评价:依据人们的实际选择决策行为而非声称
的意图,主观报告的喜欢不喜欢研究人们的倾向选择。
Yet 转折后,作者对首句方式提出质疑批评:人的理性有局限,会做出不一致选择,不
能从经验学习等等。
末句总结:人们不完全理性。
1. 答案:D
题型分类:目的功能题。
解析:依据文章结构,高亮内容出现在 yet 转折后,可知是用来反驳之前经济学家方法
的。
2. 答案:D
题型分类:词汇题
解析:由后文解释,可知‘mixed’在上下文中是‘不一致的,扭曲的,复杂的’意思。
D 最接近。

106
A 选项,ambiguous, 模糊的,歧义的,有双重解释的。

107
Passage271-Passage280 解析

Passage 271
文章结构分析:
首段主要是承认一个事实:美国妇女获得宪法赋予权利主要来自 1980-1990 年间的公民
权利条款的颁布和最高法院的判决。之前很长的历史时间没有进步。
二段首先对进步慢做出解释:州一级权力部门掌握实际权力,即使联邦宪法赋予妇女权
利,也被各州‘没收’了。直到 Reconstruction distinctions 明确联邦、州、公民的关系,
性别才进入联邦宪法的正式语言,而且对妇女地位有负面影响。
三段。基于联邦宪法演化史中性别角色的缺乏,作者对于研究集中于 1960-1990 表示理
解。这 30 年确实比之前的 170 年进步大。
四段转折。作者认为对妇女权利的历史研究少。 对历史背景的研究有助于加深妇女与
宪法关系的研究深度。
1. 答案: C
题型分类:主旨题
解析:由文章结构,可知二段、三段都是在解释首段事实。四段提出进一步研究的建议
方向。可知 C 选项概括最准确。
2. 答案: E
题型分类:细节题
解析:根据关键词 Reconstruction amendments 定位到二段末句:Only when….. negative
consequence 同义改写成 adversely affected.
3. 答案:D
题型分类:句子功能题
解析:考虑到第 4 段,A 选项‘clearly supports’不准确。
D 选项更准确,把首句当作研究的主张观点。2 段,3 段支持,4 段指出应扩大历史研究
背景。
4. 答案: B
题型分类:推理题
解析:B 选项根据关键词可定位到三段该句:Notwithstanding the defeat of the Equal
Rights Amendment helped women advanced farther toward achieving legal equality within
the 30 years before 1990 than in the preceding 170 years of concentrated efforts by the
Women’s Rights Project of the American Civil Liberties Union.

Passage 272
文章结构分析:
事实现象:昆虫捕食者通常能控制 A 在庄稼地的数量。但有时 A 会发生族群爆炸增长,
通常发生在有寒冷春天的年份。
解释:A 减少,捕食者数量下降,A 增长时,没有足够捕食者。
答案: E
题型:逻辑单题加强题
解析:如果如 E 选项,能有力加强解释:A 繁殖快,直接造成 population explosion.

Passage 273

108
文章结构分析:

文章介绍关于 Dunbar 诗歌成就评论观点的历史演化。


首句提出,Dunbar 是首位使用美国黑人乡村方言写作诗歌的人。
二句指出对 D 评价的变化:生前评价高,一战后评价负面。
三句(Some critics)解释负评价原因:社会现实角度和审美角度。
末句指出 1970 年之后 D 重新得到正评价。
1. 答案: B
题型:细节题
解析:B 选项定位到此句:Other critics, such as the post James Weldon Johnson, argued from
aesthetic grounds that dialect poetry in general was too limited as an artistic medium。。。
2. 答案: E

题型分类:推理题
解析:所在句提到,emphasize the importance of, 明显正评价,等于纠正了之前的负评
价。
3. 答案: C
题型分类:推理题
解析:literary critic’s evaluation 问题贯穿全文,使用选项定位。
C 选项可定位到末句:reverse itself in the 1970s, 开始有 psychological and historical
dimensions, 时间对比取非,可知之前的阶段没有。

109
Passage 274
文章结构分析:
首段指出特殊待研究现象:星际颗粒的形成。
二段介绍对星际颗粒形成的研究。目前只能依赖间接研究。通过观察颗粒扩散/吸收的
波长,可知其由岩石物质的核和有机化合物物的幔组成。科学家猜测:颗粒开始于成熟
恒星发射出的‘seedling’,之后其幔会有持续的物理和化学演化发生。
1. 答案: E
题型分类:目的功能题
解析:定位全文最后两句,可知作者用 seedling, 强调 grains 之后的 evolution,具有有
机物质的性质。
2. 答案: E
题型分类:类比题
解析:从文中过程:seedling+ mantle, mantle 有持续演化,可知 E 最相似。
3. 答案: B
题型分类:推理题
解析:从二段 to date 句可确定目前使用间接研究方法。

Passage 275
文章结构分析:
理由:爱伦坡所有书信中都没有提及吸毒。
结论:爱伦坡没有吸毒。
答案: C
题型分类:逻辑单题假设题
解析:
C 选项取非破环结论:他隐瞒了吸毒行为。这样,从书信没有提及就不能得出本人没有
吸毒行为的结论。

Passage 276
文章结构分析:
文章首先介绍作曲家 EV 术语创新和作曲实践创新。不被同代人所理解,EV 从中世纪
先辈寻找支持。从‘This seems appropriate’可知文章作者对于 EV 的正评价倾向。
1. 答案: C
题型分类:推理题
解析:定位到该句:This seems appropriate, for the medieval composers shared his view of
music as a technical unfitting of sound.
2. 答案: D
题型分类:目的题
解析:定位关键词‘howling sirens’所在主句中有: bold sonic experiment.

Passage 277

110
文章结构分析:
首句总体概括:两次世界大战期间以美国妇女角色定义扩大为特征。二句、三句提供支
持首句事实证据。While 句转折,后期趋势减慢。末句提供趋势减慢原因解释:经济危
机激发传统价值观。
1. 答案:B
题型分类:目的题
解析:‘Divorce rates’位于三句。由二句、三句对首句的支持关系,可知 B 选项正确。
2. 答案:A C
题型分类:推理题
解析:
A 定位:a small but growing minority of them continued working after marriage.
C 定位:While these trends continued throughout the interwar period…… 由 throughout 可
知 from the beginning.

Passage 278
文章结构分析:
首段介绍关于尾状 F 结构在细菌学家中自 19 世纪以来的争论。在 F 结构到底是有用运
动器官还是无用附属器官之间,主流细菌学家支持前者,主要证据:多数运动的细菌物
种有 F,不运动的细菌物种没有 F。AP1946 年的论文挑战主流共识。AP 认为主流观点
可能颠倒 F 结构和运动之间的因果关系。
二段介绍关于 F 的新争论的影响。对于分类学,AP 认为 F 不重要,而分类学将其作为
分类标志,必然导致分类学的混乱。对于新技术在研究中的地位作用。AP 的研究否定
其作用,认为活细胞的直接观察更重要。
三段基于后来的证据重新看当时的争论。作者认为,在该争论中,AP 的反对者对于新
电子技术的信念很重要。争论的双方对于生物学的基本假设不同,把个人的、职业的因
素绑定到特定的实验方法上。
1. 答案:A
题型分类:推理题
解析:由 2 段末 P 对于活体细胞观察方法的主张(…and that priority should be given to
observations made on living cells)可知 A 选项正确。
2. 答案:C
题型:细节题
解析:依据首段,P 的观察证据: His observations suggested that the flagella must be merely
twirls of polysaccharide material that, because of the active gyration of the cell body, spun out
behind the cell as it swan.。
3. 答案:E
题型分类:细节题
解析:依据首段,reviving an old theory……His observation….
4. 答案:A
题型分类:推理题
解析:题目问 AP 和反对者会同意的点。依据文章结构,三段对于他们争论的讨论是基
于后来的证据。由‘But at the time of the controversy, the evidence was still ambiguous’,
可确定 A 选项。

111
Passage 279
文章结构分析:
采用对比结构,文章对比了当代学者和汉代学者对于汉代妇女工作角色的不同看法。当
代学者认为:汉代妇女要做多种日常工作。汉代学者只强调妇女的织布工作。
作者支持当代学者的观点,认为当时只织布的妇女会被认为不负责任。
1. 答案:A C
题型分类:细节题
解析:A 依据:singing out cloth production as the most important job
C 依据: linking it to the concept of successful womanhood.
2. 答案:E
题型分类:推理题
解析:由文章末句作者评论,认为汉代作家描绘与当时实际情况不一致,可得 E 选项。

Passage 280
文章结构分析:
文章描述商业对于 18 至 19 世纪英国中产阶级男性社会身份角色的影响。
中产阶级开始采用工匠和上层社会管理财富的方式。男性开始有独立于家庭的工作场
所,基于经济能力、职业的男性身份出现。男性更多以从事的工作,而非之前的宗教或
家庭关系作为身份的标志。
1. 答案:C
题型:细节题
解析:定位到该句:centered on economic skills and increasingly identified with occupation.
2. 答案:D
题型:加强题
解析:通过对比,文章作者体现的核心观点是:男人身份在此阶段和职业建立紧密联系。
D 选项是体现该联系的证据。
3. 答案:A
题型:组织结构题
解析:文章首先介绍一个趋势,介绍该趋势变化的影响,最后做出结论。

112
Passage281-Passage290 解析

Passage 281
文章结构分析:
首句介绍新技术:新的海下隧道越来越依赖于沉浸式管道技术。
后文主要对比突出新技术比之前技术具有的优点。
1. 答案:C
题型:句子功能题
解析:根据文章结构,黑体句处于详细介绍新技术优势部分,用于支持前句作者对新技
术优势的判断。
2. 答案:B C
题型分类:推理题
解析:‘bored tunnels’就是新技术的对比对象。
B 选项定位:while bored tunnels are usually considered stable only if their depth under the
seabed at least equals their diameter
C 选项定位:because immersed tubes can have a rectangular cross-section, they are a more
efficient shape than……由此句,新技术取代 bored tunnels 技术,新技术能,可知 bored
tunnels 技术不能。

Passage 282
文章结构分析:
首句介绍现象:20 世纪末,概念艺术兴盛,艺术构思过程被认为比艺术作品本身更重要。
二句重复支持首句,三句列举具体的‘短期’艺术。四句解释“短期”艺术兴盛的原因。
However 句及之后,作者提出自己观点,认为该流行趋势过分了,持久艺术仍有价值。
答案:第 3 句:Performance, installations, body art, and earthworks became especially
fashionable, since they did not result in a permanent work of art.
题型分类:特定功能句子选择题
解析:transient art 概念虽然在后句出现,其应该是对前句例子的概括。

Passage 283
文章结构分析:
文章首句提出观点:商业领袖应该欢迎环保市场。But 转折后,2 句说术语混乱导致该
市场被忽略,3 句(高亮句)说,商业领袖通常把环保看作成本。Yet 转折后,作者引
用 JQ 再次强调自己观点:环保应该被看作市场机遇而不是成本。
1. 答案:A
题型分类:句子功能作用题
解析:由文章结构分析可知,3 句是与首句(作者观点)相反的事实。
2. 答案:A
题型分类:词汇题
解析:Normall, 正常情况,自然情况。

Passage 284
文章结构分析:

113
首段提出哈勒姆复兴与爱尔兰复兴的可比性。
二段首先指出哈勒姆复兴与爱尔兰复兴的一个相似性:语言都被用来当作保留民族文化
的工具。Nevertheless 转折后,指出语言不是两个复兴所关注的重点。John Synge, James
Weldon Johnson 分别是两个复兴的代表,都指出复兴在语言之外的意义。
三段指出两个复兴面临的共同困难:主流文化拒绝承认他们文化传承的合理性。但两个
复兴解决此困难的方法不同。爱尔兰作家通过复活爱尔兰英雄传说的方式。但哈勒姆作
家缺乏可利用的英雄传说资源,他们转而强调利用故土非洲资源。两个复兴怀旧的对象
不同:爱尔兰复兴怀旧的是被分离的文化,哈勒姆复兴怀旧的是被分离的土地。
1. 答案:D
题型分类:细节题
解析:关键词 Irish heroic legends 定位到第三段二句,所述行为针对 dilemma: 主流文化
拒绝合法性,爱尔兰作家争取的是文化传统的合法性。
2. 答案:B
题型分类:推理题
解析:distinctive speech patterns 定位到二段首句:distinct discernible “voice”. B 选项依
据 Nevertheless 转折后判断:两个群体的作家首要关注的都不是语言方面。
3. 答案:E

题型分类:目的题
解析:人名定位到二段。Nevertheless 转折后,作者用 John Synge, James Weldon Johnson
举例,分别代表爱尔兰和哈勒姆对语言表现的态度。而两个群体对语言表现的态度是一
致的:主要兴趣都不在语言方面。
4. 答案:C
题型分类:目的题
解析:根据人名定位到末段,定位该句,结合段落结构理解可知是举例说明非非裔美国
作家对于非裔美国传说的改编处理。

Passage 285
文章结构分析:
史前人类把狼驯化成狗。
考古证据说明,欧洲在 14000 年之前没有狗。
狗是在 13500 年前被人类驯化成独立物种的。
答案:B
题型分类:逻辑单题推理题
解析:狼在 13.5 万年前被驯化成狗,欧洲 1.4 万年前没有狗。合理推论:狗是在其他地
方被驯化的。(利用逆否命题:如果狗在欧洲驯化,欧洲在 14000 年之前应该有原始的
狗,欧洲没有原始的狗,狗不是在欧洲驯化的。)

Passage 286
文章结构分析:

114
首段提出河流研究乃至一般科学研究共通的分析问题的方式:标准化的收集分析数据的
方法。作者承认该方法的优点:不同地点和时期的可比性。但作者认为该方法的问题是:
预先设定的问题种类的框架会失去新观察的机会。
二段,作者举出河流研究的例子,河流研究科学家过于关注沉淀物移动,忽略沉淀物的
储存。作者引用 Meade 的观点: 如果能关注到个体沉淀物颗粒的移动,或许就能避免
上述研究的缺陷。
1. 答案:首段第二句:This standardized way of……
题型分类:句子功能选择
解析:根据文章结构,首句提出的研究方法在后文被作者质疑。二句是作者承认该方法
的好处,benefit.
2. 答案 D
题型分类:推理题
解析:根据人名定位到二段第二句,根据该句句末 rather than 部分(rather than on the
process of mobilizing sediment)可确定 D 选项。
3. 答案:E
题型分类:推理题
解析:根据二段,作者说颗粒在运动过程时间与静止时间相比很小,借 M 之口说相关
科学家研究集中于运动阶段,可知 E 选项:对移动沉淀物研究的关注度与沉淀物移动的
时间不成比例。

Passage 287
文章结构分析:
首句指出矛盾现象:现在更高效率的发电机比爱迪生时代的发电厂浪费更多能量。
后文提出原因解释:爱迪生时代用发电产生的蒸汽为附近的家庭和工厂提供暖气,而现
代的发电厂很少利用剩余热量。作者进一步解释说,大型发电厂需要更多土地,只能建
造在远离客户的地方,而热量不适合长距离运输。
1. 答案:C
题型分类:推理题
解析:文章提到,为了利用更便宜的能源,现代电厂远离人口中心,意思是,靠近人口
中心,燃料会更贵。
2. 答案:D
题型分类:推理题
解析:通过对比,早期电厂距离人口中心近,提供热力。现代电厂距离人口中心远,浪
费掉热力,意思是附近没有足够需求。

Passage 288
文章结构分析:
首段指出小说作为文学体裁的特殊性:反映个性化独特性的趋势。通过比较的方式,作
者指出,之前的文学作品都是反映文化的整体趋势要求,而小说强调个人经验,强调原
创性。
二段从形式方面强调小说的独特性:其他文体适合从文学结构、文学模版的角度分析评
论,但小说不适合文学模版分析。

115
三段、四段用文学发展史上具体具体例子的比较展开二段小说的独特性。Defoe 和
Richardson 是作者认为的最早的不模仿之前文体的作家。Balzac 是作者认为的 19 世纪
关注现实个人经验的小说的代表。
1. 答案:A
题型分类:细节题
解析:利用关键词 formlessness 定位到二段末句,由其前句…convey the impression of
fidelity to human experience…得到 A 选项。
2. 答案:C
题型分类:细节题
解析: 由关键词 ode 定位到二段末句。 ode 所属 previous literary forms……take
由三段对比,
their plots from mythology….
3. 答案:A
题型分类:目的题
解析:根据文章结构,三段讨论小说与传统文学形式的一个重要区别:是否从之前文本
获得 plot.
4. 答案:D
题型分类:细节题
解析:由人名关键词定位三段末:they accepted the general promise….

Passage 289
文章结构分析:
理由:年度吸烟健康成本在禁烟运动后占政府年度花费比例上升。
结论:禁烟运动失败。
答案:D
题型分类:逻辑单题假设题
解析:根据文章结构,我们尝试破环结论,如果政府年度花费大幅减少,则不能从比例
的增加得出禁烟运动失败的结论。D 选项否定了破环结论的可能性,因此是论证需要的
假设。

Passage 290
文章结构分析:
首句总体论断观点:生物人类学的发现说明土著美洲人可能来自东北亚地区。
Turner 发现概括的 S1 综合牙齿特征分布于大多数本土美洲人和东北亚人中,但没有分
布于亚洲、非洲、欧洲等其他地区人口。尽管更简化的 S2 牙齿特征存在于东南亚和本
土美洲人族群,但 Turner 认为 S1 特征能证明本土美洲人来自东北亚。
1. 答案:B C
题型分类:推理题
解析:B 选项定位:… Sinodonty, which includes three-rooted lower first molars and
shovel-shaped incisors.
C 选项定位: … but is not found in people who originated in southern Asia, Africa, or Europe.
2. 答案:B C
题型分类:细节题

116
解析:B 选项定位:Another less complex pattern—Sundadonty…
C 选项定位:Sinodonty: Turner, …prehistoric and living Native Americans …,
Sundadonty 只提到 prehistoric American populations.

117
Passage291-Passage300 解析

Passage 291
文章结构分析:
首段提出主题:对 MB 的传记作品:A Biographical Dictionary of the Celebrated Women of
Every Age and Country (1804)的评论。作者认为其对女权主义的贡献应该得到更多关注。
尽管 MB 的研究不是唯一的,但她研究关注了几位非传统的政治激进的作家,结合当时
的时代背景能显示她的研究对女性角色的影响。
二段展开对 MB 研究方法的介绍及评论。MB 使用经验传记和独立分析两种方法。她的
研究的二元性(主要依据权威来源,加入自己的评论批评)导致了争论。MBE 及几位
早期二十世纪的评论者对其正评价,但最近的评论者负评价。文章作者认为,两者都有
道理。对其评论的矛盾性首先反映了对女权主义作家作品评价标准的变化。另外,对其
评论的矛盾性也是因为 MB 的研究方法:她既遵从又颠覆主流历史观点以及对女性的定
义。
1. 答案:D
题型分类:主旨题
解析:结合文章结构分析,可知作者介绍解释关于 B 的矛盾观点。
2. 答案:B
题型分类:推理题
解析:结合文章结构定位原句:Though Katharine Antony praised the Dictionary’s
encyclopedic scope and bold subject matter as early as 1948…,可知 B 选项正确。
3. 答案:C
题型分类:目的题
解析:结合文章结构,定位该句:The other reason for….,可知是作者用来解释对 MB
矛盾评价的理由。
4. 答案:A
题型分类:推理题
解析:由 from…. to a more cautious….complexities…..结构,题干定位内容出现于 from….
位置,可知不如后来的微妙。A 选项正确。

Passage 292
文章结构分析:
首句提出待解释现象:黄色消防车的事故率低于传统的红色消防车。二句是一些评论者
对于可能解释结论(首句黑体高亮句)的质疑,理由是黄色消防车比红色消防车总体上
更新。But 句体现作者立场:质疑评论者观点。Certainly 句(第二黑体高亮句)是作者
让步,承认事故率的区分不是决定性证据。之后,作者指出关键性事故率区分只在清晨
和黄昏:黄色比红色有更高可视度的时间。
答案:E
题型分类:句子功能题
解析:作者结论句:But this criticism can be dismissed. 从结论句可知第一黑体句是作者
立场。第二句是可以用来支持批评者立场的证据,可知是作者的承认让步。

Passage 293

118
文章结构分析:
文章是关于 RJ 在 A Fine Brush on Ivory 作品中体现的关于 Austen 研究观点的介绍。RJ
认为,在对 Austen 的研究中存在破坏正常的评论的压力。主要的压力是读者对 Austen
的特殊喜爱。Austen 能引导评论者从传记或历史的角度看待她的作品,而不是现代文学
评论主导的正式方式。
1. 答案:B
题型分类:细节题
解析:B 选项依据:often in defiance of other critical fashions…
2. 答案:C
题型分类:词汇题
解析:从后文可知,appreciation of 此处指评论,鉴别。

Passage 294
文章结构分析:
文章是关于流行观点(1962 年前的联邦法院在选举政治领域是无力的)的评价。二句
(This perception)用 S v.A 案例说明该观点不完全正确。However 句支持该观点,S v.A
案例仅仅是例外。Indeed 句进一步支持该观点,作者用考古学家分层的发现,说明联邦
法院在该时期的选举政治领域是逐渐进步的,经历了观念的社会的变化积累。
1. 答案:C
题型分类:目的题
解析:定位到末句:in reviewing voters’ rights cases over the intervening decades, one feels
like an archaeologist cutting through distinct layers in which the judicial decisions uncovered
reveal a pattern of ideological and societal change.
C 选项同义改写,该比喻用来说明变化的渐进性质。
2. 答案:B
题型分类:推理题
解析:定位首句:Federal courts in the United States, especially before the famous 1962 case
of Baker v. Carr, were often thought to be powerless in the area of election law, voting rights,
and other legal questions clearly bearing on politics.
B 选项是原句的时间对比取非。

Passage 295
文章结构分析:
现象 1:热带森林比类似面积的温带森林有更多物种。
现象 2: 在冰川期,温带森林被破坏,热带森林没有被破坏。
解释:热带森林有更长的时间养育多样物种。
答案:D
题型分类:逻辑单题加强题
解析:对比加强:原文(热带周期长物种多)/选项 D(周期短的热带物种也不多)

Passage 296

119
文章结构分析:
首句(黑体高亮句)介绍关于 RBS 的发现:RBS 在高酸度地区稀少,个头小。二句解
释:酸土影响幼年 RBS 发育,或者捕食对象族群受酸影响导致 RBS 数量少。Yet 句指
出矛盾例外:高酸度的 LCW 地区发现高密度的大个头 RBS。之后是对矛盾现象的解释:
RBS 存在物种内部对于酸度的适应性。
1. 答案:E
题型分类:推理题
解析:由后文 Lake Claire Watershed 的“where soil conditions are acidic”,作者以其作
为例外反驳之前研究,可知前后研究土壤条件一致。
2. 答案:D
题型分类:目的题
解析:由文章结构分析,作者引入矛盾例外现象,高酸度的 LCW 地区发现高密度的大
个头 RBS,然后提出进一步解释。可知 D 选项对作者目的概括最准确。
A 选项没有概括 LCW 之后的解释。

Passage 297
文章结构分析:
文章首句介绍现象:美国妇女 1920 年赢得选举权。二句(Yet 句)是作者评论观点:选
举权对妇女地位的影响比女权活动家预计的要小。三句是作者用来支持自己观点的理
由:妇女是在选举不再是重要政治活动的时期赢得选举权的。后两句用对比方式支持三
句理由:19 世纪中期,妇女开始争取选举权的时候,投票率高,选举竞争激烈,政党重
要。1929 年代,妇女获得选举权的时候,选举政治竞争不再激烈,很多地区单一政党主
导,投票率降到历史低点。末句(黑体高亮句),作者做让步转折:妇女选举权的获得
对于希望得到选举权的妇女,对于希望限制妇女选举权的妇女仍然有意义。
1. 答案:B
题型分类:目的题
解析:定位可知,该内容出现在支持第三句的对比细节,作者强调前后情况不同,继而
用情况不同来支持第三句的理由。
2. 答案:C
题型分类:目的题
解析:定位末句,作者此处让步,妇女选举权对一些人仍有意义。

Passage 298
文章结构分析:
文章首先用对比方式说明 HL 摄影与 WE 肖像画的不同:WE 的肖像画对象直视镜头,
共同参与画作,而 HL 的摄影对象都是无意识的自然的。But…paradox 句是作者对 HL
作品矛盾性质的判断。后句用来支持其矛盾性判断:表面上客观,实际上主观。
1. 答案:C
题型分类:细节题
解析:根据 objectivity 定位该句:Her off-the-cuff aesthetic seemingly guarantees objectivity,
since she was recording street scenes she happened upon, 结合前句对比,可知 C 选项正确。
2. 答案:B

120
题型分类:细节题
解析:定位到该句:Whereas Evans’ subjects look directly into the camera, so that
photographer and subject conspire in the making of a portrait…可确定 B 选项。Collaborators
改写 conspire.
3. 答案:E
题型分类:推理题
解析:根据关键词“street photography”定位到该句:As a “street” photographer, before the
term’s invention, Levitt has claimed to…可确定 E 选项正确。

Passage 299
文章结构分析:
首句和二句(However 句)指出法律界把科学当作能解决法律争论的客观知识对象的趋
势及问题:对科学理想化,忽略科学本身的不稳定性争论性。
后句展开对科学理想化的不同方面及其问题:
理想化表现 1: 两个科学家争论,必定有一个是“垃圾科学家”。这会忽略能导致真正
科学争论的数据本身的局限。
理想化表现 2: 把“偏见、利益、动机”等当作天生不靠谱的。这会忽略有强烈理论偏
见、团体利益、金钱动机的科学家所做的实质性贡献。
理想化表现 3: 立法者、行政官员、法官总是要求科学提供确定性。这会忽略科学本身
的不确定性及动态历史。
最后,作者主张正视科学争论,科学方法的局限,科学家关系网,科学的出版赞助等社
会资源,科学论文的劝说性质等等。把科学的这些特征当作坏科学的代表就是对科学的
理想化。
1. 答案:A
题型分类:主旨题
解析:根据文章结构分析,本文作者主要讨论对科学的理想化看法的表现及其问题。A
选项概括最准确。
2. 答案:D
题型分类:推理题
解析:根据关键词“junk scientist”定位到该句:We see the idealization of science in law
whenever there is a presumption that if two scientific experts disagree, one of them must be a
“junk scientist”.
由该句可知,争论导致一方被当作“junk scientist”,可知理想化观点是科学不应该有争
论,能解决分歧,达成一致结论。D 选项准确概括。
3. 答案:C
题型分类:目的题
解析:定位可知,This association missed the practical advances made by scientists who have
strong theoretical biases, institutional interests, and financial motivations.
可知,这些“不好的”科学家也能做出真正贡献。C 选项准确。

Passage 300

121
文章结构分析:
文章首句陈述事实类信息:图画设计只有在二战后才成为独立领域,多数设计收藏定义
狭隘。二句用更多事实信息支持首句。三句(黑体高亮句)对首句、二句给出总结评价:
有作用,但过于孤立狭隘。四句(In contrast 句)用对比方式介绍 MB 不同的实践:广
泛,混合。
答案:A
题型分类:句子功能题
解析:根据文章结构分析,高亮句对之前句子陈述的事实信息给出概括评价。

122
KMF 考满分 GRE 课程简介

ViS GRE 全程班

以 GRE 历年考题为核心,按科目(填空、阅读、写作、数学)全面、系统地讲
授知识点、解题方法、应试技巧,同时辅以大量练习来巩固。最终达成帮助学员
一站式解决问题、考出高分的效果。课程形式为直播+录播+作业,主讲答疑+辅
导监督。基于考满分独家课程系统,实现了学习过程中的可见(visible)、互动
(interactive)、智能(smart)。极大提升学习效果。
¥ 1,999/科

ViS GRE 12 人冲分班

课程以可见(Visible),互动(Interactive),智能(Smart)为教学理念,以“分
层难度教学,真题引导带练”为教学核心,大数据智能筛选真题中的核心题目,
主讲陪伴式教学督导,实现真题互动式速练,短期冲分更有效。
¥ 5,888/科

ViS GRE 一对一

专属备考方案/ 双师督导服务/ 智能互动系统


¥ 8,000/科

更多课程优惠详情,请访问 course.kmf.com

123

You might also like